Articulate Hindi December 2022

You might also like

Download as pdf or txt
Download as pdf or txt
You are on page 1of 101

दिसंबर, 2022

प्रबोधन
Unacademy की एक पहल

UPSC CSE हेतु मासिक करंट अफेयर्स

बिक्री हेतु नहीं


Download Monthly Articulate Magazine प्रबोधन | 1
विषय-सूची

राजव्यवस्था . . . . . . . . . . . . . . . . . . . . . . . . . . . . . . . . . . . . . . . . . . . . . . . . . . . . . . . . . . . . . . . . . . . . . . . . . . . . . . . . . . . . . . . . . 7

अधिकार सम्बन्धी मुद्दे . . . . . . . . . . . . . . . . . . . . . . . . . . . . . . . . . . . . . . . . . . . . . . . . . . . . . . . . . . . . . . . . . . . . . . . . . . . . . . . . . . . . . . . . . . . . . . . . . . . . . . . . . . . . 8

• ओबीसी उप-वर्गीकरण . . . . . . . . . . . . . . . . . . . . . . . . . . . . . . . . . . . . . . . . . . . . . . . . . . . . . . . . . . . . . . . . . . . . . . . . . . . . . . . . . . . . . . . . . . . . . . . . . . . . . . . . 8

• समलैं गिक विवाह को वैध बनाना . . . . . . . . . . . . . . . . . . . . . . . . . . . . . . . . . . . . . . . . . . . . . . . . . . . . . . . . . . . . . . . . . . . . . . . . . . . . . . . . . . . . . . . . . . . . . 9

• रिकॉर्ड्स में थर्ड जेंडर विकल्प . . . . . . . . . . . . . . . . . . . . . . . . . . . . . . . . . . . . . . . . . . . . . . . . . . . . . . . . . . . . . . . . . . . . . . . . . . . . . . . . . . . . . . . . . . . . . . . . . . 9

• व्यक्तित्व अधिकार . . . . . . . . . . . . . . . . . . . . . . . . . . . . . . . . . . . . . . . . . . . . . . . . . . . . . . . . . . . . . . . . . . . . . . . . . . . . . . . . . . . . . . . . . . . . . . . . . . . . . . . . . . . . 9

संघीय मुद्दे . . . . . . . . . . . . . . . . . . . . . . . . . . . . . . . . . . . . . . . . . . . . . . . . . . . . . . . . . . . . . . . . . . . . . . . . . . . . . . . . . . . . . . . . . . . . . . . . . . . . . . . . . . . . . . . . . . . . . . . . 10

• असम-मेघालय सीमा विवाद . . . . . . . . . . . . . . . . . . . . . . . . . . . . . . . . . . . . . . . . . . . . . . . . . . . . . . . . . . . . . . . . . . . . . . . . . . . . . . . . . . . . . . . . . . . . . . . . . 10

• लद्दाख संघ राज्य क्षेत्र को छठी अनुसूची में शामिल करने की मांग . . . . . . . . . . . . . . . . . . . . . . . . . . . . . . . . . . . . . . . . . . . . . . . . . . . . . . . . . . . . . . . 11

• ग्रेटर टिपरालैं ड . . . . . . . . . . . . . . . . . . . . . . . . . . . . . . . . . . . . . . . . . . . . . . . . . . . . . . . . . . . . . . . . . . . . . . . . . . . . . . . . . . . . . . . . . . . . . . . . . . . . . . . . . . . . . . 13

संवैधानिक मुद्दे/संसद/न्यायपालिका . . . . . . . . . . . . . . . . . . . . . . . . . . . . . . . . . . . . . . . . . . . . . . . . . . . . . . . . . . . . . . . . . . . . . . . . . . . . . . . . . . . . . . . . . . . . . . 14

• अधिकारों का विभाजन . . . . . . . . . . . . . . . . . . . . . . . . . . . . . . . . . . . . . . . . . . . . . . . . . . . . . . . . . . . . . . . . . . . . . . . . . . . . . . . . . . . . . . . . . . . . . . . . . . . . . . 14

• नियम 267 . . . . . . . . . . . . . . . . . . . . . . . . . . . . . . . . . . . . . . . . . . . . . . . . . . . . . . . . . . . . . . . . . . . . . . . . . . . . . . . . . . . . . . . . . . . . . . . . . . . . . . . . . . . . . . . . . . 15

• न्यायालयों में अवकाश . . . . . . . . . . . . . . . . . . . . . . . . . . . . . . . . . . . . . . . . . . . . . . . . . . . . . . . . . . . . . . . . . . . . . . . . . . . . . . . . . . . . . . . . . . . . . . . . . . . . . . . 15

• समीक्षा याचिका . . . . . . . . . . . . . . . . . . . . . . . . . . . . . . . . . . . . . . . . . . . . . . . . . . . . . . . . . . . . . . . . . . . . . . . . . . . . . . . . . . . . . . . . . . . . . . . . . . . . . . . . . . . . . 16

चुनाव. . . . . . . . . . . . . . . . . . . . . . . . . . . . . . . . . . . . . . . . . . . . . . . . . . . . . . . . . . . . . . . . . . . . . . . . . . . . . . . . . . . . . . . . . . . . . . . . . . . . . . . . . . . . . . . . . . . . . . . . . . . . .17

• मुख्य चुनाव आयुक्त . . . . . . . . . . . . . . . . . . . . . . . . . . . . . . . . . . . . . . . . . . . . . . . . . . . . . . . . . . . . . . . . . . . . . . . . . . . . . . . . . . . . . . . . . . . . . . . . . . . . . . . . . . 17

• परिसीमन आयोग . . . . . . . . . . . . . . . . . . . . . . . . . . . . . . . . . . . . . . . . . . . . . . . . . . . . . . . . . . . . . . . . . . . . . . . . . . . . . . . . . . . . . . . . . . . . . . . . . . . . . . . . . . . . 17

• एक राष्ट्रीय पार्टी की स्थिति . . . . . . . . . . . . . . . . . . . . . . . . . . . . . . . . . . . . . . . . . . . . . . . . . . . . . . . . . . . . . . . . . . . . . . . . . . . . . . . . . . . . . . . . . . . . . . . . . . 18

प्रशासनिक मुद्दे. . . . . . . . . . . . . . . . . . . . . . . . . . . . . . . . . . . . . . . . . . . . . . . . . . . . . . . . . . . . . . . . . . . . . . . . . . . . . . . . . . . . . . . . . . . . . . . . . . . . . . . . . . . . . . . . . . . 19

• परिवार पहचान पत्र . . . . . . . . . . . . . . . . . . . . . . . . . . . . . . . . . . . . . . . . . . . . . . . . . . . . . . . . . . . . . . . . . . . . . . . . . . . . . . . . . . . . . . . . . . . . . . . . . . . . . . . . . . 19

• घर पोर्टल . . . . . . . . . . . . . . . . . . . . . . . . . . . . . . . . . . . . . . . . . . . . . . . . . . . . . . . . . . . . . . . . . . . . . . . . . . . . . . . . . . . . . . . . . . . . . . . . . . . . . . . . . . . . . . . . . . .20

• प्रसारण चैनलों के लिए प्रोग्राम कोड . . . . . . . . . . . . . . . . . . . . . . . . . . . . . . . . . . . . . . . . . . . . . . . . . . . . . . . . . . . . . . . . . . . . . . . . . . . . . . . . . . . . . . . . . .20

• प्रिं ट और डिजिटल मीडिया एसोसिएशन . . . . . . . . . . . . . . . . . . . . . . . . . . . . . . . . . . . . . . . . . . . . . . . . . . . . . . . . . . . . . . . . . . . . . . . . . . . . . . . . . . . . . . .20

• 'नई चेतना' अभियान . . . . . . . . . . . . . . . . . . . . . . . . . . . . . . . . . . . . . . . . . . . . . . . . . . . . . . . . . . . . . . . . . . . . . . . . . . . . . . . . . . . . . . . . . . . . . . . . . . . . . . . . . 21

• एसिड अटै क . . . . . . . . . . . . . . . . . . . . . . . . . . . . . . . . . . . . . . . . . . . . . . . . . . . . . . . . . . . . . . . . . . . . . . . . . . . . . . . . . . . . . . . . . . . . . . . . . . . . . . . . . . . . . . . . . 21

• पाकिस्तान में आत्महत्या को अपराध की श्रेणी से बाहर किया गया . . . . . . . . . . . . . . . . . . . . . . . . . . . . . . . . . . . . . . . . . . . . . . . . . . . . . . . . . . . . .22

शीतकालीन सत्र बिल 2022 . . . . . . . . . . . . . . . . . . . . . . . . . . . . . . . . . . . . . . . . . . . . . . . . . . . . . . . . . . . . . . . . . . . . . . . . . . . . . . . . . . . . . . . . . . . . . . . . . . . . . .23

• नई दिल्ली अंतर्राष्ट्रीय मध्यस्थता केंद्र (संशोधन) विधेयक 2022 . . . . . . . . . . . . . . . . . . . . . . . . . . . . . . . . . . . . . . . . . . . . . . . . . . . . . . . . . . . . . . .23

» नई दिल्ली अंतर्राष्ट्रीय मध्यस्थता केंद्र (एनडीआईएसी) . . . . . . . . . . . . . . . . . . . . . . . . . . . . . . . . . . . . . . . . . . . . . . . . . . . . . . . . . . . . . . . . . . . . . . . .24

• जन विश्वास (प्रावधानों का संशोधन) विधेयक 2022 . . . . . . . . . . . . . . . . . . . . . . . . . . . . . . . . . . . . . . . . . . . . . . . . . . . . . . . . . . . . . . . . . . . . . . . . . . .24

• एं टी-मैरीटाइम पाइरेसी बिल 2019-2022 . . . . . . . . . . . . . . . . . . . . . . . . . . . . . . . . . . . . . . . . . . . . . . . . . . . . . . . . . . . . . . . . . . . . . . . . . . . . . . . . . . . . .26

• ऊर्जा संरक्षण (संशोधन) विधेयक, 2022 . . . . . . . . . . . . . . . . . . . . . . . . . . . . . . . . . . . . . . . . . . . . . . . . . . . . . . . . . . . . . . . . . . . . . . . . . . . . . . . . . . . . . .26

• संविधान (अनुसूचित जाति और अनुसूचित जनजाति) आदेश (संशोधन) विधेयक 2022. . . . . . . . . . . . . . . . . . . . . . . . . . . . . . . . . . . . . . . . . . 27

2 I प्रबोधन Download Monthly Articulate Magazine


अर्थव्यवस्था . . . . . . . . . . . . . . . . . . . . . . . . . . . . . . . . . . . . . . . . . . . . . . . . . . . . . . . . . . . . . . . . . . . . . . . . . . . . . . . . . . . . . . . 28

फर्स्ट लॉस डिफॉल्ट गारंटी(FLDG) . . . . . . . . . . . . . . . . . . . . . . . . . . . . . . . . . . . . . . . . . . . . . . . . . . . . . . . . . . . . . . . . . . . . . . . . . . . . . . . . . . . . . . . . . . . . . . . .29

आरबीआई ने यूसीबी के लिए 4-स्तरीय नियामक ढांचे की घोषणा की . . . . . . . . . . . . . . . . . . . . . . . . . . . . . . . . . . . . . . . . . . . . . . . . . . . . . . . . . . . . . .29

सेंट्रल डिपॉजिटरीज सर्वि सेज इं डिया लिमिटेड . . . . . . . . . . . . . . . . . . . . . . . . . . . . . . . . . . . . . . . . . . . . . . . . . . . . . . . . . . . . . . . . . . . . . . . . . . . . . . . . . . . . . .30

राज्य वित्त और ऋण स्तर . . . . . . . . . . . . . . . . . . . . . . . . . . . . . . . . . . . . . . . . . . . . . . . . . . . . . . . . . . . . . . . . . . . . . . . . . . . . . . . . . . . . . . . . . . . . . . . . . . . . . . . . .32

पेरिस क्लब . . . . . . . . . . . . . . . . . . . . . . . . . . . . . . . . . . . . . . . . . . . . . . . . . . . . . . . . . . . . . . . . . . . . . . . . . . . . . . . . . . . . . . . . . . . . . . . . . . . . . . . . . . . . . . . . . . . . . .33

भौगोलिक संकेत (जीआई) स्थिति . . . . . . . . . . . . . . . . . . . . . . . . . . . . . . . . . . . . . . . . . . . . . . . . . . . . . . . . . . . . . . . . . . . . . . . . . . . . . . . . . . . . . . . . . . . . . . . . .33

रोडटे प योजना . . . . . . . . . . . . . . . . . . . . . . . . . . . . . . . . . . . . . . . . . . . . . . . . . . . . . . . . . . . . . . . . . . . . . . . . . . . . . . . . . . . . . . . . . . . . . . . . . . . . . . . . . . . . . . . . . . .34

राइट टू रिपेयर . . . . . . . . . . . . . . . . . . . . . . . . . . . . . . . . . . . . . . . . . . . . . . . . . . . . . . . . . . . . . . . . . . . . . . . . . . . . . . . . . . . . . . . . . . . . . . . . . . . . . . . . . . . . . . . . . . .36

भारत की विमानन सुरक्षा रैंकिंग . . . . . . . . . . . . . . . . . . . . . . . . . . . . . . . . . . . . . . . . . . . . . . . . . . . . . . . . . . . . . . . . . . . . . . . . . . . . . . . . . . . . . . . . . . . . . . . . . . 37

दुर्ल भ मृदा तत्त्व . . . . . . . . . . . . . . . . . . . . . . . . . . . . . . . . . . . . . . . . . . . . . . . . . . . . . . . . . . . . . . . . . . . . . . . . . . . . . . . . . . . . . . . . . . . . . . . . . . . . . . . . . . . . . . . . . . 37

किफायती और मध्यम आय वाले आवास के लिए विशेष खिड़की (SWAMIH). . . . . . . . . . . . . . . . . . . . . . . . . . . . . . . . . . . . . . . . . . . . . . . . . . . . . . . 38

रिटर्न की संशोधित आर्थि क आंतरिक दर (एमईआईआरआर) मॉडल . . . . . . . . . . . . . . . . . . . . . . . . . . . . . . . . . . . . . . . . . . . . . . . . . . . . . . . . . . . . . . . 38

प्रधानमंत्री ग्रामीण आवास योजना . . . . . . . . . . . . . . . . . . . . . . . . . . . . . . . . . . . . . . . . . . . . . . . . . . . . . . . . . . . . . . . . . . . . . . . . . . . . . . . . . . . . . . . . . . . . . . . .39

प्रधानमंत्री गरीब कल्याण अन्न योजना (पीएमजीकेएवाई) और राष्ट्रीय खाद्य सुरक्षा अधिनियम (एनएफएसए) . . . . . . . . . . . . . . . . . . . . . . .39

छात्रों को सरकार की प्री-मैट्रिक छात्रवृत्ति . . . . . . . . . . . . . . . . . . . . . . . . . . . . . . . . . . . . . . . . . . . . . . . . . . . . . . . . . . . . . . . . . . . . . . . . . . . . . . . . . . . . . . . . 40

अंतरराष्ट्रीय संबंध/रक्षा . . . . . . . . . . . . . . . . . . . . . . . . . . . . . . . . . . . . . . . . . . . . . . . . . . . . . . . . . . . . . . . . . . . . . . . . . . . . 42

अंतर्राष्ट्रीय संगठन . . . . . . . . . . . . . . . . . . . . . . . . . . . . . . . . . . . . . . . . . . . . . . . . . . . . . . . . . . . . . . . . . . . . . . . . . . . . . . . . . . . . . . . . . . . . . . . . . . . . . . . . . . . . . . .43

• भारत ने UNSC की अध्यक्षता ग्रहण की . . . . . . . . . . . . . . . . . . . . . . . . . . . . . . . . . . . . . . . . . . . . . . . . . . . . . . . . . . . . . . . . . . . . . . . . . . . . . . . . . . . . . . .43

• महिलाओ ं की स्थिति पर आयोग (CSW) . . . . . . . . . . . . . . . . . . . . . . . . . . . . . . . . . . . . . . . . . . . . . . . . . . . . . . . . . . . . . . . . . . . . . . . . . . . . . . . . . . . . . .43

• कोसोवो ने यूरोपीय संघ में शामिल होने के लिए आवेदन किया है . . . . . . . . . . . . . . . . . . . . . . . . . . . . . . . . . . . . . . . . . . . . . . . . . . . . . . . . . . . . . 44

• ग्रुप ऑफ फ्रेंड्स . . . . . . . . . . . . . . . . . . . . . . . . . . . . . . . . . . . . . . . . . . . . . . . . . . . . . . . . . . . . . . . . . . . . . . . . . . . . . . . . . . . . . . . . . . . . . . . . . . . . . . . . . . . . . 44

• चीन-हिं द महासागर क्षेत्र फोरम . . . . . . . . . . . . . . . . . . . . . . . . . . . . . . . . . . . . . . . . . . . . . . . . . . . . . . . . . . . . . . . . . . . . . . . . . . . . . . . . . . . . . . . . . . . . . . 45

अंतर्राष्ट्रीय शिखर सम्मेलन,कॉन्फ़्रेन्स, कार्यक्रम . . . . . . . . . . . . . . . . . . . . . . . . . . . . . . . . . . . . . . . . . . . . . . . . . . . . . . . . . . . . . . . . . . . . . . . . . . . . . . . . . 45

• G20 वित्त ट्रैक . . . . . . . . . . . . . . . . . . . . . . . . . . . . . . . . . . . . . . . . . . . . . . . . . . . . . . . . . . . . . . . . . . . . . . . . . . . . . . . . . . . . . . . . . . . . . . . . . . . . . . . . . . . . . . 45

• चीन-जीसीसी शिखर सम्मेलन . . . . . . . . . . . . . . . . . . . . . . . . . . . . . . . . . . . . . . . . . . . . . . . . . . . . . . . . . . . . . . . . . . . . . . . . . . . . . . . . . . . . . . . . . . . . . . 45

• वासेनार व्यवस्था . . . . . . . . . . . . . . . . . . . . . . . . . . . . . . . . . . . . . . . . . . . . . . . . . . . . . . . . . . . . . . . . . . . . . . . . . . . . . . . . . . . . . . . . . . . . . . . . . . . . . . . . . . . 46

• राष्ट्रीय सुरक्षा सलाहकार (एनएसए) की बैठक. . . . . . . . . . . . . . . . . . . . . . . . . . . . . . . . . . . . . . . . . . . . . . . . . . . . . . . . . . . . . . . . . . . . . . . . . . . . . . . 46

• श्वेत पत्र क्रांति. . . . . . . . . . . . . . . . . . . . . . . . . . . . . . . . . . . . . . . . . . . . . . . . . . . . . . . . . . . . . . . . . . . . . . . . . . . . . . . . . . . . . . . . . . . . . . . . . . . . . . . . . . . . . . . .47

• ईगल अधिनियम . . . . . . . . . . . . . . . . . . . . . . . . . . . . . . . . . . . . . . . . . . . . . . . . . . . . . . . . . . . . . . . . . . . . . . . . . . . . . . . . . . . . . . . . . . . . . . . . . . . . . . . . . . . .47

• रूसी तेल पर मूल्य सीमा . . . . . . . . . . . . . . . . . . . . . . . . . . . . . . . . . . . . . . . . . . . . . . . . . . . . . . . . . . . . . . . . . . . . . . . . . . . . . . . . . . . . . . . . . . . . . . . . . . . . .47

• गश्त-ए-इरशाद. . . . . . . . . . . . . . . . . . . . . . . . . . . . . . . . . . . . . . . . . . . . . . . . . . . . . . . . . . . . . . . . . . . . . . . . . . . . . . . . . . . . . . . . . . . . . . . . . . . . . . . . . . . . . 49

रक्षा . . . . . . . . . . . . . . . . . . . . . . . . . . . . . . . . . . . . . . . . . . . . . . . . . . . . . . . . . . . . . . . . . . . . . . . . . . . . . . . . . . . . . . . . . . . . . . . . . . . . . . . . . . . . . . . . . . . . . . . . . . . . . 49

• भारत -चीन क्षेत्रीय विवाद . . . . . . . . . . . . . . . . . . . . . . . . . . . . . . . . . . . . . . . . . . . . . . . . . . . . . . . . . . . . . . . . . . . . . . . . . . . . . . . . . . . . . . . . . . . . . . . . . . . 49

Download Monthly Articulate Magazine प्रबोधन | 3


• आईएनएस मोरमुगाओ . . . . . . . . . . . . . . . . . . . . . . . . . . . . . . . . . . . . . . . . . . . . . . . . . . . . . . . . . . . . . . . . . . . . . . . . . . . . . . . . . . . . . . . . . . . . . . . . . . . . . 50

• अरनाला . . . . . . . . . . . . . . . . . . . . . . . . . . . . . . . . . . . . . . . . . . . . . . . . . . . . . . . . . . . . . . . . . . . . . . . . . . . . . . . . . . . . . . . . . . . . . . . . . . . . . . . . . . . . . . . . . . . 50

• अग्नि 5 मिसाइल . . . . . . . . . . . . . . . . . . . . . . . . . . . . . . . . . . . . . . . . . . . . . . . . . . . . . . . . . . . . . . . . . . . . . . . . . . . . . . . . . . . . . . . . . . . . . . . . . . . . . . . . . . . . 51

• सी ईगल/ओर्ल न 10 UAV . . . . . . . . . . . . . . . . . . . . . . . . . . . . . . . . . . . . . . . . . . . . . . . . . . . . . . . . . . . . . . . . . . . . . . . . . . . . . . . . . . . . . . . . . . . . . . . . . . . . . 51

• वैश्विक लड़ाकू हवाई कार्यक्रम (जीसीएपी) . . . . . . . . . . . . . . . . . . . . . . . . . . . . . . . . . . . . . . . . . . . . . . . . . . . . . . . . . . . . . . . . . . . . . . . . . . . . . . . . . . . . 51

• विजय दिवस. . . . . . . . . . . . . . . . . . . . . . . . . . . . . . . . . . . . . . . . . . . . . . . . . . . . . . . . . . . . . . . . . . . . . . . . . . . . . . . . . . . . . . . . . . . . . . . . . . . . . . . . . . . . . . . . . 51

• रक्षा अभ्यास . . . . . . . . . . . . . . . . . . . . . . . . . . . . . . . . . . . . . . . . . . . . . . . . . . . . . . . . . . . . . . . . . . . . . . . . . . . . . . . . . . . . . . . . . . . . . . . . . . . . . . . . . . . . . . . . .52

» सूर्या-किरण . . . . . . . . . . . . . . . . . . . . . . . . . . . . . . . . . . . . . . . . . . . . . . . . . . . . . . . . . . . . . . . . . . . . . . . . . . . . . . . . . . . . . . . . . . . . . . . . . . . . . . . . . . . . . . . . .52

» युद्ध-अभ्यास . . . . . . . . . . . . . . . . . . . . . . . . . . . . . . . . . . . . . . . . . . . . . . . . . . . . . . . . . . . . . . . . . . . . . . . . . . . . . . . . . . . . . . . . . . . . . . . . . . . . . . . . . . . . . . . .52

» क़ाज़िं द-22 . . . . . . . . . . . . . . . . . . . . . . . . . . . . . . . . . . . . . . . . . . . . . . . . . . . . . . . . . . . . . . . . . . . . . . . . . . . . . . . . . . . . . . . . . . . . . . . . . . . . . . . . . . . . . . . . . .52

» इं ड-इं डो समन्वय ग़श्त . . . . . . . . . . . . . . . . . . . . . . . . . . . . . . . . . . . . . . . . . . . . . . . . . . . . . . . . . . . . . . . . . . . . . . . . . . . . . . . . . . . . . . . . . . . . . . . . . . . . . . .52

भूगोल . . . . . . . . . . . . . . . . . . . . . . . . . . . . . . . . . . . . . . . . . . . . . . . . . . . . . . . . . . . . . . . . . . . . . . . . . . . . . . . . . . . . . . . . . . . . 53

तटीय लाल रेत के टीले . . . . . . . . . . . . . . . . . . . . . . . . . . . . . . . . . . . . . . . . . . . . . . . . . . . . . . . . . . . . . . . . . . . . . . . . . . . . . . . . . . . . . . . . . . . . . . . . . . . . . . . . . . 54

विश्व मृदा दिवस (डब्ल्यूएसडी) 2022 . . . . . . . . . . . . . . . . . . . . . . . . . . . . . . . . . . . . . . . . . . . . . . . . . . . . . . . . . . . . . . . . . . . . . . . . . . . . . . . . . . . . . . . . . . . . . 54

बाजरा 2023 के अंतर्राष्ट्रीय वर्ष के उद्घाटन समारोह का रोम में आयोजन . . . . . . . . . . . . . . . . . . . . . . . . . . . . . . . . . . . . . . . . . . . . . . . . . . . . . . . . . . 55

ग्रीनलैं ड के उत्तरी किनारे पर पर्माफ्रॉस्ट . . . . . . . . . . . . . . . . . . . . . . . . . . . . . . . . . . . . . . . . . . . . . . . . . . . . . . . . . . . . . . . . . . . . . . . . . . . . . . . . . . . . . . . . . . 55

आइल ऑफ़ मैन द्वीप . . . . . . . . . . . . . . . . . . . . . . . . . . . . . . . . . . . . . . . . . . . . . . . . . . . . . . . . . . . . . . . . . . . . . . . . . . . . . . . . . . . . . . . . . . . . . . . . . . . . . . . . . . . . 56

पर्यावरण . . . . . . . . . . . . . . . . . . . . . . . . . . . . . . . . . . . . . . . . . . . . . . . . . . . . . . . . . . . . . . . . . . . . . . . . . . . . . . . . . . . . . . . . . . 58

पर्यावरण संरक्षण के लिए अंतर्राष्ट्रीय संगठन और समझौते . . . . . . . . . . . . . . . . . . . . . . . . . . . . . . . . . . . . . . . . . . . . . . . . . . . . . . . . . . . . . . . . . . . . . . . 59

• जैविक विविधता पर सीओपी 15 सम्मेलन . . . . . . . . . . . . . . . . . . . . . . . . . . . . . . . . . . . . . . . . . . . . . . . . . . . . . . . . . . . . . . . . . . . . . . . . . . . . . . . . . . . . 59

• 30X30 गोल. . . . . . . . . . . . . . . . . . . . . . . . . . . . . . . . . . . . . . . . . . . . . . . . . . . . . . . . . . . . . . . . . . . . . . . . . . . . . . . . . . . . . . . . . . . . . . . . . . . . . . . . . . . . . . . . 60

• आइची जैव विविधता लक्ष्य . . . . . . . . . . . . . . . . . . . . . . . . . . . . . . . . . . . . . . . . . . . . . . . . . . . . . . . . . . . . . . . . . . . . . . . . . . . . . . . . . . . . . . . . . . . . . . . . . . 60

• प्रकृति के लिए गठबंधन . . . . . . . . . . . . . . . . . . . . . . . . . . . . . . . . . . . . . . . . . . . . . . . . . . . . . . . . . . . . . . . . . . . . . . . . . . . . . . . . . . . . . . . . . . . . . . . . . . . . . 61

• पर्यावरण शिक्षा, जागरूकता और प्रशिक्षण (EEAT) . . . . . . . . . . . . . . . . . . . . . . . . . . . . . . . . . . . . . . . . . . . . . . . . . . . . . . . . . . . . . . . . . . . . . . . . . . . . .62

जीव-जंतु . . . . . . . . . . . . . . . . . . . . . . . . . . . . . . . . . . . . . . . . . . . . . . . . . . . . . . . . . . . . . . . . . . . . . . . . . . . . . . . . . . . . . . . . . . . . . . . . . . . . . . . . . . . . . . . . . . . . . . . .62

• ग्रेट इं डियन बस्टर्ड . . . . . . . . . . . . . . . . . . . . . . . . . . . . . . . . . . . . . . . . . . . . . . . . . . . . . . . . . . . . . . . . . . . . . . . . . . . . . . . . . . . . . . . . . . . . . . . . . . . . . . . . . . . 62

• नाटोवेनेटर पॉलीडोन्टस . . . . . . . . . . . . . . . . . . . . . . . . . . . . . . . . . . . . . . . . . . . . . . . . . . . . . . . . . . . . . . . . . . . . . . . . . . . . . . . . . . . . . . . . . . . . . . . . . . . . .62

• लिसु व्रेन बैबलर . . . . . . . . . . . . . . . . . . . . . . . . . . . . . . . . . . . . . . . . . . . . . . . . . . . . . . . . . . . . . . . . . . . . . . . . . . . . . . . . . . . . . . . . . . . . . . . . . . . . . . . . . . . . .63

• रेड-क्राउन्ड रूफ्ड और लीथ्स सॉफ्टशेल कछुए . . . . . . . . . . . . . . . . . . . . . . . . . . . . . . . . . . . . . . . . . . . . . . . . . . . . . . . . . . . . . . . . . . . . . . . . . . . . . . . .63

प्रदूषण . . . . . . . . . . . . . . . . . . . . . . . . . . . . . . . . . . . . . . . . . . . . . . . . . . . . . . . . . . . . . . . . . . . . . . . . . . . . . . . . . . . . . . . . . . . . . . . . . . . . . . . . . . . . . . . . . . . . . . . . . . 64

• फ्लू गैस डिसल्फराइजेशन . . . . . . . . . . . . . . . . . . . . . . . . . . . . . . . . . . . . . . . . . . . . . . . . . . . . . . . . . . . . . . . . . . . . . . . . . . . . . . . . . . . . . . . . . . . . . . . . . . 64

• कार्बन संग्रहण, उपयोग और भंडारण (CCUS) नीतिगत ढांचा . . . . . . . . . . . . . . . . . . . . . . . . . . . . . . . . . . . . . . . . . . . . . . . . . . . . . . . . . . . . . . . . . . 64

• ग्रीन स्टील . . . . . . . . . . . . . . . . . . . . . . . . . . . . . . . . . . . . . . . . . . . . . . . . . . . . . . . . . . . . . . . . . . . . . . . . . . . . . . . . . . . . . . . . . . . . . . . . . . . . . . . . . . . . . . . . . 65

• नमामि गंगे कार्यक्रम . . . . . . . . . . . . . . . . . . . . . . . . . . . . . . . . . . . . . . . . . . . . . . . . . . . . . . . . . . . . . . . . . . . . . . . . . . . . . . . . . . . . . . . . . . . . . . . . . . . . . . . 65

4 I प्रबोधन Download Monthly Articulate Magazine


विज्ञान . . . . . . . . . . . . . . . . . . . . . . . . . . . . . . . . . . . . . . . . . . . . . . . . . . . . . . . . . . . . . . . . . . . . . . . . . . . . . . . . . . . . . . . . . . . . 68

तकनीक . . . . . . . . . . . . . . . . . . . . . . . . . . . . . . . . . . . . . . . . . . . . . . . . . . . . . . . . . . . . . . . . . . . . . . . . . . . . . . . . . . . . . . . . . . . . . . . . . . . . . . . . . . . . . . . . . . . . . . . . 69

• डिजीयात्रा पहल -फ़े स रिकॉग्निशन टे क्नोलॉजी . . . . . . . . . . . . . . . . . . . . . . . . . . . . . . . . . . . . . . . . . . . . . . . . . . . . . . . . . . . . . . . . . . . . . . . . . . . . . 69

• पहला ड्रोन स्टेशन . . . . . . . . . . . . . . . . . . . . . . . . . . . . . . . . . . . . . . . . . . . . . . . . . . . . . . . . . . . . . . . . . . . . . . . . . . . . . . . . . . . . . . . . . . . . . . . . . . . . . . . . . . .70

IT . . . . . . . . . . . . . . . . . . . . . . . . . . . . . . . . . . . . . . . . . . . . . . . . . . . . . . . . . . . . . . . . . . . . . . . . . . . . . . . . . . . . . . . . . . . . . . . . . . . . . . . . . . . . . . . . . . . . . . . . . . . . . . . .70

• चैटजीपीटी चैटबॉट . . . . . . . . . . . . . . . . . . . . . . . . . . . . . . . . . . . . . . . . . . . . . . . . . . . . . . . . . . . . . . . . . . . . . . . . . . . . . . . . . . . . . . . . . . . . . . . . . . . . . . . . . . .70

• एं ड-टू -एं ड एन्क्रिप्शन . . . . . . . . . . . . . . . . . . . . . . . . . . . . . . . . . . . . . . . . . . . . . . . . . . . . . . . . . . . . . . . . . . . . . . . . . . . . . . . . . . . . . . . . . . . . . . . . . . . . . . . .70

• सेमी-ऑटोमेटेड ऑफसाइड टे क्नोलॉजी (एसपीएटी)-फीफा 2022 . . . . . . . . . . . . . . . . . . . . . . . . . . . . . . . . . . . . . . . . . . . . . . . . . . . . . . . . . . . . . .71

• कैप्टिव गैर-सार्वजनिक नेटवर्क (सीएनपीएन) . . . . . . . . . . . . . . . . . . . . . . . . . . . . . . . . . . . . . . . . . . . . . . . . . . . . . . . . . . . . . . . . . . . . . . . . . . . . . . . 72

• सामान्य नेटवर्क एक्सेस विनियमन . . . . . . . . . . . . . . . . . . . . . . . . . . . . . . . . . . . . . . . . . . . . . . . . . . . . . . . . . . . . . . . . . . . . . . . . . . . . . . . . . . . . . . . . . . 72

साइबर ख़तरे . . . . . . . . . . . . . . . . . . . . . . . . . . . . . . . . . . . . . . . . . . . . . . . . . . . . . . . . . . . . . . . . . . . . . . . . . . . . . . . . . . . . . . . . . . . . . . . . . . . . . . . . . . . . . . . . . . . . . 72

• रैंसमवेयर . . . . . . . . . . . . . . . . . . . . . . . . . . . . . . . . . . . . . . . . . . . . . . . . . . . . . . . . . . . . . . . . . . . . . . . . . . . . . . . . . . . . . . . . . . . . . . . . . . . . . . . . . . . . . . . . . . . 72

• ब्लू बगिंग . . . . . . . . . . . . . . . . . . . . . . . . . . . . . . . . . . . . . . . . . . . . . . . . . . . . . . . . . . . . . . . . . . . . . . . . . . . . . . . . . . . . . . . . . . . . . . . . . . . . . . . . . . . . . . . . . . . . 73

• डीपफेक तकनीक . . . . . . . . . . . . . . . . . . . . . . . . . . . . . . . . . . . . . . . . . . . . . . . . . . . . . . . . . . . . . . . . . . . . . . . . . . . . . . . . . . . . . . . . . . . . . . . . . . . . . . . . . . . 73

• डॉक्सिं ग . . . . . . . . . . . . . . . . . . . . . . . . . . . . . . . . . . . . . . . . . . . . . . . . . . . . . . . . . . . . . . . . . . . . . . . . . . . . . . . . . . . . . . . . . . . . . . . . . . . . . . . . . . . . . . . . . . . . .74

अंतरिक्ष . . . . . . . . . . . . . . . . . . . . . . . . . . . . . . . . . . . . . . . . . . . . . . . . . . . . . . . . . . . . . . . . . . . . . . . . . . . . . . . . . . . . . . . . . . . . . . . . . . . . . . . . . . . . . . . . . . . . . . . . . .74

• जेमिनिड्स उल्का बौछार . . . . . . . . . . . . . . . . . . . . . . . . . . . . . . . . . . . . . . . . . . . . . . . . . . . . . . . . . . . . . . . . . . . . . . . . . . . . . . . . . . . . . . . . . . . . . . . . . . . . .74

• भारतीय तारामंडल के साथ नेविगेशन (NavIC) . . . . . . . . . . . . . . . . . . . . . . . . . . . . . . . . . . . . . . . . . . . . . . . . . . . . . . . . . . . . . . . . . . . . . . . . . . . . . . .75

• भारत का पहला निजी वाहन लॉन्चपैड . . . . . . . . . . . . . . . . . . . . . . . . . . . . . . . . . . . . . . . . . . . . . . . . . . . . . . . . . . . . . . . . . . . . . . . . . . . . . . . . . . . . . . . .76

• सरस 3: रेडियो टे लीस्कोप . . . . . . . . . . . . . . . . . . . . . . . . . . . . . . . . . . . . . . . . . . . . . . . . . . . . . . . . . . . . . . . . . . . . . . . . . . . . . . . . . . . . . . . . . . . . . . . . . . . .76

• वालाबाई . . . . . . . . . . . . . . . . . . . . . . . . . . . . . . . . . . . . . . . . . . . . . . . . . . . . . . . . . . . . . . . . . . . . . . . . . . . . . . . . . . . . . . . . . . . . . . . . . . . . . . . . . . . . . . . . . . . . 77

• सतह जल और महासागरीय स्थलाकृति (SWOT) . . . . . . . . . . . . . . . . . . . . . . . . . . . . . . . . . . . . . . . . . . . . . . . . . . . . . . . . . . . . . . . . . . . . . . . . . . . . . 77

स्वास्थ्य . . . . . . . . . . . . . . . . . . . . . . . . . . . . . . . . . . . . . . . . . . . . . . . . . . . . . . . . . . . . . . . . . . . . . . . . . . . . . . . . . . . . . . . . . . . . . . . . . . . . . . . . . . . . . . . . . . . . . . . . . . 77

• ज़ोंबी वायरस . . . . . . . . . . . . . . . . . . . . . . . . . . . . . . . . . . . . . . . . . . . . . . . . . . . . . . . . . . . . . . . . . . . . . . . . . . . . . . . . . . . . . . . . . . . . . . . . . . . . . . . . . . . . . . . . 77

• डिजीज X . . . . . . . . . . . . . . . . . . . . . . . . . . . . . . . . . . . . . . . . . . . . . . . . . . . . . . . . . . . . . . . . . . . . . . . . . . . . . . . . . . . . . . . . . . . . . . . . . . . . . . . . . . . . . . . . . . . .78

• खसरा . . . . . . . . . . . . . . . . . . . . . . . . . . . . . . . . . . . . . . . . . . . . . . . . . . . . . . . . . . . . . . . . . . . . . . . . . . . . . . . . . . . . . . . . . . . . . . . . . . . . . . . . . . . . . . . . . . . . . . .78

• मंकीपॉक्स का नाम बदलकर MPOX कर दिया गया . . . . . . . . . . . . . . . . . . . . . . . . . . . . . . . . . . . . . . . . . . . . . . . . . . . . . . . . . . . . . . . . . . . . . . . . . .79

• सर्वावैक-सरवाइकल कैंसर टीका . . . . . . . . . . . . . . . . . . . . . . . . . . . . . . . . . . . . . . . . . . . . . . . . . . . . . . . . . . . . . . . . . . . . . . . . . . . . . . . . . . . . . . . . . . . 80

• BF.7: ओमिक्रॉन का सब-वैरिएं ट . . . . . . . . . . . . . . . . . . . . . . . . . . . . . . . . . . . . . . . . . . . . . . . . . . . . . . . . . . . . . . . . . . . . . . . . . . . . . . . . . . . . . . . . . . . . . 81

• राष्ट्रीय दुर्ल भ रोग नीति 202181 . . . . . . . . . . . . . . . . . . . . . . . . . . . . . . . . . . . . . . . . . . . . . . . . . . . . . . . . . . . . . . . . . . . . . . . . . . . . . . . . . . . . . . . . . . . . . . 81

• अन्यूरिज़्म . . . . . . . . . . . . . . . . . . . . . . . . . . . . . . . . . . . . . . . . . . . . . . . . . . . . . . . . . . . . . . . . . . . . . . . . . . . . . . . . . . . . . . . . . . . . . . . . . . . . . . . . . . . . . . . . . . .82

• बिहार में जहरीली शराब त्रासदी . . . . . . . . . . . . . . . . . . . . . . . . . . . . . . . . . . . . . . . . . . . . . . . . . . . . . . . . . . . . . . . . . . . . . . . . . . . . . . . . . . . . . . . . . . . . . . .82

• मातृ मृत्यु दर . . . . . . . . . . . . . . . . . . . . . . . . . . . . . . . . . . . . . . . . . . . . . . . . . . . . . . . . . . . . . . . . . . . . . . . . . . . . . . . . . . . . . . . . . . . . . . . . . . . . . . . . . . . . . . . 83

• बेस/क्षार संपादन तकनीक . . . . . . . . . . . . . . . . . . . . . . . . . . . . . . . . . . . . . . . . . . . . . . . . . . . . . . . . . . . . . . . . . . . . . . . . . . . . . . . . . . . . . . . . . . . . . . . . . . 83

Download Monthly Articulate Magazine प्रबोधन | 5


इतिहास/संस्कृति . . . . . . . . . . . . . . . . . . . . . . . . . . . . . . . . . . . . . . . . . . . . . . . . . . . . . . . . . . . . . . . . . . . . . . . . . . . . . . . . . . 85

अमूर्त सांस्कृतिक विरासत:बगेट ( Baguette) . . . . . . . . . . . . . . . . . . . . . . . . . . . . . . . . . . . . . . . . . . . . . . . . . . . . . . . . . . . . . . . . . . . . . . . . . . . . . . . . . . . 86

यूनेस्को की विश्व धरोहर स्थलों की संभावित सूची में तीन और स्थल जोड़े गए . . . . . . . . . . . . . . . . . . . . . . . . . . . . . . . . . . . . . . . . . . . . . . . . . . . . . . 86

अष्टाध्यायी . . . . . . . . . . . . . . . . . . . . . . . . . . . . . . . . . . . . . . . . . . . . . . . . . . . . . . . . . . . . . . . . . . . . . . . . . . . . . . . . . . . . . . . . . . . . . . . . . . . . . . . . . . . . . . . . . . . . . .87

संग्रहालय अनुदान योजना (एमजीएस) और विज्ञान की संस्कृति को बढ़ावा देने के लिए योजना (एसपीओसी) . . . . . . . . . . . . . . . . . . . . . . . .87

कोच्चि-मुजिरिस बिएनेल . . . . . . . . . . . . . . . . . . . . . . . . . . . . . . . . . . . . . . . . . . . . . . . . . . . . . . . . . . . . . . . . . . . . . . . . . . . . . . . . . . . . . . . . . . . . . . . . . . . . . . . . .87

अल रिहला. . . . . . . . . . . . . . . . . . . . . . . . . . . . . . . . . . . . . . . . . . . . . . . . . . . . . . . . . . . . . . . . . . . . . . . . . . . . . . . . . . . . . . . . . . . . . . . . . . . . . . . . . . . . . . . . . . . . . . 88

महत्त्वपूर्ण दिन . . . . . . . . . . . . . . . . . . . . . . . . . . . . . . . . . . . . . . . . . . . . . . . . . . . . . . . . . . . . . . . . . . . . . . . . . . . . . . . . . . . . . . . . . . . . . . . . . . . . . . . . . . . . . . . . . . 88

खबरों में व्यक्ति/सामान्य ज्ञान . . . . . . . . . . . . . . . . . . . . . . . . . . . . . . . . . . . . . . . . . . . . . . . . . . . . . . . . . . . . . . . . . . . . . . 92

जे रॉबर्ट ऑपेन्हायमर- फादर ओफ़ अटॉमिक बॉम . . . . . . . . . . . . . . . . . . . . . . . . . . . . . . . . . . . . . . . . . . . . . . . . . . . . . . . . . . . . . . . . . . . . . . . . . . . . . . . . .93

पी टी ऊषा . . . . . . . . . . . . . . . . . . . . . . . . . . . . . . . . . . . . . . . . . . . . . . . . . . . . . . . . . . . . . . . . . . . . . . . . . . . . . . . . . . . . . . . . . . . . . . . . . . . . . . . . . . . . . . . . . . . . . . .93

फ़ीफ़ा वर्ल्ड कप . . . . . . . . . . . . . . . . . . . . . . . . . . . . . . . . . . . . . . . . . . . . . . . . . . . . . . . . . . . . . . . . . . . . . . . . . . . . . . . . . . . . . . . . . . . . . . . . . . . . . . . . . . . . . . . . . .93

जुल्स रिमेट ट्रॉफी . . . . . . . . . . . . . . . . . . . . . . . . . . . . . . . . . . . . . . . . . . . . . . . . . . . . . . . . . . . . . . . . . . . . . . . . . . . . . . . . . . . . . . . . . . . . . . . . . . . . . . . . . . . . . . .93

किताब:ऐज गुड ऐज माइ वर्ड . . . . . . . . . . . . . . . . . . . . . . . . . . . . . . . . . . . . . . . . . . . . . . . . . . . . . . . . . . . . . . . . . . . . . . . . . . . . . . . . . . . . . . . . . . . . . . . . . . . . . 94

किताब:सिनेमा स्पेक्युलेशन . . . . . . . . . . . . . . . . . . . . . . . . . . . . . . . . . . . . . . . . . . . . . . . . . . . . . . . . . . . . . . . . . . . . . . . . . . . . . . . . . . . . . . . . . . . . . . . . . . . . . 94

95 वाँ अकैडमी अवार्डस शोर्टलिस्ट . . . . . . . . . . . . . . . . . . . . . . . . . . . . . . . . . . . . . . . . . . . . . . . . . . . . . . . . . . . . . . . . . . . . . . . . . . . . . . . . . . . . . . . . . . . . . . 94

ऑक्सफ़ोर्ड डिक्शनरी - वर्ड ओफ़ द ईयर . . . . . . . . . . . . . . . . . . . . . . . . . . . . . . . . . . . . . . . . . . . . . . . . . . . . . . . . . . . . . . . . . . . . . . . . . . . . . . . . . . . . . . . . . 94

मेरियम वेब्स्टर- वर्ड ऑफ़ द ईयर . . . . . . . . . . . . . . . . . . . . . . . . . . . . . . . . . . . . . . . . . . . . . . . . . . . . . . . . . . . . . . . . . . . . . . . . . . . . . . . . . . . . . . . . . . . . . . . . 94

बहुविकल्पीय प्रश्न . . . . . . . . . . . . . . . . . . . . . . . . . . . . . . . . . . . . . . . . . . . . . . . . . . . . . . . . . . . . . . . . . . . . . . . . . . . . . . . . 95

Enroll Now

6 I प्रबोधन Download Monthly Articulate Magazine


Download Monthly Articulate Magazine प्रबोधन | 7
अधिकार सम्बन्धी मुद्दे
ओबीसी उप-वर्गीकरण

खबरों में क्यों?

अन्य पिछड़ा वर्ग (ओबीसी) के उप-वर्गीकरण के लिए आयोग अब • ओबीसी के भीतर उप-श्रेणियां बनाने से सभी ओबीसी समुदायों
लगभग 3,000 जाति समूहों को और वर्गीकृत करने के फॉर्मूले के के बीच प्रतिनिधित्व का “समान वितरण” सुनिश्चित होगा।
साथ आने के अपने कार्य को पूरा करने के अंतिम चरण में है। • इसकी जांच के लिए रोहिणी आयोग का गठन 2 अक्टूबर, 2017
को किया गया था।

प्रमुख बिं दु: भारत में आरक्षण:


• अक्टूबर 2017: भारतीय संविधान के अनुच्छेद 340 के तहत • ओबीसी आरक्षण को पहली बार 1980 की मंडल आयोग की
न्यायमूर्ति जी रोहिणी आयोग, दिल्ली उच्च न्यायालय के पूर्व रिपोर्ट में परिभाषित किया गया था।
मुख्य न्यायाधीश की स्थापना की।
• रिपोर्ट में अनुमान लगाया गया है कि अनुसूचित जाति/जनजाति
» आरक्षण के उद्देश्य से ओबीसी के बड़े समूह के भीतर उप-श्रेणियां
(एससी/एसटी) को छोड़कर ओबीसी भारत की आबादी का
बनाने का विचार है।
लगभग 52 प्रतिशत है।
• 2022: आयोग जल्द ही रिपोर्ट जारी करेगा।
• इसलिए, ओबीसी को शामिल करने के लिए, रिपोर्ट ने सरकारी
उप-वर्गीकरण की आवश्यकता: सेवाओ ं और केंद्रीय / राज्य शैक्षणिक संस्थानों में इन समुदायों
के लिए 27 प्रतिशत आरक्षण की सिफारिश की।
• ओबीसी को केंद्र सरकार के तहत नौकरियों और शिक्षा में 27%
आरक्षण दिया जाता है। • वर्तमान में, एससी/एसटी समुदायों के पास 22.5 प्रतिशत
आरक्षण है, ओबीसी के पास 27 प्रतिशत और आर्थि क रूप
• हालाँकि, बहस इस धारणा से उत्पन्न होती है कि ओबीसी की
से कमजोर वर्ग (ईडब्ल्यूएस) के सदस्यों के पास सरकारी
केंद्रीय सूची में शामिल 2,600 में से केवल कुछ संपन्न समुदायों
नौकरियों और शैक्षणिक संस्थानों में 10 प्रतिशत आरक्षण है।
ने 27% आरक्षण का एक बड़ा हिस्सा हासिल किया है।

पॉलिसी विवरण

ओबीसी के लिए राज्यवार आरक्षण: • 2021 में, संसद ने 127वां संवैधानिक संशोधन पारित किया, जिसने राज्यों और केंद्र
शासित प्रदेशों को सामाजिक और शैक्षिक रूप से पिछड़े वर्गों (SEBC) की अपनी सूची
तैयार करने की अनुमति दी।

मेडिकल प्रवेश में ओबीसी कोटा: • जुलाई 2021 में, केंद्र ने शैक्षणिक वर्ष 2021-22 से स्नातक और स्नातकोत्तर चिकित्सा
और दंत चिकित्सा पाठ्यक्रमों के लिए ओबीसी के लिए 27 प्रतिशत और ईडब्ल्यूएस
श्रेणी के लिए 10 प्रतिशत कोटा की घोषणा की।

चुनाव में ओबीसी कोटा: • राजनीतिक प्रतिनिधित्व सुनिश्चित करने के लिए, लोकसभा में 543 सीटों में से 84
सीटें अनुसूचित जाति के लिए, 47 सीटें अनुसूचित जनजाति के लिए और 2 एं ग्लो-
इं डियन सदस्यों (राष्ट्रपति द्वारा मनोनीत) के लिए आरक्षित हैं।

समाचार स्रोत: द हिं दू

8 I प्रबोधन Download Monthly Articulate Magazine


समलैं गिक विवाह को वैध बनाना अंतर्राष्ट्रीय प्राथमिकता:

खबरों में क्यों? • दुनिया भर के कुल 32 देशों ने समान-लिंग विवाहों को वैध कर


दिया है।
हाल ही में, सुप्रीम कोर्ट की एक खंडपीठ ने विशेष विवाह अधिनियम,
(SMA) 1954 के तहत समान-लिंग विवाह को अनुमति देने के लिए • 2001 में समलैं गिक विवाह को वैध बनाने वाला नीदरलैं ड
केंद्र को नोटिस जारी किया। पहला देश था।

प्रमुख बिं दु: समाचार स्रोत: द हिं दू


• विशेष विवाह अधिनियम, (एसएमए) 1954 के तहत समलैं गिक
जोड़ों द्वारा समलैं गिक विवाह की अनुमति देने के लिए दो PYQ
याचिकाएं दायर की गई थीं। Q. निजता के अधिकार पर सर्वोच्च न्यायालय के नवीनतम
• याचिकाकर्ता ने कहा कि जिस तरह यह समान-लिंग जोड़ों और निर्णय के आलोक में मौलिक अधिकारों के दायरे की जांच
विपरीत-लिंग जोड़ों के बीच भेदभाव करता है, समान सेक्स करें। (2017)
विवाह संविधान सम्मत नहीं है(अल्ट्रा वायर्स)।

सुप्रीम कोर्ट के अहम फैसले :

नवतेज जौहर का 2018 का फैसला: रिकॉर्ड में तीसरे लिं ग का विकल्प


• सुप्रीम कोर्ट के पांच जजों की बेंच ने समलैं गिकता को अपराध
खबरों में क्यों?
की श्रेणी से बाहर कर दिया था।
स्विस सरकार ने आधिकारिक रिकॉर्ड के लिए तीसरे लिंग विकल्प
• इसने माना कि भारतीय दंड संहिता की धारा 377 समान
या गैर-लिंग विकल्प पेश करने के विचार को खारिज कर दिया।
लिंग के वयस्कों के बीच निजी सहमति से यौन आचरण का
अपराधीकरण असंवैधानिक था। फैसले के बारे में:

• यौन अभिविन्यास के आधार पर भेदभाव भाषण और अभिव्यक्ति • स्विस समाज में द्विआधारी लिंग विचार अभी भी गहराई से जुड़ा
की स्वतंत्रता का उल्लं घन है। हुआ है।

• लोग केवल पुरुष या महिला के रूप में नागरिक रजिस्ट्री में


नालसा बनाम भारत संघ का फैसला (2014):
प्रवेश कर सकेंगे।
• न्यायालय ने कहा था कि गैर-बाइनरी व्यक्तियों को संविधान
• जर्मनी, ऑस्ट्रिया और बेल्जियम जैसे पड़ोसी देशों ने
के तहत संरक्षित किया गया था और मौलिक अधिकार जैसे
आधिकारिक रिकॉर्ड के लिए तीसरे लिंग या गैर-लिंग विकल्पों
समानता, गैर-भेदभाव, जीवन, स्वतंत्रता, और इसी तरह उन
को अपनाया है।
लोगों तक सीमित नहीं किया जा सकता जो जैविक रूप से पुरुष
या महिला थे। भारत में स्थिति:

सरकार का रुख: • भारत सरकार और इसके विभिन्न विभागों, एजेंसियों आदि ने


रूपों में तीसरे लिंग के विकल्प के लिए सरकारी परीक्षा फॉर्म,
• केंद्र ने कहा कि एक ही लिंग के दो व्यक्तियों के बीच विवाह
भारतीय पासपोर्ट और मतदाता पहचान पत्र पर प्रावधान किए
संस्था की स्वीकृति किसी भी असंहिताबद्ध व्यक्तिगत कानूनों
हैं।
या किसी भी संहिताबद्ध वैधानिक कानूनों में न तो मान्यता
प्राप्त है और न ही स्वीकृत है। • यह भारतीय समाज में एक प्रगतिशील प्रवृत्ति को दर्शाता है।

व्यक्तित्व अधिकार

Download Monthly Articulate Magazine प्रबोधन | 9


खबरों में क्यों?

दिल्ली उच्च न्यायालय ने एक अभिनेता के व्यक्तित्व अधिकारों के


उल्लं घन का हवाला देते हुए बॉलीवुड स्टार के नाम, छवियों और
आवाज के गैरकानूनी उपयोग को रोकने के लिए एक अंतरिम
आदेश पारित किया।

प्रमुख बिं दु:

के बारे में

व्यक्तित्व अधिकार
• व्यक्तित्व अधिकार एक व्यक्ति के निजता या संपत्ति के
अधिकार के तहत उसके व्यक्तित्व की रक्षा करने के अधिकार
को संदर्भित करता है।
» उदाहरण के लिए: अमिताभ बच्चन ने फर्जी कौन बनेगा
करोड़पति (केबीसी) लॉटरी घोटाले और अन्य धोखाधड़ी के
खिलाफ अपने प्रचार अधिकारों की सुरक्षा की मांग की, जहां
जनता को धोखा देने के लिए उनकी आवाज और फोटो का
छवि स्रोत: नवभारत टाइम्स
इस्तेमाल किया जा रहा था।
• ये प्रचार अधिकारों से भिन्न हैं।
» प्रचार अधिकार ट्रेड मार्क अधिनियम 1999 और कॉपीराइट
अधिनियम 1957 जैसी विधियों द्वारा शासित होते हैं।
• किसी सेलेब्रिटी की व्यक्तिगत विशेषताओ ं को संरक्षित किया
जाता है, जैसे नाम, उपनाम, मंच का नाम, चित्र, समानता, छवि,
और कोई पहचान योग्य व्यक्तिगत संपत्ति, जैसे विशिष्ट रेस
कार।

व्यक्तित्व अधिकार क्यों महत्वपूर्ण हैं?


• अपनी बिक्री को बढ़ावा देने के लिए विभिन्न कंपनियों द्वारा
विज्ञापनों में मशहूर हस्तियों के नाम, फोटो या आवाज के
दुरुपयोग को रोकने के लिए। विवाद के प्रमुख बिं दु:
• मेघालय का लं गपीह जिला: मेघालय का लं गपीह जिला, असम
व्यक्तित्व अधिकारों में शामिल हैं: के कामरूप जिले की सीमा से पश्चिम गारो हिल्स में, दो पड़ोसी
• प्रचार का अधिकार: इसमें किसी की छवि और समानता को राज्यों के बीच विवाद का एक प्रमुख बिं दु है।
बिना अनुमति के व्यावसायिक रूप से शोषण किए जाने से • मिकिर हिल्स: विवाद का एक अन्य बिं दु मिकिर हिल्स है, जिसे
रोकने का अधिकार शामिल है। असम अपना हिस्सा मानता है।
• निजता का अधिकार: इसका अर्थ है बिना अनुमति के किसी
के व्यक्तित्व को सार्वजनिक रूप से प्रदर्शित नहीं करने का हाल ही में उठाए गए कदम:
अधिकार। • दोनों राज्यों ने सीमा विवाद समाधान समितियों का गठन किया
है और सीमा विवादों को चरणबद्ध तरीके से हल करने के लिए
समाचार स्रोत: द हिं दू दो क्षेत्रीय पैनल स्थापित करने का निर्णय लिया है।

• दोनों राज्य सरकारों ने पहले चरण में समाधान के लिए 12


विवादित क्षेत्रों में से छह की पहचान की।
संघीय मुद्दे • कई बैठकों और विवादित क्षेत्रों में टीमों के दौरे के बाद, दोनों
असम-मेघालय सीमा विवाद पक्षों ने परस्पर सहमत पांच सिद्धांतों के आधार पर रिपोर्ट प्रस्तुत
की:
खबरों में क्यों? » ऐतिहासिक विचारों के आधार पर।
NHRC ने पाया कि दो राज्यों - असम और मेघालय के बीच सीमा » स्थानीय आबादी की जातीयता।
विवाद के कारण मुकरोह गांव में गोलीबारी की घटना हुई थी। असम
» सीमा के साथ निकटता।
और मेघालय दोनों इस मामले की जांच के लिए एक स्वतंत्र केंद्रीय
एजेंसी चाहते हैं। » लोगों की इच्छा और

» प्रशासनिक सुविधा।

10 I प्रबोधन Download Monthly Articulate Magazine


• इसमें शामिल छह बिं दु थे: ताराबारी, गिजांग, हाशिम, बाकलापारा, खानापारा-पिलिंगकाटा और राताचेर्रा।

निष्कर्ष:
• यह समझौता सहकारी संघवाद का उदाहरण है और राज्यों के बीच अन्य सीमा विवादों के समाधान के लिए एक रोडमैप प्रदान करेगा।

समझौता दल

एनएलएफटी समझौता (त्रिपुरा शांति समझौता) भारत सरकार, त्रिपुरा सरकार, एनएलएफटी

ब्रू-रियांग समझौता भारत सरकार, त्रिपुरा की राज्य सरकार, ब्रू-रियांग प्रतिनिधि,


मिजोरम की राज्य सरकार

बोडो समझौता भारत सरकार, असम सरकार, बोडो प्रतिनिधि

कार्बी-एं गलोंग समझौता असम के पांच विद्रोही समूह, केंद्र, असम की राज्य सरकार

समाचार स्त्रोत: द इं डियन एक्सप्रेस

PYQ

Q. अंतर-राज्यीय जल विवादों को हल करने के लिए संवैधानिक तंत्र समस्याओ ं को हल करने और हल करने में विफल रहे हैं। क्या
विफलता संरचनात्मक या प्रक्रिया की अपर्याप्तता या दोनों के कारण है? चर्चा क़रिए।(2013)

लद्दाख संघ राज्य क्षेत्र को छठी अनुसूची में शामिल करने की मांग

खबरों में क्यों?

हाल ही में, केंद्रीय गृह मंत्रालय ने संविधान की छठी अनुसूची के तहत लद्दाख को शामिल करने पर सीधा जवाब देने से इनकार कर दिया।

छठी अनुसूची के बारे में:


• भारतीय संविधान के अनुच्छेद 244 के अनुसार, छठी अनुसूची में असम, मेघालय, त्रिपुरा और मिजोरम में आदिवासी क्षेत्रों के प्रशासन
के प्रावधान शामिल हैं।

• छठी अनुसूची के तहत, निर्वाचित प्रतिनिधियों द्वारा प्रशासित स्वायत्त जिलों और परिषदों को जनजातीय लोगों के हितों की रक्षा के लिए
कानून बनाने के लिए अलग-अलग स्वायत्तता प्राप्त है।

5वीं अनुसूची 6 ठवी अनुसूची

यह छठी अनुसूची वाले राज्यों के अलावा अन्य राज्यों में अनुसूचित छठी अनुसूची: असम, मेघालय, त्रिपुरा और मिजोरम में आदिवासी
क्षेत्रों और अनुसूचित जनजातियों के प्रशासन और नियंत्रण के लिए क्षेत्रों के प्रशासन से संबंधित है।
प्रावधान करता है।

अनुसूचित क्षेत्रों में जो 5वीं अनुसूची के अंतर्गत आते हैं, यह राज्य जबकि छठी अनुसूची में उन क्षेत्रों को शामिल किया गया है जो
का राज्यपाल होता है जिसके पास जनजातीय आबादी की सुरक्षा स्वशासन पर निर्भर हैं।
के लिए विशेष शक्तियां और जिम्मेदारियां होती हैं।

Download Monthly Articulate Magazine प्रबोधन | 11


स्वायत्त जिला परिषदें (ADCs):
• एडीसी राज्य के भीतर कुछ विधायी, न्यायिक और प्रशासनिक स्वायत्तता का आनंद लेते हैं।

• उनके पास पांच साल की अवधि के साथ 30 सदस्य हैं।

• जनादेश: भूमि, वन, जल, कृषि, ग्राम सभा, स्वास्थ्य, स्वच्छता, ग्राम और नगर स्तर की पुलिसिंग, विरासत, विवाह और तलाक, सामाजिक
रीति-रिवाजों और खनन के संबंध में कानून, नियम और विनियम बनाना।

370 हटने के पहले 2019: 370 हटने के बाद

लद्दाख के विधायक जम्मू-कश्मीर विधानसभा में अपनी आवाज केंद्र शासित प्रदेश जम्मू-कश्मीर (विधायिका के साथ) और केंद्र
उठा सकते हैं शासित प्रदेश लद्दाख (विधायिका के बिना) का गठन किया गया।

आवाज़ नहीं उठा सकते। और लद्दाख क्षेत्र को पूरी तरह से नौकरशाहों


द्वारा प्रशासित किया जाता है। इसलिए लद्दाखी राजनीतिक नेता
6वीं अनुसूची को शामिल करना चाहते हैं।

छवि स्रोत: द इकोनॉमिक टाइम्स की, क्योंकि इसके मुख्य रूप से आदिवासी (97% से अधिक)
लद्दाख को छठी अनुसूची में शामिल करने की मांग के कारण: लोग हैं।
• जम्मू और कश्मीर में बदली हुई अधिवास नीति ने इस क्षेत्र में • लद्दाख को छठी अनुसूची में सूचीबद्ध किए जाने के बाद, यह क्षेत्र
अपनी भूमि, रोजगार, जनसांख्यिकी और सांस्कृतिक पहचान में शक्तियों के लोकतांत्रिक हस्तांतरण का मार्ग प्रशस्त करेगा।
को ले कर भय पैदा कर दिया है।
लद्दाख को छठी अनुसूची में शामिल करने के पीछे कठिनाइयाँ:
• इसके अलावा, अनुसूचित जनजाति के लिए राष्ट्रीय आयोग ने
लद्दाख को छठी अनुसूची के तहत शामिल करने की सिफारिश • संविधान की छठी अनुसूची के प्रावधान चार उत्तर-पूर्व राज्यों से

12 I प्रबोधन Download Monthly Articulate Magazine


संबंधित हैं: असम, मेघालय, मिजोरम और त्रिपुरा।
सरकार का विशेषाधिकार है।
• विशेष रूप से, पूर्वोत्तर के बाहर के किसी भी क्षेत्र को छठी
अनुसूची में शामिल नहीं किया गया है। समाचार स्रोत: द इं डियन एक्सप्रेस
» यहां तक कि मणिपुर में भी जहां कुछ जगहों पर मुख्य रूप से
जनजातीय आबादी है, उनकी स्वायत्त परिषदों को छठी अनुसूची
में शामिल नहीं किया गया है।
ग्रेटर टिपरालैं ड
• देश के बाकी हिस्सों में आदिवासी क्षेत्रों के लिए पांचवीं अनुसूची
है। खबरों में क्यों?
• हालाँकि, छठी अनुसूची में किसी अन्य क्षेत्र को शामिल करने • टिपरा मोथा पार्टी ने “ग्रेटर टिपरालैं ड” की मांग को उठाने के
के लिए संविधान में संशोधन करने के लिए एक विधेयक लाना लिए जंतर-मंतर पर दो दिवसीय धरने का नेतृत्व किया।

Download Monthly Articulate Magazine प्रबोधन | 13


पार्श्वभूमि: संवैधानिक मुद्दे/संसद/न्यायपालिका
• त्रिपुरा 13वीं शताब्दी के अंत से 15 अक्टू बर, 1949 को भारत अधिकारों का विभाजन
सरकार के साथ विलय के साधन पर हस्ताक्षर करने तक
माणिक्य राजवंश द्वारा शासित एक राज्य था। खबरों में क्यों?
• मांग राज्य के जनसांख्यिकीय में परिवर्तन के संबंध में स्वदेशी हाल के दिनों में विभिन्न घटनाओ ं के कारण “शक्तियों के पृथक्करण”
समुदायों की चिंता से उत्पन्न होती है, जिससे वे अल्पसंख्यक का सिद्धांत प्रकाश में आया है।
हो गए हैं।
क्या हैं घटनाएं ?

तात्कालिक कारण: • कॉले जियम सिस्टम को ले कर केंद्रीय कानून मंत्री और जजों


के बीच जुबानी जंग
• टीआईपीआरए मोथा के उदय के साथ राज्य की राजनीति में
मंथन और 2023 की शुरुआत में होने वाले विधानसभा चुनाव • दिल्ली में आप सरकार और एलजी के बीच खींचतान उदाहरण
विकास के दो प्रमुख कारण हैं। के लिए.- राजनीतिक विज्ञापनों पर।

• सिखों को उड़ानों में कृपाण ले जाने की अनुमति देने के केंद्र


घटना विवरण सरकार के फैसले के खिलाफ एक जनहित याचिका दायर की
गई थी। दिल्ली उच्च न्यायालय ने शक्तियों के पृथक्करण का
बंगालियों का विस्थापन 1947 और 1971 के बीच
हवाला देते हुए इसे खारिज कर दिया।
तत्कालीन पूर्वी पाकिस्तान से
बंगालियों को विस्थापित किया
गया था।
शक्तियों के पृथक्करण के बारे में:
त्रिपुरा में जनजातीय आबादी में त्रिपुरा में आदिवासियों की
• शक्तियों के पृथक्करण के सिद्धांत से पता चलता है कि शासन
कमी आबादी 1881 में 63.77% से
की शक्तियों का स्वतंत्र रूप से तीन अलग-अलग अंगों द्वारा
घटकर 2011 तक 31.80% हो
प्रयोग किया जाना चाहिए।
गई।

मांग:
• पार्टि यां पूर्वोत्तर राज्य के स्वदेशी समुदायों के लिए ‘ग्रेटर
टिप्रालैं ड’ के एक अलग राज्य की मांग कर रही हैं।

• वे चाहते हैं कि केंद्र संविधान के अनुच्छेद 2 और 3 के तहत एक


अलग राज्य बनाए।
» अनुच्छेद 2: यह संसद को ‘भारत के संघ में प्रवेश करने, या
ऐसे नियमों और शर्तों पर नए राज्यों की स्थापना करने का
अधिकार देता है, जैसा कि वह उचित समझता है।

» अनुच्छेद 3: इसने संसद को नए राज्यों के गठन और क्षेत्रों,


सीमाओ ं या मौजूदा राज्यों के नामों में परिवर्तन से संबंधित
कानून बनाने का अधिकार दिया।
• उनकी शक्तियों और जिम्मेदारियों को चित्रित और स्पष्ट रूप से
परिभाषित किया जाना चाहिए।
समस्या के समाधान के लिए पहल :

त्रिपुरा जनजातीय क्षेत्र स्वायत्त जिला परिषद: शक्तियों के पृथक्करण से संबंधित संवैधानिक प्रावधान:
• त्रिपुरा आदिवासी क्षेत्र स्वायत्त जिला परिषद (TTADC) का • अनुच्छेद 50: न्यायपालिका को कार्यपालिका से अलग करने
गठन 1985 में संविधान की छठी अनुसूची के तहत किया गया के लिए निर्देशक सिद्धांत।
था ताकि आदिवासी समुदायों के विकास और अधिकारों और
• अनुच्छेद 121 और 211: महाभियोग प्रस्ताव को छोड़कर,
सांस्कृतिक विरासत को सुरक्षित किया जा सके।
विधानमंडल न्यायाधीशों के आचरण पर चर्चा नहीं कर सकते

समाचार स्रोत: द इं डियन एक्सप्रेस • अनुच्छेद 123: देश में कार्यपालिका का प्रमुख होने के नाते
राष्ट्रपति कुछ शर्तों के तहत विधायी शक्तियों का प्रयोग कर
सकते हैं।

• अनुच्छेद 361: राष्ट्रपति और राज्यपालों को अदालती कार्यवाही


से छूट प्राप्त है।

14 I प्रबोधन Download Monthly Articulate Magazine


उड़ानों पर कृपाण ले जाना:
• केंद्र सरकार की अधिसूचना के अनुसार, सिख यात्री नागरिक एयरलाइन द्वारा संचालित घरेलू उड़ानों पर निर्दिष्ट आयामों के साथ
कृपाण ले जा सकते हैं।

नियम 267

खबरों में क्यों? • एक सांसद को नियम 267 के तहत मामले पर चर्चा करने के
लिए राज्यसभा के सभापति की मंजूरी ले नी होती है।
राज्यसभा के सभापति ने सदन के नियम 267 के तहत सांसदों के
नोटिस को स्वीकार नहीं किया। • यदि किसी मुद्दे को नियम 267 के तहत स्वीकार किया जाता है,
तो यह दर्शाता है कि यह दिन का सबसे महत्वपूर्ण राष्ट्रीय मुद्दा
• दो नोटिस जारी किए गए थे : चीन के साथ सीमा का मुद्दा और
है।
आवश्यक वस्तुओ ं की बढ़ती कीमतें।
• आखिरी बार 2016 में नोटबंदी पर चर्चा के लिए इसका इस्तेमाल
नियम 267 किया गया था।
• राज्य सभा के प्रक्रिया और कार्य संचालन नियमों के नियम
267 के तहत, एक जरूरी मामले को उठाने के लिए दिन के एक समाचार स्रोत: द हिं दू
सूचीबद्ध कार्य को निलं बित किया जा सकता है।

न्यायालयों में अवकाश

खबरों में क्यों? दिन बाद यह घोषणा की गई।


भारत के मुख्य न्यायाधीश, डी वाई चंद्रचूड़ ने कहा कि शीतकालीन
अवकाश के दौरान शीर्ष अदालत में कोई अवकाश पीठ उपलब्ध नहीं कोर्ट वेकेशन क्या होते हैं ?
होगी। • SC अपनी वार्षि क गर्मी की छुट्टी के लिए ब्रेक लेता है, जो आम
• केंद्रीय कानून मंत्री द्वारा अदालत की लं बी छुट्टियों और इससे तौर पर सात सप्ताह (मई-जुलाई) का होता है, दशहरा और
वादियों को होने वाली असुविधा की आलोचना करने के एक दिवाली के लिए एक सप्ताह का ब्रेक और दिसंबर के अंत में दो
सप्ताह का ब्रेक होता है।

Download Monthly Articulate Magazine प्रबोधन | 15


न्यायालय अवकाश की आलोचना:
न्यायालय प्रति वर्ष कार्य दिवस
• औपनिवेशिक उत्पत्ति: भारत के संघीय न्यायालय के यूरोपीय
सुप्रीम कोर्ट 193 दिन
न्यायाधीशों ने भारतीय गर्मि यों को बहुत गर्म पाया और
हाई कोर्ट 210 दिन क्रिसमस के लिए शीतकालीन अवकाश लिया।

ट्रायल कोर्ट 245 दिन • मामलों की बढ़ती लं बितता: 1 दिसंबर को अदालत की लं बितता
69,598 है वर्ष 2023 के कैलें डर से पता चलता है कि अदालत
लगभग 180 दिनों तक पूरी तरह कार्यात्मक नहीं होगी।
कानूनी प्रावधान:
• प्रशासनिक कार्य में व्यवधान: 2023 के कैलें डर से पता चलता है
• सुप्रीम कोर्ट के नियम, 2013 में प्रावधान है कि छुट्टी के दौरान
कि SC लगभग 180 दिनों तक पूरी तरह कार्यात्मक नहीं रहेगा।
मामलों की सुनवाई के लिए सीजेआई एक या एक से अधिक
न्यायाधीशों की नियुक्ति कर सकते हैं (अवकाश बेंच) सुझाव:
• इसमें तत्काल प्रकृति के सभी मामले (जमानत, बेदखली, • न्यायमूर्ति मालिमथ समिति: अवकाश की अवधि को 21 दिनों
आदि) शामिल हो सकते हैं और वह तत्काल मामलों की सुनवाई से कम किया जाना चाहिए और हर साल एससी को 206 दिनों
के लिए एक डिवीजन कोर्ट भी नियुक्त कर सकता है। और एचसी को 231 दिनों के लिए काम करना चाहिए।

• उदाहरण के लिए, 2015 में, SC की पांच-न्यायाधीशों की • भारतीय विधि आयोग: उच्च न्यायपालिका में छुट्टियों को कम
खंडपीठ ने गर्मी की छुट्टी के दौरान NJAC की स्थापना के लिए से कम 10 से 15 दिनों तक कम किया जाना चाहिए और काम के
संवैधानिक संशोधन की चुनौती पर सुनवाई की। घंटों को कम से कम आधा घंटा बढ़ाया जाना चाहिए।

समीक्षा याचिका

खबरों में क्यों? • कानून का शासन भारत को नियंत्रित करता है, और यदि किसी
• सुप्रीम कोर्ट ने 2002 के गुजरात दंगों की शिकार बिलकिस फैसले में अन्याय होता है, तो इसे एक समीक्षा याचिका के
बानो द्वारा दायर पुनर्वि चार याचिका को खारिज कर दिया है। माध्यम से पलटा जा सकता है।

• समीक्षा याचिका कभी भी किसी अपील की नई सुनवाई नहीं


प्रमुख बिं दु
होती है, और यह पिछले निर्णय में “पेटेंट की गलती” को ठीक
के बारे में करने की क्षमता है।

समीक्षा याचिका • फैसला सुनाए जाने के 30 दिनों के भीतर एक समीक्षा याचिका


• एक समीक्षा याचिका सर्वोच्च न्यायालय की अपने निर्णयों की दायर की जानी चाहिए।
फिर से जांच करने की क्षमता है। » मृत्युदंड के मामलों को छोड़कर, समीक्षा याचिकाओ ं की सुनवाई
न्यायाधीशों द्वारा उनके कक्षों में “परिसंचरण” के माध्यम से की
• सर्वोच्च न्यायालय को संविधान के अनुच्छेद 137 के तहत अपने
जाती है।
स्वयं के निर्णयों की समीक्षा करने की विवेकाधीन शक्तियाँ
प्रदान की गई हैं। » आमतौर पर खुले कोर्ट में उनकी सुनवाई नहीं होती।

16 I प्रबोधन Download Monthly Articulate Magazine


जिन मामलों में समीक्षा याचिका स्वीकार की गई जिन मामलों में समीक्षा याचिका खारिज कर दी गई

• सबरीमाला मामला। • राफेल डील

• अनुसूचित जाति और अनुसूचित जनजाति अत्याचार • राजीव गांधी के दोषियों की रिहाई


अधिनियम।

समाचार स्रोत: द हिं दू पुरानी प्रथा के संबंध में तर्क दिए गए हैं जिसमें सीईसी का
कार्यकाल हमेशा संक्षिप्त रहा है।

चुनाव • इससे चुनाव आयोग का संचालन और प्रभावशीलता बाधित


होती है।
मुख्य चुनाव आयुक्त
वर्तमान प्रावधान:
खबरों में क्यों?
• संविधान निर्दिष्ट करता है कि मुख्य चुनाव आयुक्त और अन्य
सुप्रीम कोर्ट की संविधान पीठ द्वारा जारी एक बयान के अनुसार
चुनाव आयुक्तों की नियुक्ति राष्ट्रपति द्वारा की जाएगी। (अनुच्छेद
मुख्य चुनाव आयुक्त (सीईसी) का छोटा कार्यकाल “कार्यालय की
324)
स्वतंत्रता” को बाधित कर रहा है।.
• सीईसी को उसी तरह से हटाया जाएगा जैसे सर्वोच्च न्यायालय
मुद्दा:
के न्यायाधीश को हटाया जाता है, अर्थात संसदीय प्रक्रिया के
• कई वर्षों से यह देखा गया है कि सीईसी का कार्यकाल अभी माध्यम से। साथ ही, सीईसी की सिफारिश के बाद ही ईसी को
भी बहुत छोटा है। 2014 और 2022 के बीच आठ सीईसी नियुक्त हटाया जा सकता है। (अनुच्छेद 324)
किए गए थे।
• चुनाव आयोग (चुनाव आयुक्तों की सेवा की शर्तें और व्यापार
• 2004 के बाद से किसी भी सीईसी ने अपना छह साल का का ले न-देन) अधिनियम 1991 प्रदान करता है कि सीईसी और
कार्यकाल पूरा नहीं किया है। ईसी छह साल की अवधि के लिए या 65 वर्ष की आयु तक, जो
• किसी ऐसे व्यक्ति को कार्यालय में नियुक्त करने की केंद्र की भी पहले हो, पद धारण करेंगे।

मुद्दा मौजूदा स्थिति एससी का सुझाव

नियुक्ति समिति कैबिनेट की सिफारिश पर राष्ट्रपति द्वारा CJI सहित कॉले जियम-प्रकार का निकाय
नियुक्त

योग्यता उल्ले ख नहीं है टीएन शेषन जैसे काबिल और चरित्रवान लोगों


को लाने के लिए तंत्र की जरूरत है

आगे की राह: परिसीमन आयोग:


• न्यायपालिका ने परंपरागत रूप से ईसीआई के साथ जो संबंध • परिसीमन एक राष्ट्र के भौगोलिक निर्वाचन क्षेत्रों की सीमाओ ं
बनाए रखा है और उसे कायम रखा है, उसके संबंध को बदलने को निर्धारित करने का कार्य या प्रक्रिया है।
का कोई कारण नहीं है।
• अनुच्छेद 82 के अनुसार, प्रत्येक जनगणना के बाद, संसद एक
• नियुक्ति पैनल में भारत के मुख्य न्यायाधीश या विपक्ष के नेता परिसीमन अधिनियम बनाएगी और केंद्र सरकार परिसीमन
को शामिल करने के बारे में संसद को निर्णय ले ने और चर्चा आयोग की स्थापना करेगी।
करने की अनुमति ।
• अनुच्छेद 170 के तहत संवैधानिक जनादेश के अनुसार,
विधानसभा और संसदीय सीटों के पुनर्समायोजन के उद्देश्य से
जनगणना डेटा का उपयोग किया जाएगा।

• यह राष्ट्रपति द्वारा नियुक्त किया जाता है और यह भारत के


परिसीमन अभ्यास
चुनाव आयोग के समन्वय में काम करता है।

ख़बरों में क्यों? • लोकसभा या राज्य विधान सभा परिसीमन आयोग के निर्देशों
को एक बार उनके सामने रखने के बाद उन्हें संशोधित नहीं कर
डीएमके के एक सांसद ने कहा कि प्रभावी जनसंख्या नियंत्रण
सकती हैं।
नीतियों के कारण जनसंख्या आधारित परिसीमन द्वारा दक्षिणी राज्यों
के साथ गलत व्यवहार किया जाता है, और इसलिए प्रतिनिधित्व में • आयोग के निर्णय अंतिम होते हैं और उन्हें कानून की अदालत में
वंचित हैं। चुनौती नहीं दी जा सकती है।

Download Monthly Articulate Magazine प्रबोधन | 17


• पहला आयोग 1950-51 में राष्ट्रपति द्वारा गठित किया गया था • ‘एक वोट, एक मूल्य’ के सिद्धांत का पालन करें।
और बाद में 1952 में एक अधिनियम पारित किया गया था।
असम के परिसीमन के बारे में:
• आज तक, आयोग का गठन 4 बार किया गया: 1952,1963, 1973
• भारत के चुनाव आयोग ने दिसंबर 2022 में असम में विधानसभा
और 2002 में
और संसदीय सीटों के लिए परिसीमन अभ्यास शुरू कर दिया है

आयोग की संरचना: • यह अभ्यास 2011 की जनगणना पर आधारित होगा।


• सुप्रीम कोर्ट के सेवानिवृत्त न्यायाधीश। • अंतिम परिसीमन 1976 में 1971 की जनगणना को आधार
• संबंधित राज्यों के चुनाव आयुक्त बनाकर किया गया था।

• भारत के मुख्य चुनाव आयुक्त। • अनुच्छेद 330 और 332 के अनुसार, क्रमशः अनुसूचित जाति
और अनुसूचित जनजाति समुदायों के लिए सीटों के लिए
परिसीमन की आवश्यकता: आरक्षण होगा।
• आबादी के सभी वर्गों के लिए उचित प्रतिनिधित्व सुनिश्चित • परिसीमन के दौरान, आयोग निम्नलिखित कारकों पर विचार
करने के लिए। करेगा
• भौगोलिक क्षेत्रों को समान रूप से विभाजित किया जाना चाहिए » भौतिक विशेषताऐ ं।
ताकि कोई भी राजनीतिक दल चुनाव को अपने पक्ष में प्रभावित » प्रशासनिक इकाइयों की मौजूदा सीमाएं ।
न कर सके।
» संचार की सुविधा।

» जनता की सुविधा।

अन्य पूर्वोत्तर राज्यों में परिसीमन: आरपी अधिनियम 1950 की


धारा 8 (ए) अरुणाचल प्रदेश, असम, नागालैं ड और मणिपुर में
परिसीमन की अनुमति देती है।

हाल ही में आयोग ने जम्मू-कश्मीर के लिए परिसीमन पूरा


किया और इसने जम्मू-कश्मीर विधानसभा के लिए 7 अतिरिक्त
सीटों और विधानसभा की कुल सीटों की संख्या 83 से बढ़ाकर
90 करने की सिफारिश की।

एक राष्ट्रीय पार्टी की स्थिति

18 I प्रबोधन Download Monthly Articulate Magazine


खबरों में क्यों?
• ईसीआई के राजनीतिक दलों और चुनाव चिह्न, 2019 हैंडबुक
विधानसभा चुनाव में शानदार प्रदर्शन के बाद आम आदमी पार्टी के अनुसार, एक राजनीतिक दल को एक राष्ट्रीय दल माना
(आप) को राष्ट्रीय पार्टी का दर्जा मिलने की पूरी तैयारी है। जाएगा यदि:
प्रमुख बिं दु » यह एक राज्य पार्टी के रूप में चार या अधिक राज्यों में ‘मान्यता
के बारे में प्राप्त’ है; या

राष्ट्रीय पार्टी » यदि उसके उम्मीदवारों को पिछले लोकसभा या विधानसभा


चुनावों में किन्हीं चार या अधिक राज्यों में कुल वैध मतों का
• भारत के चुनाव आयोग (ECI) ने एक पार्टी को राष्ट्रीय पार्टी के
कम से कम 6% प्राप्त हुआ हो और पिछले लोकसभा चुनावों में
रूप में मान्यता देने के लिए तकनीकी मानदंड निर्धारित किए
कम से कम चार सांसद हों, या
हैं।
» यदि उसने कम से कम तीन राज्यों से लोकसभा की कुल सीटों
• इन निर्धारित शर्तों की पूर्ति के आधार पर एक पार्टी समय-समय
में से कम से कम 2% सीटें जीती हों।
पर राष्ट्रीय पार्टी का दर्जा प्राप्त या खो सकती है।

वर्तमान में राष्ट्रीय दल:

क्रम नाम दल

1 भारतीय जनता पार्टी बीजेपी

2 नेशनल पीपल पार्टी एनपीपी

3 ऑल इं डिया तृणमूल कांग्रेस एआईटीसी

4 नेशनलिस्ट कांग्रेस पार्टी एनसीपी

5 बहुजन समाज पार्टी बीएसपी

6 कम्युनिस्ट पार्टी ऑफ इं डिया सीपीआई

7 कम्युनिस्ट पार्टी ऑफ इं डिया(मार्क्सिस्ट) सीपीआई एम

8 इं डियन नेशनल कांग्रेस आईएनसी

9 आम आदमी पार्टी आप

• निर्धारित शर्तों की पूर्ति के आधार पर एक पार्टी समय-समय • उनके उम्मीदवारों को आम चुनाव के दौरान मतदाता सूची की
पर राष्ट्रीय पार्टी का दर्जा प्राप्त या खो सकती है। एक प्रति निःशुल्क मिलती है।

राष्ट्रीय पार्टी को लाभ: समाचार स्रोत: द इं डियन एक्सप्रेस

• एक राष्ट्रीय या एक राज्य पार्टी के रूप में मान्यता यह सुनिश्चित


करती है कि उस पार्टी के चुनाव चिन्ह का उपयोग पूरे भारत
के चुनावों में किसी अन्य राजनीतिक दल द्वारा नहीं किया जा प्रशासनिक मुद्दे
सकता है।
परिवार पहचान पत्र
• मान्यता प्राप्त ‘राज्य’ और ‘राष्ट्रीय’ पार्टी को नामांकन दाखिल
करने के लिए केवल एक प्रस्तावक की आवश्यकता होती है। खबरों में क्यों?

• मान्यता प्राप्त ‘राज्य’ और राष्ट्रीय दलों को चुनाव आयोग द्वारा जम्मू और कश्मीर सरकार ने केंद्र शासित प्रदेश के निवासियों के
मुफ्त में मतदाता सूची के दो सेट दिए जाते हैं। लिए एक परिवार पहचान पत्र (पहचान पत्र) पेश करने की घोषणा
की है।
• उन्हें पार्टी कार्यालय स्थापित करने के लिए मुफ्त भूमि/भवन,
राष्ट्रीय और राज्य टे लीविजन पर टाइम स्लॉट और संदेश समान पहचान पत्र वाले अन्य राज्य:
प्रसारित करने के लिए रेडियो दिया जाता है। • परिवार पहचान पत्र की अवधारणा को लागू करने वाला
• राष्ट्रीय दलों में 40 स्टार प्रचारक हो सकते हैं जबकि अन्य में हरियाणा पहला राज्य था।
20 स्टार प्रचारक हो सकते हैं। स्टार प्रचारकों के आने-जाने और • पंजाब सरकार की विभिन्न सामाजिक सेवा योजनाओ ं के लिए
अन्य खर्चों पर होने वाला खर्च पार्टी प्रत्याशी के चुनावी खर्च में पात्र परिवारों को लाभ के सीधे हस्तांतरण के लिए 2021 में इसे
शामिल नहीं होता है. पेश किया था।

Download Monthly Articulate Magazine प्रबोधन | 19


• उत्तर प्रदेश सरकार ने इसी तरह के उद्देश्यों के लिए यूपी परिवार समाचार स्त्रोत: PIB
कल्याण कार्ड लॉन्च करने का फैसला किया।

• राजस्थान सरकार ने “जन आधार” नाम से कुछ पेश किया


है जिसका उद्देश्य “एक परिवार और एक व्यक्ति की एकल
प्रसारण चैनलों के लिए प्रोग्राम कोड
पहचानकर्ता होना है”।

खबरों में क्यों?


जम्मू-कश्मीर के निवासियों के लिए प्रस्तावित परिवार पहचान पत्र
के बारे में: केंद्र सरकार ने निजी एफएम रेडियो चैनलों को “शराब, ड्रग्स, हथियार
• यह एक पहचान पत्र होगा जिसमें प्रत्येक परिवार और उसके और गैंगस्टर/बंदक
ू संस्कृति का महिमामंडन” करने वाले गाने नहीं
सदस्यों की परिवार के मुखिया के माध्यम से पहचान करने के चलाने का निर्देश दिया है।
लिए एक अद्वितीय आठ अंकों की अल्फ़ान्यूमेरिक संख्या होगी। पहले क्या चिं ताएं उठाई गई थीं?
• यह संघ राज्य क्षेत्र में प्रत्येक परिवार और व्यक्ति के लिए एक • पंजाब और हरियाणा उच्च न्यायालय ने न्यायिक संज्ञान लिया
एकल पहचानकर्ता होगा। है कि ऐसी सामग्री प्रभावशाली उम्र के बच्चों को प्रभावित करती
है।
• फ़ायदे: यह सरकार की कल्याणकारी योजनाओ ं के लिए
उनकी पात्रता की तुरत
ं पुष्टि करेगा। • इससे पहले पंजाब सरकार ने भी इसी तरह का नोटिफिकेशन
जारी कर राज्य में ड्रग कल्चर को ले कर चिंता जताई थी।
उद्दे श्य
• पात्र लाभार्थि यों को कल्याणकारी योजनाओ ं की त्वरित और क्या है निर्देश?
पारदर्शी डोरस्टेप डिलीवरी सुनिश्चित करने के लिए जम्मू- • एफएम चैनलों को सलाह दी जाती है कि वे उसी कार्यक्रम
कश्मीर में परिवारों का एक प्रामाणिक, सत्यापित और और विज्ञापन कोड का पालन करें जिसका पालन ऑल इं डिया
विश्वसनीय डेटाबेस बनाना। रेडियो (एआईआर) द्वारा किया जाता है, जैसा कि समय-समय
पर संशोधित किया जाता है।
• यह प्रामाणिक, अद्यतन जनसंख्या डेटा के आधार पर सरकार
की योजना नीति में मदद करेगा। • इसने अनुमतियों के निलं बन सहित दंडात्मक कार्रवाई की भी
चेतावनी दी है।
चिं ता:
• सरकार द्वारा गोपनीयता/निगरानी रणनीति के नुकसान का आकाशवाणी प्रसारण कोड:
डर। किसी भी व्यक्ति द्वारा ऑल इं डिया रेडियो (AIR) पर प्रसारण
निम्नलिखित की अनुमति नहीं देगा:
समाचार स्रोत: द इं डियन एक्सप्रेस
• मित्र देशों की आलोचना।

• धर्म या समुदायों पर हमला।

• कुछ भी अश्लील या मानहानिकारक।


घर पोर्ट ल
• कानून और व्यवस्था बनाए रखने के खिलाफ हिं सा या कुछ
खबरों में क्यों? भी करने के लिए उकसाना।

राष्ट्रीय बाल अधिकार संरक्षण आयोग ने GHAR पोर्टल लॉन्च किया • न्यायालय की अवमानना क
​​​​ े लिए कुछ भी।
है। • कुछ भी संविधान आदि के लिए अनादर दिखा रहा है।
प्रमुख बिं दु:

के बारे में
• GHAR - ‘गो होम एं ड री-यूनाइट’ के लिए खड़ा है
प्रिंट और डिजिटल मीडिया एसोसिएशन (PADMA)
• उद्दे श्य: बच्चों को बहाली (देश के भीतर भटके हुए बच्चे) और
प्रत्यावर्तन (देशों के बीच यानी राष्ट्रीय सीमाओ ं के पार भटके खबरों में क्यों?
हुए बच्चे)।
सूचना और प्रसारण मंत्रालय ने देश में समाचार और समसामयिक
• मुख्य विशेषताएं मामलों के प्रकाशकों के लिए एक स्व-नियामक निकाय के निर्माण
» किशोर न्याय प्रणाली में बच्चों की डिजिटल ट्रैकिंग और को मंजूरी दे दी है।
निगरानी और दूसरे देश / राज्य / जिले में प्रत्यावर्तित किया
संगठन के बारे में:
जाना है।
• यह डिजिटल समाचार मीडिया चैनलों, जैसे- जागृति वीकली, द
» राज्य के संबंधित किशोर न्याय बोर्ड/बाल कल्याण समिति को फैक्ट इं डिया से आने वाली शिकायतों से निपटेगा।
बच्चों के मामलों का डिजिटल हस्तांतरण।
• पद्मा की संरचना इस प्रकार होगी:

20 I प्रबोधन Download Monthly Articulate Magazine


» पूर्व एचसी न्यायाधीश मूलचंद गर्ग की अध्यक्षता में। आचार संहिता का पालन सुनिश्चित करेगा।आचार संहिता के
पहलुओ ं के साथ प्रकाशकों को प्रदान करेगा:
» सदस्य के रूप में वरिष्ठ नौकरशाह एवं पत्रकार अशोक कुमार
टं डन। » आचार संहिता के पहलु ओ ं के साथ प्रकाशकों को प्रदान करना।

» सदस्य के रूप में पत्रकार एवं ले खक मनोज कुमार मिश्रा। » 15 दिनों के भीतर प्रकाशक का निवारण नहीं होने पर शिकायतों
का समाधान करना।
• I&B मंत्रालय द्वारा दी गई सूची के अनुसार, PADMA से जुड़े 47
डिजिटल समाचार प्रकाशक होंगे। » प्रकाशकों के निर्णय के खिलाफ शिकायतें।

• सूचना प्रौद्योगिकी (मध्यस्थ दिशानिर्देश और डिजिटल मीडिया » प्रकाशकों को आवश्यक जानकारी प्रस्तुत करने के लिए
आचार संहिता) नियम, 2021 के अनुसार, यह प्रकाशकों द्वारा कहना।

डोमेन रेगुलेटर संगठन की प्रकृति

मुद्रित समाचार पत्र/प्रेस प्रेस काउं सिल ऑफ इं डिया वैधानिक, 1978

डिजिटल समाचार प्रिं ट और डिजिटल मीडिया एसोसिएशन स्व-नियामक, 2022

टी वी समाचार • समाचार प्रसारण और डिजिटल मानक • स्व-नियामक, 2007; 2021 में नाम बदला
प्राधिकरण (NBDSA) गया

टीवी गैर-समाचार/सामान्य मनोरंजन • सूचना एवं प्रसारण मंत्रालय BCCC: स्व नियामक निकाय IBF (इं डियन
चैनल ब्रॉडकास्टिं ग फाउं डेशन) द्वारा वर्ष 2011 में
• प्रसारण सामग्री शिकायत परिषद
स्थापित
(BCCC),

ओटीटी प्ले टफॉर्म सूचना एवं प्रसारण मंत्रालय _

टिप्पणी:
‘नई चेतना’ अभियान
• टीवी चैनलों को केबल टे लीविजन नेटवर्क (विनियमन)
• यह लिंग आधारित हिं सा से लड़ने के लिए महिलाओ ं के लिए
अधिनियम, 1995 और सिनेमैटोग्राफ अधिनियम 1952 के
उपलब्ध संस्थागत तंत्र के बारे में जागरूकता बढ़ाने का एक
तहत विनियमित किया जाता है।
अभियान है।
• सूचना प्रौद्योगिकी (मध्यस्थों के लिए दिशानिर्देश और
• ग्रामीण विकास मंत्रालय द्वारा शुरू किया गया है।
डिजिटल मीडिया आचार संहिता) नियम 2021 के अनुसार
ओटीटी प्ले टफार्मों को विनियमित किया जाता है। समाचार स्रोत: पीआईबी

एसिड अटै क

खबरों में क्यों?

दिल्ली में एक लड़की पर तेजाब जैसे रसायन से हमले ने गंभीर मुद्दे को फिर से तूल दे दिया है।

एसिड अटै क पर कानून 2013 से पहले 2013 के बाद

वर्गीकरण अलग अपराध के रूप में मान्यता प्राप्त नहीं IPC की नई धारा 326A घरेलू हिं सा
थी। अधिनियम 2005 के प्रावधानों के साथ
इससे संबंधित है।

Download Monthly Articulate Magazine प्रबोधन | 21


सज़ा 10 साल तक की कैद और जुर्माना। न्यूनतम 10 वर्ष का कारावास और
अधिकतम आजीवन कारावास और 10 लाख
रुपए तक का न्यूनतम जुर्माना ।

मुआवज़ा पीड़ितों के लिए कोई प्रावधान नहीं मुआवजा पाने की हकदार है, जिसका
भुगतान राज्य सरकार करेगी

एसिड की बिक्री को कैसे नियंत्रित किया जाता है? पीड़ित देखभाल और मुआवजा:
• गृह मंत्रालय ने तेजाब की बिक्री को रेगुलेट करने और वेंडर्स के • सुप्रीम कोर्ट के आदेशों के बाद, गृह मंत्रालय (एमएचए) ने
रजिस्ट्रेशन के लिए राज्य सरकार को गाइडलाइं स जारी की हैं। राज्यों को यह सुनिश्चित करने का निर्देश दिया कि एसिड अटै क
पीड़ितों को कम से कम 3 लाख रुपये मिले । बाद की देखभाल
• विक्रेताओ ं को 15 दिनों के भीतर संबंधित सब-डिवीजनल
और पुनर्वास की लागत को कवर करने के लिए संबंधित राज्य
मजिस्ट्रेट (एसडीएम) के समक्ष एसिड के सभी स्टॉक की घोषणा
सरकार / केंद्र शासित प्रदेश से मुआवजे के रूप में किसी भी
करनी होती है।
सरकारी या निजी
• एसडीएम तेजाब के अघोषित स्टॉक को जब्त कर सकता है और
• अस्पताल में तेजाब हमले के पीड़ितों की देखभाल राज्य द्वारा
किसी भी निर्देश का उल्लं घन करने पर 50,000 रुपये तक का
बिना किसी शुल्क के प्रदान किए जाने की अपेक्षा की जाती है।
जुर्माना लगा सकता है।
• सरकार ने विकलांग व्यक्तियों के अधिकार अधिनियम, 2016
के तहत विकलांग श्रेणी में एसिड अटै क सर्वाइवर्स को शामिल
करने की घोषणा की, ताकि वे आरक्षण का लाभ उठा सकें।

पाकिस्तान में आत्महत्या को अपराध की श्रेणी से बाहर किया गया

पहले : पाकिस्तान दंड संहिता, 1860 2023 सुधार

आत्महत्या के लिए जेल बीमारी के रूप में इलाज किया जाएगा।

भारत में स्थिति:


• राष्ट्रीय अपराध रिकॉर्ड ब्यूरो के अनुसार, 2021 में देश में कुल » बीमारी (18.6%)
1,64,033 आत्महत्याएं दर्ज की गईं, जो कि 7.2% की वृद्धि है।
» ड्रग/अल्कोहल संबंधी (6.4%)
• एनसीआरबी (2021) के अनुसार आत्महत्या के विभिन्न कारणों
» विवाह संबंधी(4.8%)
के लिए प्रतिशत हिस्सेदारी:
» अन्य: प्रेम प्रसंग, दिवालियापन, बेरोजगारी।
» पारिवारिक समस्याएं (33.2%)

भारत में कानून:

आईपीसी की धारा 309 मानसिक स्वास्थ्य देखभाल अधिनियम 2017

आत्महत्या का प्रयास एक वर्ष तक के कारावास या जुर्माना, या अधिनियम की धारा 115: आत्महत्या करने का प्रयास करने वाले
दोनों के साथ दंडनीय है व्यक्ति को, जब तक कि अन्यथा सिद्ध न किया जाए, अत्यधिक
तनाव में माना जाएगा और उक्त कोड के तहत मुकदमा नहीं चलाया
आत्महत्या के लिए उकसाना या सहायता करना भी धारा 306 के
जाएगा और दंडित नहीं किया जाएगा।
तहत दंडनीय है

इस प्रकार, आईपीसी की धारा 309 जो आत्महत्या के प्रयास को


आपराधिक बनाती है और मानसिक स्वास्थ्य देखभाल अधिनियम,
2017 जो अभियोजन को रोकता है, के बीच एक संघर्ष मौजूद है।

आत्महत्या को अपराध की श्रेणी से बाहर करने की सिफारिशें: • सुप्रीम कोर्ट ने 2018 में कॉमन कॉज़ बनाम भारत संघ मामले
• विधि आयोग ने अपनी 42वीं रिपोर्ट (1971) में आईपीसी की धारा में सिफारिश की थी कि संसद अपराध को कम करने पर विचार
309 को निरस्त करने की सिफारिश की थी। करे।

22 I प्रबोधन Download Monthly Articulate Magazine


राष्ट्रीय आत्महत्या रोकथाम रणनीति, 2022:

नोडल मंत्रालय स्वास्थ्य और परिवार कल्याण मंत्रालय

लक्ष्य • 2030 तक देश में आत्महत्या मृत्यु दर को 10% तक कम करना,

• सार्वजनिक स्वास्थ्य प्राथमिकता के रूप में आत्महत्या को


रोकना

उद्दे श्य • प्रभावी निगरानी तंत्र स्थापित करना,

• मनोरोग बाह्य रोगी विभागों की स्थापना,

• सभी शैक्षणिक संस्थानों में मानसिक कल्याण पाठ्यक्रम को


एकीकृत करना

पांच प्रमुख हितधारक • राष्ट्रीय स्तर के मंत्रिस्तरीय हितधारक,

• राज्य स्तर के सरकारी हितधारक,

• जिला स्तर के सरकारी हितधारक,

• निमहांस-बैंगलोर और अन्य शीर्ष मानसिक स्वास्थ्य संस्थान,

• रणनीतिक सहयोगी

आत्महत्या रोकथाम के लिए जारी पहल: खबरों में क्यों?


• राष्ट्रीय मानसिक स्वास्थ्य नीति (2014), • संसद ने नई दिल्ली अंतर्राष्ट्रीय मध्यस्थता केंद्र (संशोधन)
विधेयक, 2022 पारित किया है।
• मानसिक स्वास्थ्य देखभाल अधिनियम 2017,
• यह नई दिल्ली अंतर्राष्ट्रीय मध्यस्थता केंद्र अधिनियम, 2019 में
• राष्ट्रीय मानसिक स्वास्थ्य कार्यक्रम,
संशोधन करेगा।
• राष्ट्रीय उपशामक देखभाल कार्यक्रम, (अंततः बीमार रोगियों
• पूर्व एससी न्यायाधीश हेमंत गुप्ता को नई दिल्ली अंतर्राष्ट्रीय
के लिए)
मध्यस्थता केंद्र (एनडीआईएसी) के प्रमुख के रूप में नियुक्त
• नशा मुक्ति अभियान टास्क फोर्स। किया गया था।

विधेयक के बारे में:

शीतकालीन सत्र बिल 2022 • यह अधिनियम नई दिल्ली अंतर्राष्ट्रीय मध्यस्थता केंद्र को


“राष्ट्रीय महत्व” के संस्थान के रूप में नामित करने का प्रावधान
नई दिल्ली अंतर्राष्ट्रीय मध्यस्थता केंद्र (संशोधन) विधेयक
करता है।
2022
• यह कानून और न्याय मंत्रालय के अंतर्गत आता है।

पुनः नामकरण • विधेयक नई दिल्ली अंतर्राष्ट्रीय मध्यस्थता केंद्र का नाम


बदलकर भारत अंतर्राष्ट्रीय मध्यस्थता केंद्र करता है।

वैकल्पिक विवाद समाधान (एडीआर) • मूल अधिनियम में एनडीआईएसी को अंतरराष्ट्रीय और घरेलू
मध्यस्थता और सुलह की सुविधा की आवश्यकता है। संशोधन
विधेयक एडीआर के अन्य रूपों को शामिल करने के लिए
इसका विस्तार करता है।

• मध्यस्थता और वैकल्पिक विवाद समाधान के आचरण का


ढं ग अन्य रूप केंद्र सरकार द्वारा नियमों के माध्यम से निर्दिष्ट
होंगे।

उद्दे श्य: रिक्त स्थान और प्रशासनिक सहायता प्रदान करना।


• वैकल्पिक संघर्ष समाधान से संबंधित विषयों पर अनुसंधान को
• मध्यस्थता, मध्यस्थता और सुलह सत्रों का नेतृत्व करने के लिए
प्रोत्साहित करना, प्रशिक्षण देना और सम्मेलनों और सेमिनारों
योग्य विशेषज्ञों की एक टीम बनाए रखना।
की योजना बनाना।

• मध्यस्थता, मध्यस्थता और सुलह सत्रों के संचालन के लिए स्रोत: द हिं दू

Download Monthly Articulate Magazine प्रबोधन | 23


नई दिल्ली अंतर्राष्ट्रीय मध्यस्थता केंद्र (एनडीआईएसी)

ख़बरों में क्यों? (आईसीएडीआर) की स्थापना।

पूर्व एससी न्यायाधीश हेमंत गुप्ता को नई दिल्ली अंतर्राष्ट्रीय • 2019: NDIAC की स्थापना नई दिल्ली अंतर्राष्ट्रीय मध्यस्थता
मध्यस्थता केंद्र (एनडीआईएसी) के प्रमुख के रूप में नियुक्त किया केंद्र अधिनियम, 2019 के तहत की गई थी
गया था। • NDIAC ने वैकल्पिक विवाद समाधान के लिए अंतर्राष्ट्रीय केंद्र
एनडीआईएसी के बारे में (ICADR) का स्थान लिया
• 1995: वैकल्पिक विवाद समाधान के लिए अंतर्राष्ट्रीय केंद्र • संरचना: NDIAC में सात सदस्य होते हैं।

संगठनात्मक संरचना विवरण

अध्यक्ष सर्वोच्च न्यायालय या उच्च न्यायालय के न्यायाधीश, या मध्यस्थता


के संचालन या प्रशासन में विशेष ज्ञान और अनुभव वाले एक
प्रतिष्ठित व्यक्ति।

दो प्रतिष्ठित व्यक्ति संस्थागत मध्यस्थता में पर्याप्त ज्ञान और अनुभव

तीन पदेन सदस्य वित्त मंत्रालय से एक नामिती और एक मुख्य कार्यकारी अधिकारी


(एनडीआईएसी के दिन-प्रतिदिन के प्रशासन के लिए जिम्मेदार)
सहित

एक अंशकालिक सदस्य वाणिज्य और उद्योग के एक मान्यता प्राप्त निकाय से प्रतिनिधि


(घूर्णी आधार)

• कार्यकाल: अध्यक्ष की सेवानिवृत्ति की आयु 70 वर्ष है, और अन्य (प्रावधानों का संशोधन) बिल, 2022 पेश किया।
सदस्यों की सेवानिवृत्ति की आयु 67 वर्ष है।
उद्देश्य:

यह कई छोटे अपराधों को कम करने के लिए मौद्रिक दंड के साथ


बदलने का सुझाव देता है।
जन विश्वास (प्रावधानों में संशोधन) एक बार जब उपाय कानून में पारित हो जाता है, तो प्रस्ताव का
एक विशिष्ट पहलू प्रत्येक तीन साल की समाप्ति के बाद निर्धारित
खबरों में क्यों? न्यूनतम जुर्माने और जुर्माने की राशि में 10% की वृद्धि है।
केंद्रीय वाणिज्य और उद्योग मंत्री ने लोकसभा में जन विश्वास

24 I प्रबोधन Download Monthly Articulate Magazine


विधेयक के प्रमुख प्रावधान:

कुछ अपराधों को कम करना • कुछ ऐसे अपराध जिनमें पहले कुछ कानूनों के तहत कारावास
होता था, अब केवल मौद्रिक दंड की आवश्यकता है।

• उदाहरण के लिए:
» आईटी अधिनियम, 2000 - कानूनी अनुबंध के उल्लं घन में
व्यक्तिगत जानकारी का खुलासा करने पर 3 साल तक की
कैद, या 5 लाख रुपये तक का जुर्माना, या दोनों हो सकते हैं।

» बिल इसके स्थान पर 25 लाख रुपए तक के जुर्माने का


प्रावधान करता है

जुर्माने और दंड में संशोधन • बिल के तहत कुछ अपराधों के लिए जुर्माने और दंड में वृद्धि
की गई है।

• इसमें प्रत्येक तीन वर्षों में न्यूनतम राशि के 10% की वृद्धि की


जाएगी।

निर्णायक अधिकारियों की नियुक्ति • बिल केंद्र सरकार को दंड निर्धारित करने के लिए एक या
एक से अधिक सहायक अधिकारियों को नियुक्त करने की
अनुमति देता है।

• निर्णायक अधिकारी कर सकते हैं-


» व्यक्तियों को साक्ष्य के लिए बुलाना

» सम्मानित अधिनियमों के उल्लं घन की जांच करें

अपीलीय तंत्र • विधेयक में न्यायनिर्णयन अधिकारियों द्वारा पारित आदेशों


के खिलाफ अपील करने के लिए व्यक्तियों के लिए प्रावधान
शामिल हैं।

• उदाहरण के लिए, पर्यावरण (संरक्षण) अधिनियम, 1986 में


आदेश के 60 दिनों के भीतर राष्ट्रीय हरित अधिकरण में अपील
दायर की जा सकती है।

यह बिल व्यापार करने में आसानी को और सुविधाजनक कैसे इस बिल द्वारा प्रदान करने वाले कुछ और लाभ निम्नलिखित हैं:
बनाएगा? • इससे जनता का विश्वास बढ़ेगा और उनमें और अधिक विश्वास
• मामूली अपराधों के लिए कारावास व्यापार विकास और पैदा होगा।
व्यक्तिगत आत्मविश्वास में बाधा डालता है।
• मानवाधिकारों के उल्लं घन को रोका जा सकता है और जेल में
• पुराने नियम और विनियम एक विश्वास घाटा पैदा करते हैं। भीड़भाड़ से बचा जा सकता है।

• इसलिए, प्राथमिकता “न्यूनतम सरकार, अधिकतम शासन” • इसके अतिरिक्त, प्रस्तावित कानून न्यायपालिका के कार्यभार
सुधारों के माध्यम से देश के विनियामक परिदृश्य में सुधार को हल्का करने में सहायता करेगा।
करना है ताकि जीवन यापन में आसानी और व्यापार करने में
• इसके अतिरिक्त, यह सरकारी मुकदमेबाजी को कम करने में
आसानी हो।
सहायता करेगा।

Download Monthly Articulate Magazine प्रबोधन | 25


एं टी-मैरीटाइम पाइरेसी बिल 2019-2022

खबरों में क्यों? चुनौतियां:

दिसंबर 2022 में संसद के शीतकालीन सत्र में राज्यसभा ने एं टी- • अनिवार्य मौत की सजा का प्रावधान सुप्रीम कोर्ट के दिशानिर्देशों
मैरीटाइम पाइरेसी बिल 2019 पारित किया। के खिलाफ है जो कहते हैं कि अनिवार्य मौत की सजा ए-14 और
ए-21 के खिलाफ है।
विधेयक की आवश्यकता क्यों है?
• ओवरलै पिंग प्रावधान जैसे चोरी में भाग ले ने वाला व्यक्ति
• संचार के समुद्री मार्गों की सुरक्षा भारत के लिए महत्वपूर्ण है,
(14 साल की कैद) और किसी व्यक्ति की स्वैच्छिक भागीदारी
क्योंकि 90% वैश्विक व्यापार और 80% से अधिक ऊर्जा की
(आजीवन कारावास)। इन परिस्थितियों में, यह स्पष्ट नहीं है कि
जरूरतें समुद्र के माध्यम से पूरी होती हैं।
सजा कैसे निर्धारित की जाएगी।
• भारत ने 1995 में समुद्र के कानून पर संयुक्त राष्ट्र सम्मेलन
• बिल की भौगोलिक प्रयोज्यता का प्रावधान एक अन्य महत्वपूर्ण
(यूएनसीएलओएस) की पुष्टि की।
चुनौती है, क्योंकि संप्रभुता के अधिकार 12 समुद्री मील पर लागू
• वर्तमान में, समुद्री डकैती से संबंधित कोई कानून नहीं है| होते हैं।

विधेयक के बारे में: स्रोत: द इं डियन एक्सप्रेस


प्रमुख विशेषताएँ :
• इसका उद्देश्य समुद्री डकैती जैसे अपराधों में शामिल व्यक्तियों
द्वारा चोरी और उत्पीड़न को रोकना है। ऊर्जा संरक्षण (संशोधन) विधेयक, 2022
• बिल का क्षेत्राधिकार भारत के विशिष्ट आर्थि क क्षेत्र (ईईजेड) के
खबरों में क्यों?
निकट और उसके बाहर लागू होता है।
अगस्त 2022 में लोकसभा में पेश किया गया यह बिल अब राज्यसभा
बिल के तहत पाइरेसी की परिभाषा: द्वारा शीतकालीन सत्र में पारित कर दिया गया है।
• किसी जहाज, विमान, व्यक्ति या संपत्ति के खिलाफ की गई विधेयक के प्रमुख प्रावधान:
हिं सा, हिरासत या विनाश का कोई भी अवैध कार्य;
• कार्बन क्रेडिट ट्रेडिंग योजना निर्दिष्ट करने के लिए केंद्र सरकार
• उकसाना या जानबूझकर अवैध कार्यों को सुविधाजनक को सशक्त बनाना।
बनाना;
• तेजी से डीकार्बोनाइजेशन के लिए गैर-जीवाश्म स्रोतों और
• एक समुद्री डाकू जहाज के संचालन में स्वेच्छा से भाग ले ना। कार्बन क्रेडिट ट्रेडिंग का अनिवार्य उपयोग।

विधेयक के तहत अपराध और दंड: • वाहनों और जहाजों के लिए ऊर्जा खपत मानकों के उल्लं घन के
लिए जुर्माना बढ़ाना।
• आजीवन कारावास या मौत अगर समुद्री डकैती के कारण मौत
हो जाती है या मौत का कारण बनने का प्रयास किया जाता है। ऊर्जा संरक्षण अधिनियम, 2001:
• 14 साल की कैद और जुर्माना। • ऊर्जा के कुशल उपयोग को बढ़ावा देता है और भारत में ऊर्जा की
• अपराध प्रत्यर्पण योग्य हैं, जिसका अर्थ है कि भारत आरोपी को बर्बादी को कम करता है।
किसी भी देश में भेज सकता है जिसके साथ उसने प्रत्यर्पण संधि • ऊर्जा दक्षता ब्यूरो (बीईई) की स्थापना करता है, जो ऊर्जा दक्षता
पर हस्ताक्षर किए हैं। को बढ़ावा देने के लिए जिम्मेदार है।

26 I प्रबोधन Download Monthly Articulate Magazine


• उद्योग और वाणिज्य में ऊर्जा-कुशल तकनीकों और प्रथाओ ं के संविधान (अनुसूचित जाति और अनुसूचित जनजाति) आदेश
उपयोग और ऊर्जा-कुशल बिल्डिं ग कोड को प्रोत्साहित करता है (संशोधन) विधेयक, 2022
• ऊर्जा ऑडिट, रेट्रोफिट और ऊर्जा कुशल प्रौद्योगिकियों के
खबरों में क्यों?
उपयोग सहित ऊर्जा संरक्षण उपायों को अपनाने को प्रोत्साहित
करने के लिए वित्तीय प्रोत्साहन प्रदान करना एससी और एसटी की सूचियों को संशोधित करने के लिए
शीतकालीन सत्र में कई संशोधन पेश किए गए और पारित किए गए।
• ऊर्जा संरक्षण निधि की स्थापना करता है, जिसका उपयोग ऊर्जा
संरक्षण गतिविधियों को वित्तपोषित करने के लिए किया जाता ये संविधान आदेश क्या हैं ?
है। • संविधान (अनुसूचित जाति) आदेश, 1950 और संविधान
(अनुसूचित जनजाति) आदेश, 1950 कानूनी उपकरण हैं जो
ऊर्जा दक्षता के लिए भारत की प्रतिबद्धता: विशिष्ट समुदायों को अनुसूचित जाति और अनुसूचित जनजाति
COP-26 शिखर सम्मेलन (2021) के दौरान, निम्नलिखित के रूप में मान्यता देते हैं।
प्रतिबद्धताएँ की गईं: • अनुच्छेद 341 और 342 के तहत जारी ये भारत के राष्ट्रपति को
• 2030 तक कुल अनुमानित कार्बन उत्सर्जन को एक बिलियन समुदायों की सूची निर्दिष्ट करने का अधिकार देते हैं।
टन कम करना,
• भारत का संविधान इन समुदायों के सामाजिक, शैक्षिक और
• 2005 के स्तर की तुलना में 2030 तक अर्थव्यवस्था की कार्बन आर्थि क उत्थान को सुनिश्चित करने के लिए सकारात्मक
तीव्रता को 45% तक कम करना। [कार्बन तीव्रता को सकल कार्रवाई प्रदान करता है।
घरेलू उत्पाद की प्रति इकाई कार्बन उत्सर्जन की मात्रा के रूप में
परिभाषित किया गया है।] किन समूहों को शामिल/बहिष्कृ त किया गया है ?

• 500 GW गैर-जीवाश्म ऊर्जा क्षमता और 2030 तक अनुसूचित जाति की सूची


नवीकरणीय स्रोतों से ऊर्जा आवश्यकताओ ं का 50%। • झारखंड में भोगटा समुदाय को हटा दिया गया और एसटी सूची
में स्थानांतरित कर दिया गया।
समाचार स्रोत: इं डियन एक्सप्रेस
समुदायों को एसटी सूची में जोड़ा गया

राज्य समुदाय/जनजाति का नाम

झारखंड देशवरी, गंझू, दौलबन्दी (द्वालबंदी), पटबंदी, राउत, मझिया, खैरी


(खीरी), तमरिया (तमाड़िया), और पूरन

त्रिपुरा कुकी जनजाति की एक उप-जनजाति के रूप में डार्लॉन्ग समुदाय

कर्नाटक कडु कुरुबा के पर्याय के रूप में बेट्टा-कुरुबा

तमिलनाडु नारीकोरवन और कुरीविककरण

एससी और एसटी की सूची कैसे अपडेट की जाती है? सांस्कृतिक स्थितियाँ भिन्न हैं।
• राष्ट्रीय अनुसूचित जाति आयोग (NCSC) और राष्ट्रीय • संविधान निर्दिष्ट करता है कि भारत के राष्ट्रपति के पास
अनुसूचित जनजाति आयोग (NCST) की सिफारिशों के आधार समुदायों के नाम निर्दिष्ट करने वाली “अधिसूचना” बनाने की
पर सूचियों की समय-समय पर समीक्षा की जाती है। शक्ति है।
• भारत में प्रत्येक राज्य और केंद्रशासित प्रदेश के लिए सूचियाँ • इसके बाद, इन सूचियों में कोई भी परिवर्तन संवैधानिक
अलग-अलग हैं, क्योंकि देश भर में सामाजिक-आर्थि क और संशोधन द्वारा किया जाना चाहिए।

Download Monthly Articulate Magazine प्रबोधन | 27


28 I प्रबोधन Download Monthly Articulate Magazine
फर्स्ट लॉस डिफॉल्ट गारंटी(FLDG)

छवि स्त्रोत:mrunal.org
संस्थाओ ं (RE) के ऋण पोर्टफोलियो में डिफ़ॉल्ट के एक निश्चित
खबरों में क्यों? प्रतिशत तक क्षतिपूर्ति की गारंटी देता है।
हाल ही में बैंकों, एनबीएफसी और फिनटे क ने आरबीआई से फर्स्ट • विनियमित संस्थाओ ं में वाणिज्यिक बैंक, क्षेत्रीय ग्रामीण बैंक,
लॉस डिफॉल्ट गारंटी (एफएलडीजी) पर स्पष्टीकरण मांगा है। शहरी सहकारी बैंक, गैर-बैंकिंग वित्तीय कंपनियां आदि शामिल
प्रमुख बिं दु: हैं।

फर्स्ट लॉस डिफॉल्ट गारंटी (FLDG) के बारे में: • इसके तहत फिनटे क एक ऋण की शुरुआत करता है और
ग्राहकों द्वारा चुकाने में विफल रहने की स्थिति में भागीदारों को
• FLDG एक फिनटे क और एक विनियमित संस्था के बीच एक
पूर्व-निर्धारित प्रतिशत तक मुआवजा देने का वादा करता है।
ऋण देने वाला मॉडल है जिसमें एक तीसरा पक्ष विनियमित

आरबीआई ने यूसीबी के लिए 4-स्तरीय नियामक ढांचे की घोषणा की

छवि स्त्रोत: mrunal.org

Download Monthly Articulate Magazine प्रबोधन | 29


खबरों में क्यों? वर्गीकरण की आवश्यकता:

भारतीय रिजर्व बैंक (आरबीआई) ने शहरी सहकारी बैंकों (अर्बन • सभी आकार के यूसीबी के बीच पारस्परिकता और सहयोग की
को ऑपरेटिव बैंक - यूसीबी) के वर्गीकरण के लिए चार स्तरीय भावना को संतुलित करना।
नियामक ढांचे की घोषणा की। • अपने संचालन के क्षेत्र का विस्तार करने और अधिक जटिल
प्रमुख बिं दु: व्यावसायिक गतिविधियों को करने के लिए।

• चार-स्तरीय नियामक ढांचा यूसीबी की जमा राशि के आकार • यूसीबी की वित्तीय सुदृढ़ता को मजबूत करने के लिए।
पर आधारित होगा और तत्काल प्रभाव से लागू होगा।

नियामक उद्देश्यों के लिए निम्नलिखित चार स्तर:

टियर 1 100 करोड़ रुपये तक जमा राशि वाले यूसीबी

टियर 2 100 करोड़ रुपये से अधिक और 1000 करोड़ रुपये तक की जमा राशि वाले यूसीबी

टियर 3 1000 करोड़ रुपये से अधिक और 10,000 करोड़ रुपये तक की जमा राशि वाले यूसीबी।

टियर 4 10,000 करोड़ रुपये से अधिक जमा राशि वाले यूसीबी।

सहकारी बैंकों के बारे में: सहकारी समिति अधिनियम, 2002 के तहत पंजीकृत हैं।
• यह साधारण बैंकिंग व्यवसाय से निपटने के लिए सहकारी • वे मोटे तौर पर शहरी और ग्रामीण सहकारी बैंकों में विभाजित हैं।
आधार पर स्थापित एक संस्था है।
शहरी सहकारी बैंक (यूसीबी):
• सहकारी बैंकों की स्थापना शेयरों के माध्यम से धन एकत्र
करने, जमा स्वीकार करने और ऋण देने से होती है। • वे शहरी और अर्ध-शहरी क्षेत्रों में स्थित प्राथमिक सहकारी बैंक
हैं।
• वे संबंधित राज्य के सहकारी समिति अधिनियम या बहु-राज्य

विशेषता व्यावसायिक बैंक सहकारी बैंक

स्वामित्व के प्रकार निजी सहयोगी

बेसिक कार्य लाभ पर केंद्रित सेवा उन्मुख

शासन शेयरधारकों द्वारा चुने गए निदेशक मंडल सदस्यों द्वारा चुने गए निदेशक मंडल द्वारा
शासित

वोटिंग शेयर होल्डिं ग पर निर्भर करता है एक व्यक्ति,एक वोट,योगदान पूँजी को ध्यान


में न रखते हुए त

समाचार स्रोत: लाइवमिं ट

सेंट्रल डिपॉजिटरीज सर्वि सेज इं डिया लिमिटे ड

खबरों में क्यों?

संदिग्ध साइबर हमले के कारण सेंट्रल डिपॉजिटरी सर्वि सेज इं डिया लिमिटेड (सीडीएसएल) की कुछ सेवाएं बाधित हो गईं।

30 I प्रबोधन Download Monthly Articulate Magazine


प्रमुख बिं दु:

के बारे में डिपॉजिटरी

सेंट्रल डिपॉजिटरीज सर्विसेज इं डिया लिमिटे ड (सीडीएसएल) • एक डिपॉजिटरी डिमटे रियलाइज्ड रूप में प्रतिभूतियों को रखने
के लिए एक सुविधा और प्रतिभूतियों के ले नदेन के लिए एक
• सीडीएसएल सेबी द्वारा पंजीकृत शेयर डिपॉजिटरी है।
सक्षमकर्ता है।
» शेयर डिपॉजिटरी शेयरों को इले क्ट्रॉनिक रूप में रखते हैं।
• यह बॉम्बे स्टॉक एक्सचेंज द्वारा स्टेट बैंक ऑफ इं डिया, यूनियन मार्केट इं फ्रास्ट्रक्चर इं स्टीट्यूशंस क्या हैं ?
बैंक ऑफ इं डिया, बैंक ऑफ बड़ौदा, बैंक ऑफ इं डिया और स्टैंडर्ड • स्टॉक एक्सचेंज, डिपॉजिटरी और क्लियरिंग हाउस सभी मार्के ट
चार्टर्ड बैंक के सहयोग से आयोजित किया जाता है। इं फ्रास्ट्रक्चर इं स्टीट्यूशंस हैं और देश के महत्वपूर्ण आर्थि क
• यह एक मार्केट इं फ्रास्ट्रक्चर इं स्टीट्यूशन है जिसे पूंजी बाजार इं फ्रास्ट्रक्चर का एक महत्वपूर्ण हिस्सा हैं।
संरचना का एक महत्वपूर्ण हिस्सा माना जाता है। » एक समाशोधन गृह एक वित्तीय संस्थान है जो भुगतान,
प्रतिभूतियों या डेरिवेटिव ले नदेन के आदान-प्रदान की सुविधा
सीडीएसएल के लाभ के लिए बनाया गया है।
• चूंकि शेयर इले क्ट्रॉनिक रूप में रखे जाते हैं, इसलिए चोरी, हानि • ई अर्थव्यवस्था में धन के इष्टतम उपयोग में मदद करते हैं और
या क्षति का कोई जोखिम नहीं होता है। आर्थि क विकास को बढ़ावा देते हैं।
• इले क्ट्रॉनिक स्वरूप में प्रतिभूतियों के हस्तांतरण के दौरान
स्टाम्प शुल्क बचाता है। समाचार स्रोत: द इं डियन एक्सप्रेस

Download Monthly Articulate Magazine प्रबोधन | 31


राज्य वित्त और ऋण स्तर

खबरों में क्यों?

केंद्र वित्त वर्ष 24 में जीएसडीपी के 3.5% पर राज्यों के लिए उधार ले ने की सीमा निर्धारित करने के लिए तैयार है।

प्रमुख बिं दु: साथ उधार ले ने की सीमा निर्धारित की है, जिससे कुल जगह
• केंद्र वित्त वर्ष 24 में राज्यों के लिए सकल राज्य घरेलू उत्पाद 4% हो जाती है।
(जीएसडीपी) के 3.5% पर उधार ले ने की सीमा निर्धारित करेगा, • इन सुधारों में शामिल हैं: परिचालन घाटे में कमी, राजस्व अंतर,
जिसमें बिजली क्षेत्र के सुधारों से जुड़े 50 आधार अंक शामिल हैं। राजस्व के प्रतिशत के रूप में प्रत्यक्ष लाभ हस्तांतरण और टै रिफ
सब्सिडी को अपनाकर नकद सब्सिडी का भुगतान।
• राज्यों के लिए एफसी द्वारा अनुशंसित राजकोषीय घाटा सीमा:

वर्ष राजकोषीय घाटे की सीमा की सिफारिश वित्त आयोग के बारे में:


(जीएसडीपी के % के रूप में) • संविधान के अनुच्छेद 280 के तहत स्थापित एक संवैधानिक
2021-22 4 निकाय जो राजकोषीय संघवाद के केंद्र में है।

2022-23 3.5 • यह राज्यों के बीच इन करों के वितरण का निर्धारण करने


वाले सिद्धांत निर्धारित करता है।
2023-26 3
• पहला वित्त आयोग 1951 में स्थापित किया गया था और अब
• यदि राज्य बिजली क्षेत्र में सुधार करते हैं, तो केंद्र ने वित्त वर्ष 23 तक पंद्रह हो चुके हैं।
के लिए जीएसडीपी के 3.5% पर एक और 50 बीपीएस विंडो के • 15वें वित्त आयोग के अध्यक्ष श्री एन.के. सिंह।

32 I प्रबोधन Download Monthly Articulate Magazine


पेरिस क्लब

खबरों में क्यों? अधिकारों (आईपीआर) के एक घटक के रूप में शामिल हैं।
पेरिस क्लब ने श्रीलं काई कर्ज पर 10 साल की मोहलत का प्रस्ताव • भारत में: वस्तुओ ं का भौगोलिक संकेत (पंजीकरण और संरक्षण
रखा। अधिनियम), 1999 इसे नियंत्रित करता है।
• पेरिस क्लब 22 देशों के अधिकारियों का एक समूह है जो » भौगोलिक संकेत (जीआई) रजिस्ट्री चेन्नई में स्थित है।
कर्जदार देशों के सामने आने वाली चुनौतियों का स्थायी • ट्रेडमार्क अधिनियम, 1999 के तहत नियुक्त पेटेंट, डिजाइन और
समाधान ढूंढता है। ट्रेडमार्क महानियंत्रक, भौगोलिक संकेतक के रजिस्ट्रार के रूप
• प्रमुख सदस्य: यूके, यूएसए, जापान में कार्य करता है।

• भारत सदस्य नहीं है और तदर्थ आधार पर भाग लेता है। • एक जीआई टै ग 10 साल के लिए वैध होता है और इसका
नवीनीकरण किया जा सकता है।
जी20 कॉमन फ्रेमवर्क
» जीआई टै ग के लिए जुर्माने में 6 महीने से 3 साल तक की कैद
• यह ऋण सेवा निलं बन पहल (DSSI) से परे ऋण उपचार के और पचास हजार से दो लाख रुपये तक का जुर्माना शामिल है।
लिए पेरिस क्लब के साथ मिलकर G20 द्वारा समर्थित एक
पहल है। जीआई टै ग का महत्व

• इसका उद्देश्य कम आय वाले देशों को अस्थिर ऋण का • एक बार किसी उत्पाद को यह टै ग मिल जाने के बाद, कोई अन्य
समर्थन करना है। उस नाम के समान समान वस्तु को नहीं बेच सकता है।

• सुरक्षा प्रदान करता है और उत्पाद को दूसरों द्वारा अनधिकृत


समाचार स्रोत: इं डियन एक्सप्रेस उपयोग से रोकता है।

• निर्यात को बढ़ावा देने में मदद करता है।

• GI टै ग उत्पाद
भौगोलिक संकेत (जीआई) स्थिति

खबरों में क्यों?

केरल के पांच कृषि उत्पादों को भौगोलिक संकेत (जीआई) का दर्जा


दिया गया है।

प्रमुख बिं दु

के बारे में

जीआई टै ग उत्पाद
• एक जीआई मुख्य रूप से एक कृषि, प्राकृतिक, या निर्मित
उत्पाद (हस्तशिल्प और औद्योगिक सामान) है जो एक निश्चित
भौगोलिक क्षेत्र से उत्पन्न होता है।
» उदाहरण - दार्जिलिंग चाय (ऊपर बाएं से घड़ी की दिशा में) ओनाट्टु करा एलू , कोडुंगल्लू र
• अंतर्राष्ट्रीय स्तर पर: भौगोलिक संकेत औद्योगिक संपत्ति पोट्टु वेलारी, अट्टापडी अटु कोम्बु अवारा, अट्टापडी थुवारा और
के संरक्षण के लिए पेरिस कन्वेंशन के तहत बौद्धिक संपदा कंथल्लू र-वट्टावदा वेलुथुल्ली।)

Download Monthly Articulate Magazine प्रबोधन | 33


उत्पाद राज्य विशेषता/विवरण

• कंथल्लूर-वत्तावदा वेलुथुल्ली केरल • एलिसिन से भरपूर, जो माइक्रोबियल


(लहसुन) संक्रमण, ब्लड शुगर और कैंसर के खिलाफ
प्रभावी है,

• कोडुंगलूर पोट्टुवेलारी (स्नैप तरबूज) केरल • इसमें उच्च मात्रा में विटामिन सी होता है।

• ओनाटुकारा एलु (तिल) केरल • ओनाटु कारा एलू में उच्च एं टीऑक्सीडेंट
सामग्री मुक्त कणों से लड़ने में मदद करती
है, जो शरीर की कोशिकाओ ं को नष्ट कर
देते हैं।

• अट्टापदी थुवारा (लाल चना) केरल • प्रोटीन, कार्बोहाइड्रेट, फाइबर, कैल्शियम


और मैग्नीशियम से भरपूर।

• अट्टापडी अटुकोम्बु अवारा (बीन्स) केरल • इसकी उच्च एं थोसायनिन सामग्री इसके
मधुमेह विरोधी गुणों के साथ-साथ हृदय
रोगों के खिलाफ सहायक है।

• गमोचा असम • ‘गमोचा’ विभिन्न डिजाइनों और रूपांकनों


के साथ हाथ से बुना हुआ आयताकार सूती
कपड़ा होता है।

• यह असमिया संस्कृति के प्रतीक का


प्रतिनिधित्व करता है।

Q. एक वैश्वीकृत दुनिया में, बौद्धिक संपदा अधिकार महत्व


ग्रहण करते हैं और मुकदमेबाजी का एक स्रोत हैं। कॉपीराइट,
पेटेंट और ट्रेड सीक्रे ट्स जैसे शब्दों के बीच मोटे तौर पर अंतर
स्पष्ट करें। (2014)

समाचार स्रोत: द हिं दू

रोडटे प योजना

छवि क्रेडिट: डेक्कन हेराल्ड खबरों में क्यों?

निर्यातित उत्पादों पर शुल्कों और करों की छूट (Remission of Du-


PYQ
ties and Taxes on Export Products -RoDTEP) योजना का
विस्तार रसायन, फार्मास्यूटिकल्स और लौह एवं इस्पात की वस्तुओ ं
तक हो गया है

34 I प्रबोधन Download Monthly Articulate Magazine


रोडटे प योजना के बारे में:
शुल्कों का निर्यात नहीं किया जाना चाहिए।
• योजना की घोषणा 2020 में की गई थी।
• यह पहले की मर्चेंडाइज एक्सपोर्ट फ्रॉम इं डिया स्कीम (MEIS)
• इसका उद्देश्य स्थानीय करों, कोयला उपकर, मंडी कर आदि
को प्रतिस्थापित करता है।
जैसे निर्यातकों द्वारा किए गए करों और शुल्कों की प्रतिपूर्ति
करना है, जिन्हें किसी अन्य मौजूदा योजना के तहत छूट या • रोडटे प के तहत छूट आउटबाउं ड खेपों के फ्री-ऑन-बोर्ड मूल्य
वापसी नहीं मिल रही है। के 0.5% से 4.3% तक की कानूनी सलाह के अनुसार WTO
शिकायत के तहत हैं।
• यह निर्यात की शून्य रेटिंग सुनिश्चित करता है अर्थात करों और

विवरण MEIS RoDTEP

प्रोत्साहन योजना अन्य रिफंड और उक्त निर्यात करने में निर्यात उत्पाद के निर्माण में उपयोग किए गए इनपुट
उपलब्ध कमियों के अलावा माल के पर अप्रत्यक्ष करों की वापसी, जिसकी वर्तमान में
निर्यात पर अतिरिक्त प्रोत्साहन। किसी अन्य मौजूदा योजना द्वारा प्रतिपूर्ति नहीं की
जाती है।

डब्ल्यूटीओ अनुपालन विश्व व्यापार संगठन व्यापार मानदंडों विश्व व्यापार संगठन व्यापार मानदंडों के अनुरूप
के साथ गैर-अनुपालन

Download Monthly Articulate Magazine प्रबोधन | 35


प्रोत्साहन प्रतिशत निर्यात के एफओबी मूल्य का 2% से उत्पाद आधारित % - मौजूदा एमईआईएस प्रोत्साहन
5%। योजना से कम होने की उम्मीद है। [बाद में सूचित
किया जाएगा]

जारी करने का तरीका हस्तांतरणीय के रूप में जारी (हार्ड हस्तांतरणीय शुल्क क्रेडिट/इले क्ट्रॉनिक स्क्रिप के
कॉपी/डाउनलोड करने योग्य) रूप में जारी करना, जिसे इले क्ट्रॉनिक खाता बही में
रखा जाएगा।

लाभ:
• घरेलू उद्योग को समान अवसर प्रदान करके निर्यात की वैश्विक
प्रतिस्पर्धात्मकता

• रोजगार सृजन;

• विदेशी मुद्रा भंडार और निवेश को प्रोत्साहन मिलेगा।

समाचार स्रोत: पीआईबी

राइट टू रिपेयर पोर्टल

खबरों में क्यों?

खाद्य और उपभोक्ता मामलों के मंत्री ने राइट टू रिपेयर पोर्टल सहित


राइट टू रिपेयर की आवश्यकता:
कई नई पहलें शुरू कीं।
• नए मॉडल जारी होते ही इले क्ट्रॉनिक गैजेट अक्सर जल्दी से
पुराने हो जाते हैं।
राइट टू रिपेयर क्या है? • नतीजतन, समस्याएँ सामने आने लगती हैं - उपकरणों का
• निर्माता स्वतंत्र मरम्मत को रोकने के लिए बाधाओ ं का धीमा होना, खराब पुर्जे आदि।
उपयोग करते हैं, जैसे उपकरणों और घटकों तक पहुंच को • अक्सर, मरम्मत दुर्गम या अत्यधिक महंगा मामला होता है।
सीमित करना, या सॉफ़्टवेयर लॉक का उपयोग करना।
• गैजेट्स के लगातार निपटान के कारण बड़ी मात्रा में ई-कचरा
• मरम्मत का अधिकार कानून उपभोक्ताओ ं को केवल पैदा होता है जो पर्यावरण को प्रदूषित करता है।
निर्माता की सेवाओ ं का उपयोग करने के लिए प्रतिबंधित
• यह भारत के सर्कु लर अर्थव्यवस्था लक्ष्य के लिए एक कुंजी के
करने के बजाय अपने उत्पादों को ठीक करने और संशोधित
रूप में कार्य करेगा।
करने की अनुमति देता है।

• इसे अमेरिका, ब्रिटे न और यूरोपीय संघ सहित दुनिया भर के कार्यान्वयन में चुनौतियां:
कई देशों में मान्यता दी गई है। • संभावित आईपीआर उल्लं घन: तीसरे पक्ष की मरम्मत सेवाओ ं
के लिए बौद्धिक संपदा को खोलकर रिवर्स इं जीनियरिंग।

• उपकरण सुरक्षा: अप्रशिक्षित मरम्मत करने वाला व्यक्ति


पोर्ट ल किस लिए है? उपकरणों और उपयोगकर्ताओ ं की सुरक्षा को प्रभावित कर
• निर्माता उत्पाद विवरण के मैनुअल को ग्राहकों के साथ साझा सकता है।
करेंगे, जिससे वे मूल निर्माताओ ं पर निर्भर रहने के बजाय स्वयं
• व्यक्तिगत डेटा सुरक्षा: ऐसी पहल उपयोगकर्ताओ ं के डेटा को
या तीसरे पक्ष द्वारा मरम्मत करने में सक्षम होंगे।
जोखिम में डाल सकती हैं।
• शुरुआत में मोबाइल, इले क्ट्रॉनिक्स, कंज्यूमर ड्यूरब
े ल्स,
• उत्पाद अखंडता: तकनीकी दिग्गज केवल अधिकृत
ऑटोमोबाइल और कृषि उपकरण शामिल हैं।
तकनीशियनों द्वारा मरम्मत की अनुमति देते हैं और अखंडता
सुनिश्चित करने और आकस्मिक दृष्टिकोण को हतोत्साहित
करने के लिए स्पेयर पार्ट्स या DIY मैनुअल प्रदान नहीं करते हैं।

आगे का रास्ता:
• स्पेयर पार्ट्स और प्रमाणित मरम्मत की दुकानों के उपयोग के
लिए कड़े गुणवत्ता आश्वासन खंड की आवश्यकता है।

• निर्माता प्रमाणित मरम्मत करने वालों/व्यवसायियों के साथ

36 I प्रबोधन Download Monthly Articulate Magazine


आईपी की सुरक्षा के लिए गैर-प्रकटीकरण समझौतों पर दुर्ल भ मृदा तत्त्व
हस्ताक्षर कर सकते हैं।
खबरों में क्यों?
• मरम्मत और नवीनीकरण के क्षेत्र में कौशल और प्रशिक्षण
हाल ही में, केंद्रीय विज्ञान और प्रौद्योगिकी राज्य मंत्री ने जानकारी दी
पेशेवरों।
कि दुर्ल भ मृदा तत्वों के लिए भारत चीन पर निर्भर नहीं है।
• उपभोक्ता संरक्षण अधिनियम में राइट टू रिपेयर को शामिल
प्रमुख बिं दु
कर सकते है।
के बारे में
• उत्पाद दायित्व लगाने की अवधि को विभिन्न उत्पाद वर्गों के
लिए विनियमित किया जा सकता है। • ‘दुर्ल भ मृदा तत्त्व (आरईई)’ आवर्त सारणी में 17 तत्वों का एक
परिवार है, जिसमें 15 लैं थेनाइड्स समूह तत्व शामिल हैं।

• भारत में उपलब्ध कुछ आरईई में शामिल हैं- लैं थेनम, सेरियम,
R2 आचार संहिता” पुनर्चक्रण उद्योग में पर्यावरणीय रूप से जिम्मेदार नियोडिमियम, प्रेजोडिमियम और समैरियम।
प्रथाओ को
ं अपनाने को बढ़ावा देने के लिए एक उपकरण है। मरम्मत
• इन खनिजों में अद्वितीय चुंबकीय, ल्यूमिनेसेंट और विद्युत
का अधिकार उस दिशा में एक महत्वपूर्ण कदम है।[यूपीएससी
रासायनिक गुण हैं।
सीएसई प्रारंभिक 2021]
• उन्हें ‘दुर्ल भ मृदा तत्त्व’ कहा जाता है क्योंकि पहले तकनीकी
रूप से उन्हें उनके ऑक्साइड रूपों से निकालना मुश्किल था।

• चीन के पास सबसे बड़ा रिजर्व (37 प्रतिशत), उसके बाद ब्राजील
भारत की विमानन सुरक्षा रैंकिंग और वियतनाम (18 प्रतिशत प्रत्येक), रूस (15 प्रतिशत) और शेष
राष्ट्र (12 प्रतिशत) हैं।
नवीनतम रैंकिंग के अनुसार, भारत की स्थिति 2018 में 102वें स्थान
से बढ़कर 48वें स्थान पर पहुंच गई है।
देश दुर्ल भ मृदा तत्त्व %
साल 2018 2022 चीन 37%
भारत की रैंक 102 48 ब्राज़ील 18%

वियतनाम 18%
रैंकिंग के बारे में: रूस 15%
• अंतर्राष्ट्रीय नागरिक उड्डयन संगठन द्वारा जारी है। अन्य राष्ट्र 12%
• सिं गापुर 187 देशों में पहले स्थान पर है।

• सूचकांक सुरक्षा निरीक्षण प्रणाली के आठ महत्वपूर्ण तत्वों को • इन तत्वों को दो रूपों में बांटा गया है:
मापता है, जिसमें प्राथमिक विमानन कानून, सुरक्षा मुद्दों का
» भारी दुर्ल भ मृदा तत्व: मांग में उच्च और कम उपलब्धता
समाधान आदि शामिल हैं।
» हल्के दुर्ल भ मृदा तत्व: प्रचुर मात्रा में उपलब्ध।

Download Monthly Articulate Magazine प्रबोधन | 37


समाचार स्रोत: द हिं दू

किफायती और मध्यम आय वाले आवास के लिए रिटर्न की संशोधित आर्थि क आंतरिक दर


विशेष खिड़की (SWAMIH) (एमईआईआरआर) मॉडल
• केंद्र सरकार ने SWAMIH निवेश कोष में लगभग 5,000 करोड़ ख़बरों में क्यों?
रुपये डाले हैं।
रेल मंत्रालय ने इस नए ‘संशोधित आर्थि क आंतरिक प्रतिफल दर’
• SWAMIH फंड का उद्देश्य स्ट्रेस्ड, ब्राउनफील्ड और RE- मॉडल के आधार पर अपने निवेश को सही ठहराते हुए नीति आयोग
RA-पंजीकृत आवासीय परियोजनाओ ं को पूरा करने के लिए को मूल्यांकन के लिए चार नए परियोजना प्रस्तावों का एक सेट
प्राथमिकता ऋण वित्तपोषण प्रदान करना है। भेजा है।
• यह एक श्रेणी II वैकल्पिक निवेश कोष (एआईएफ) है जिसे
2019 में घोषित किया गया था।

पिछली नीति की तुलना में परिवर्तन:

पहले बाद में


• रेलवे एक परियोजना को मंजूरी देगा यदि केवल न्यूनतम 12% • रेलरोड को परियोजना को मंजूरी देनी चाहिए, भले ही वह
लाभ शामिल हो। केवल 2-3% लाभ कमाती हो।

• रेलवे को “आर्थि क रूप से अव्यवहार्य” परियोजनाओ ं के लिए • दूरस्थ क्षेत्रों को जोड़ने और वायु प्रदूषण, यातायात भीड़ को कम
वित्तीय स्वीकृति प्राप्त करने के लिए संघर्ष करना पड़ा। करने आदि के लिए आवश्यक परियोजनाओ ं को प्राथमिकता
दी जा सकती है।

नए मॉडल की मुख्य विशेषताएं • दूर-दराज, पिछड़े और पहाड़ी क्षेत्रों आदि के लिए कनेक्टिविटी
• नीतिगत परिवर्तन सरकार को नई लाइनों, आमान परिवर्तन, परियोजनाओ ं में अधिक निवेश का मार्ग प्रशस्त करता है।
लाइनों के दोहरीकरण आदि की कनेक्टिविटी परियोजनाओ ं • सरकार ने “अमूर्त लाभों” को महत्व देने का निर्णय लिया है,
को उचित ठहराने के लिए आवश्यक छूट देता है, भले ही वे जिसमें सामाजिक, पर्यावरणीय और नेटवर्क प्रभाव शामिल हैं,
आवश्यक रूप से वित्तीय रिटर्न उत्पन्न न करते हों। को भी ध्यान में रखा जाना चाहिए।

38 I प्रबोधन Download Monthly Articulate Magazine


सरकार द्वारा अनुशंसित परियोजनाएं • COVID महामारी के कारण, समय सीमा दो साल बढ़ाकर मार्च
• महाराष्ट्र में 30 किमी कल्याण-मुरबाद नई लाइन। 2024 तक कर दी गई है ।

• महाराष्ट्र में 300 किमी जालना-जलगाँव नई लाइन। • इसका उद्देश्य 2024 तक सभी बेघर परिवारों और कच्चे और
जीर्ण-शीर्ण घरों में रहने वाले परिवारों को बुनियादी सुविधाओ ं
• महाराष्ट्र में 98 किमी अंकाई-औरंगाबाद का दोहरीकरण।
के साथ पक्का घर उपलब्ध कराना है।
• गुजरात में 100 किलोमीटर लं बी साबरमती-सरखेज-धोले रा
• इकाई सहायता राशि समतल राज्यों में 1.20 लाख रुपए पहाड़ी
नई लाइन।
राज्यों में 1.30 लाख रुपए है।

• मंत्रालय: ग्रामीण विकास मंत्रालय।

• लाभार्थि यों का चयन: तीन चरणों के सत्यापन के माध्यम से


प्रधानमंत्री आवास योजना ग्रामीण - सामाजिक-आर्थि क जाति जनगणना 2011, ग्राम सभा और
जियो-टै गिंग।
खबरों में क्यों?
• लागत साझा करना:
हाल ही में, केंद्र ने राज्यों को सूचित किया कि यदि वे PMAY-G
» मैदानी क्षेत्र: केंद्र और राज्य सरकारें 60:40 के अनुपात में
के तहत भूमिहीन लाभार्थि यों को भूमि उपलब्ध नहीं कराते हैं, तो
धनराशि वापस ले ली जाएगी। » उत्तर पूर्वी और पहाड़ी राज्य: उत्तर पूर्वी और पहाड़ी राज्यों के लिए
90:10।
प्रधानमंत्री आवास योजना ग्रामीण के बारे में:
• मार्च 2022 तक ग्रामीण गरीबों के लिए 2.95 करोड़ घर बनाने समाचार स्रोत: द हिं दू
के लक्ष्य के साथ यह योजना अप्रैल 2016 में शुरू की गई थी।

लागत साझा करना मैदानी क्षेत्र उत्तर पूर्वी और पहाड़ी राज्य

60:40 - केंद्र और राज्य सरकारें। 90:10 - केंद्र और राज्य सरकारें

प्रधानमंत्री गरीब कल्याण अन्न योजना (पीएमजीकेएवाई) और राष्ट्रीय खाद्य सुरक्षा अधिनियम (एनएफएसए)

Download Monthly Articulate Magazine प्रबोधन | 39


खबरों में क्यों? एनएफएसए के तहत जिम्मेदारियां:

हाल ही में केंद्र सरकार ने PMGKAY को NFSA के साथ विलय करने • केंद्र खाद्यान्न के आवंटन, परिवहन और राज्यों को एफसीआई
की घोषणा की। से खाद्यान्न की घर-घर डिलीवरी में सहायता करने के लिए
जिम्मेदार है।
• इसके अलावा एनएफएसए के तहत शुरू की गई इस नई
पहल का नाम “प्रधानमंत्री गरीब कल्याण अन्न योजना • राज्यों और केंद्रशासित प्रदेशों में पात्र परिवारों की पहचान,
(पीएमजीकेएवाई)” रखा गया है। राशन कार्ड जारी करना, खाद्यान्न वितरण करना, उचित मूल्य
की दुकान को लाइसेंस जारी करना और उनकी निगरानी
यह क्यों विलय किया गया? करना, एक प्रभावी शिकायत निवारण तंत्र स्थापित करना और
• राजकोषीय विवेक में सुधार के लिए (अर्थात सुचारू मौद्रिक टीपीडीएस को मजबूत करना।
संचालन और लं बे समय से चली आ रही राजकोषीय स्थिति को
बनाए रखने की सरकार की क्षमता) और भारतीय लोगों की स्रोत: इं डियन एक्सप्रेस
खाद्य सुरक्षा के लिए।

एनएफएसए, 2013 के बारे में:


छात्रों को सरकार की प्री-मैट्रिक छात्रवृत्ति
• लक्षित सार्वजनिक वितरण प्रणाली (टीपीडीएस) के तहत
कवरेज और पात्रता खबरों में क्यों?
• यह अधिनियम 2011 की जनगणना के अनुमानों के आधार पर सरकार ने फैसला किया है कि प्री-मैट्रिक स्कॉलरशिप अब केवल
देश की कुल जनसंख्या के लगभग दो-तिहाई हिस्से को कवरेज कक्षा 9 और 10 तक ही सीमित रहेगी।
प्रदान करता है।

• राशन कार्ड जारी करने के उद्देश्य से लाभार्थी परिवार की सबसे


बड़ी महिला (18 वर्ष या उससे अधिक) को ‘परिवार की मुखिया’
माना जाता है।

नए एनएफएसए में बदलाव:


• यह अधिनियम प्रति माह प्रति एएवाई (अंत्योदय अन्न योजना)
परिवार को 35 किलोग्राम खाद्यान्न का अधिकार देता है, जबकि
प्रति पीएचएच (प्राथमिकता वाले परिवार) व्यक्ति को प्रति माह
5 किलोग्राम खाद्यान्न मुफ्त में मिलता है जो पहले कम कीमत
पर उपलब्ध था।

उद्दे श्य:
• लोगों को वहनीय मूल्य पर पर्याप्त गुणवत्तापूर्ण भोजन सुनिश्चित
करने के साथ-साथ मानव जीवन चक्र दृष्टिकोण लागू करके संबंधित समाचार
पोषण और खाद्य सुरक्षा प्रदान करना।
• सामाजिक न्याय और अधिकारिता पर स्थायी समिति ने अधिक
अनुसूचित जाति और अन्य पिछड़ा वर्ग के छात्रों को पोस्ट-
कवरेज:
मैट्रिक और अन्य छात्रवृत्ति का लाभ उठाने की अनुमति देने के
• 75% ग्रामीण और 50% शहरी आबादी (कुल आबादी का 2/3)
लिए आय सीमा बढ़ाने की सिफारिश की है।
को लक्षित सार्वजनिक वितरण प्रणाली (टीपीडीएस) के तहत
सब्सिडी वाला खाद्यान्न मिलेगा। • वर्तमान 2.5 लाख रुपये की वार्षि क आय सीमा कई जरूरतमंद
छात्रों को छात्रवृत्ति प्राप्त करने से रोकती है।
योजना के तहत पात्रता मानदंड:
अनुसूचित जाति / अन्य पिछड़ा वर्ग के छात्रों के लिए उल्ले खनीय
• 15000 रुपये तक की वार्षि क आय प्राप्त करने वाले परिवार
योजनाएँ
अंत्योदय अन्न योजना के लिए पात्र हैं जो एनएफएसए के तहत
स्वचालित रूप से पात्र परिवार हैं। • यंग अचीवर्स स्कीम (श्रेयस) के लिए उच्च शिक्षा के लिए
छात्रवृत्ति
• वे परिवार जो एएवाई के अंतर्गत नहीं आते हैं, पीएचएच के
अंतर्गत आते हैं, जैसा कि राज्य सरकारों द्वारा पहचाना जाता है। • एससी और ओबीसी के लिए मुफ्त कोचिंग
वे भी पात्र हैं। • अनुसूचित जाति के लिए शीर्ष श्रेणी की शिक्षा
• एनएसएस घरेलू खपत सर्वेक्षण डेटा 2011-12 केंद्रीय और राज्य • अनुसूचित जाति के लिए राष्ट्रीय विदेशी छात्रवृत्ति
स्तर पर लाभार्थि यों की पहचान के लिए उपयोग किया जाता है।

40 I प्रबोधन Download Monthly Articulate Magazine


Enroll for FREE
42 I प्रबोधन Download Monthly Articulate Magazine
अंतर्राष्ट्रीय संगठन प्रमुख बिं दु:
भारत ने यूएनएससी की अध्यक्षता ग्रहण की संयुक्त राष्ट्र सुरक्षा परिषद (UNSC) के बारे में:
• यह 1945 में संयुक्त राष्ट्र चार्ट र द्वारा स्थापित किया गया था। यह
खबरों में क्यों?
संयुक्त राष्ट्र के छह प्रमुख अंगों में से एक है।
भारत ने 1 दिसंबर को संयुक्त राष्ट्र सुरक्षा परिषद (यूएनएससी) की
• मुख्यालय: न्यूयॉर्क ।
मासिक (दिसंबर) चक्रीय अध्यक्षता ग्रहण की।

• सदस्य: » क्या कार्रवाई की जानी चाहिए इसकी सिफारिश करने के लिए।


» परिषद में 15 सदस्य होते हैं: पांच स्थायी सदस्य और दस गैर- • परिषद अध्यक्ष के रूप में भारत की प्राथमिकताएं :
स्थायी सदस्य दो साल के लिए चुने जाते हैं। » भारत की प्राथमिकताओ ं में सीरिया, लीबिया, मध्य पूर्व,
» पांच स्थायी सदस्य संयुक्त राज्य अमेरिका, रूसी संघ, फ्रांस, चीन कोलं बिया, दक्षिण सूडान और कांगो सहित अन्य देशों में वैश्विक
और यूनाइटेड किंगडम हैं। विकास पर ब्रीफिंग शामिल है।

» हर साल, महासभा दो साल के कार्यकाल के लिए पांच गैर- » संयुक्त राष्ट्र सुधारों को बढ़ावा दें
स्थायी सदस्यों (कुल दस में से) का चुनाव करती है। » आतंकवाद प्रतिरोध
» दस गैर-स्थायी सीटों को क्षेत्रीय आधार पर वितरित किया जाता » भारत अंतर्राष्ट्रीय विश्व व्यवस्था में, विशेष रूप से संयुक्त राष्ट्र में
है। सुधारों के लिए दबाव डालता रहा है।
» परिषद की अध्यक्षता हर महीने अपने 15 सदस्यों के बीच » भारत के मार्गदर्शक सिद्धांत प्रधान मंत्री द्वारा व्यक्त किए गए
बदलती है। पांच बिं दु ‘एस’ होंगे, अर्थात् सम्मान ,संवाद ,सहयोग ,समृद्धि
• वोट: सुरक्षा परिषद के प्रत्येक सदस्य के पास एक वोट होता है। और शांति ।
» पांच स्थायी सदस्यों में से एक का “नहीं” वोट संकल्प के पारित
समाचार स्रोत: द हिं दू
होने को रोकता है।

» संयुक्त राष्ट्र का कोई भी सदस्य जो सुरक्षा परिषद का सदस्य


नहीं है, बिना मतदान के चर्चा में भाग ले सकता है।
महिलाओ ं की स्थिति पर आयोग (CSW)
• यूएनएससी की भूमिकाएं और शक्तियां:
» “संयुक्त राष्ट्र के सिद्धांतों और उद्देश्यों के अनुसार अंतर्राष्ट्रीय खबरों में क्यों?
शांति” बनाए रखना,
अमेरिकी अभियान के बाद ईरान को संयुक्त राष्ट्र महिला निकाय
» शांति या आक्रामकता के कार्य के लिए खतरे के अस्तित्व का ECOSOC से बाहर कर दिया गया था।
निर्धारण करने के लिए और

Download Monthly Articulate Magazine प्रबोधन | 43


प्रमुख बिं दु:

प्रमुख बिं दु:


महिलाओ ं की स्थिति पर आयोग (CSW) के बारे में: • हालांकि राष्ट्र 2008 से स्वतंत्र रहा है, यूरोपीय संघ में शामिल
• महिलाओ ं की स्थिति पर आयोग (CSW) मुख्य वैश्विक होना वर्तमान में पहुंच से बाहर है।
अंतरसरकारी निकाय है जो विशेष रूप से लैं गिक समानता और • मुख्य बाधा यह है कि यूरोपीय संघ के पांच सदस्य - स्पेन,
महिलाओ ं के सशक्तिकरण को बढ़ावा देने के लिए समर्पित है। रोमानिया, स्लोवाकिया, ग्रीस और साइप्रस - कोसोवो की
• आर्थि क और सामाजिक परिषद (ईसीओएसओसी) का एक स्वतंत्रता को मान्यता नहीं देते हैं।
कार्यात्मक आयोग। • यह राष्ट्र, जो अब लगभग पूरी तरह से अल्बानियाई लोगों द्वारा
• स्थापना: जून 1946 के ECOSOC संकल्प द्वारा। आबाद है, पहले यूगोस्लाविया या सर्बि या का एक हिस्सा था।

• सदस्य: 45 सदस्य राज्यों में से प्रत्येक का एक प्रतिनिधि होता


है।

• अवधि: सदस्यों को चार साल की अवधि के लिए चुना जाता है। ग्रुप ऑफ फ्रेंड्स

ईरान को हटाया जाना: खबरों में क्यों?

• आयोग में समान भौगोलिक वितरण के आधार पर आर्थि क और शांति सैनिकों के खिलाफ अपराधों के लिए जवाबदेही को बढ़ावा देने
सामाजिक परिषद द्वारा 54-सदस्यीय संयुक्त राष्ट्र आर्थि क और के लिए भारत ने ग्रुप ऑफ फ्रेंड्स को लॉन्च किया है।
सामाजिक परिषद (ECOSOC) ने 2022-2026 की शेष अवधि प्रमुख बिं दु
के लिए महिलाओ ं की स्थिति पर आयोग से ईरान को हटाने के
के बारे में
लिए एक अमेरिकी-मसौदा प्रस्ताव को अपनाया।
ग्रुप ऑफ फ्रेंड्स’
• हटाने का कारण: ईरान की नीतियां महिलाओ ं और लड़कियों
के अधिकारों के विपरीत थीं। • भारत, बांग्लादेश, मिस्र, फ्रांस, मोरक्को और नेपाल सह-अध्यक्ष
हैं।
• भारत की स्थिति: महिलाओ ं की स्थिति पर आयोग से ईरान
को बाहर करने के लिए संयुक्त राष्ट्र आर्थि क और सामाजिक • यह संयुक्त राष्ट्र के शांति सैनिकों के खिलाफ हिं सा के सभी
परिषद के प्रस्ताव से भारत दूर रहा है। कृत्यों के लिए जवाबदेही को बढ़ावा देने के लिए एक मंच के रूप
में कार्य करेगा, मेजबान राज्य को समर्थन देकर, जागरूकता
पैदा करके और संसाधन जुटाकर।

कोसोवो ने यूरोपीय संघ में शामिल होने के लिए आवेदन किया है संयुक्त राष्ट्र शांति मिशन
• यू.एन. पीसकीपिंग मिशन मेजबान देशों को संघर्ष की स्थितियों
खबरों में क्यों?
से शांति की स्थिति में परिवर्तन करने में सहायता करता है।
हाल ही में कोसोवो ने यूरोपीय संघ में शामिल होने के लिए आवेदन » उदाहरण के लिए: हैती में संयुक्त राष्ट्र स्थिरीकरण मिशन (MI-
किया है। NUSTAH) संयुक्त राष्ट्र द्वारा 2004 में राष्ट्रपति जीन-बर्ट्रें ड
एरिस्टाइड के निष्कासन के बाद हैती को स्थिर करने में मदद

44 I प्रबोधन Download Monthly Articulate Magazine


करने के लिए स्थापित एक शांति मिशन था। भारत की स्थिति:
• संयुक्त राष्ट्र के शांति सैनिक संघर्षग्रस्त देशों को सुरक्षा और • भारत ने इस क्षेत्र में चीन की हालिया चालों को संदेह की दृष्टि
राजनीतिक और शांति निर्माण सहायता प्रदान करते हैं। से देखा है जिसमें चीनी सैन्य ट्रैकिंग पोत, युआन वांग 5 की
श्रीलं का की हालिया यात्रा शामिल है।
• संयुक्त राष्ट्र शांति सैनिकों का अंतर्राष्ट्रीय दिवस: 29 मई
• भारत हिं द महासागर रिम एसोसिएशन (IORA) को इस क्षेत्र के
संयुक्त राष्ट्र शांति मिशन में भारत का योगदान: लिए पहले से ही स्थापित मंच के रूप में देखता है।
• 1948: 200,000 से अधिक भारतीयों ने 1948 से 49 संयुक्त राष्ट्र
शांति मिशनों में सेवा की है। वर्तमान में, 5,581 भारतीय विभिन्न समाचार स्रोत: द हिं दू
संयुक्त राष्ट्र शांति मिशनों का हिस्सा हैं।

• 1950: संयुक्त राष्ट्र शांति स्थापना में भारत का योगदान 1950


के दशक में कोरिया में संयुक्त राष्ट्र के संचालन में अपनी
अंतर्राष्ट्रीय शिखर सम्मेलन,कॉन्फ़्रेन्स और घटनायें:
भागीदारी के साथ शुरू हुआ।
G20 वित्त ट्रैक
• 2007: भारत यू.एन. शांति मिशन के लिए पूरी तरह से महिलाओ ं
की टु कड़ी को तैनात करने वाला पहला देश बना। खबरों में क्यों?
• भारतीय पशुचिकित्सक दक्षिण सूडान में संयुक्त राष्ट्र मिशन वित्त मंत्रालय और भारतीय रिजर्व बैंक ने संयुक्त रूप से भारत की
(यूएनएमआईएसएस) में कार्यरत हैं। जी20 अध्यक्षता के तहत “वित्त ट्रैक” एजेंडा पर चर्चा के साथ तीन
• यूनाइट अवेयर: भारत “स्थितिजन्य जागरूकता बढ़ाने और दिवसीय सभा की मेजबानी की।
शांति सेना को इलाके से संबंधित जानकारी प्रदान करने में प्रमुख बिं दु:
मदद करने के लिए इस मंच को विकसित कर रहा है।
G20 वित्त ट्रैक के बारे में:
संभावित प्रश्न: • G20 वित्त ट्रैक का नेतृत्व केंद्रीय बैंक के गवर्नर और G20 देशों
के वित्त मंत्री करते हैं।
Q. संयुक्त राष्ट्र शांति मिशन में भारत के योगदान पर चर्चा करें।
इसने किस हद तक भारत की वैश्विक छवि बनाने में मदद • यह आर्थि क और वित्तीय मुद्दों पर केंद्रित है।
की है? • यह अंतरराष्ट्रीय आर्थि क मुद्दों की चर्चा और नीति के समन्वय
के लिए एक उपयोगी मंच प्रदान करता है।
समाचार स्रोत: द इं डियन एक्सप्रेस
बैठक के दौरान एजेंडे में शामिल थे:
• इक्कीसवीं सदी की आम वैश्विक चुनौतियों का सामना करने
के लिए अंतरराष्ट्रीय वित्तीय संस्थानों को नया रूप देना,
चीन-हिं द महासागर क्षेत्र फोरम
• दुनिया के कर्ज से जुड़े जोखिमों का प्रबंधन।
खबरों में क्यों? • वित्तीय समावेशन और उत्पादकता लाभ को बढ़ावा देना।
हाल ही में, चीन ने दक्षिण-पश्चिमी चीनी शहर कुनमिंग में पहला • उत्पादकता लाभ
“चीन-हिं द महासागर क्षेत्र फोरम” आयोजित किया।
• जलवायु कार्रवाई के लिए धन
प्रमुख बिं दु:
• संयुक्त राष्ट्र सतत विकास लक्ष्यों और अंतरराष्ट्रीय कराधान
चीन-हिं द महासागर क्षेत्र फोरम के बारे में: एजेंडा को आगे बढ़ाना।
• हिं द महासागर क्षेत्र में चीन के हित को बढ़ावा देने के लिए चीन
और देशों द्वारा संयुक्त रूप से आयोजित पहला उच्च स्तरीय समाचार स्त्रोत: द इं डियन एक्सप्रेस
आधिकारिक विकास सहयोग मंच है।

• द्वारा आयोजित: चीन अंतर्राष्ट्रीय विकास सहयोग एजेंसी (CI-


DCA) चीन-जीसीसी शिखर सम्मेलन
• सदस्य: हिं द महासागर की सीमा से लगे 19 देश।
खबरों में क्यों?
» इं डोनेशिया, पाकिस्तान, म्यांमार, श्रीलं का, बांग्लादेश, मालदीव,
• हाल ही में, पहला चीन-खाड़ी सहयोग परिषद शिखर सम्मेलन
नेपाल, अफगानिस्तान, ईरान, ओमान, दक्षिण अफ्रीका, केन्या,
रियाद में आयोजित किया गया।
मोजाम्बिक, तंजानिया, सेशेल्स, मेडागास्कर, मॉरीशस, जिबूती
और ऑस्ट्रेलिया। प्रमुख बिं दु:
• इसने हिं द महासागर क्षेत्र में चीन और देशों के बीच एक समुद्री • यह चीन और रूस के बीच संबंधों का एक नया ऐतिहासिक
आपदा रोकथाम और शमन सहयोग तंत्र स्थापित करने का चरण है।
प्रस्ताव रखा।
• सऊदी ने क्लाउड कंप्यूटिंग और सऊदी शहरों में हाई-टे क

Download Monthly Articulate Magazine प्रबोधन | 45


कॉम्प्लेक्स के निर्माण पर हुआवेई के साथ एक समझौता • सचिवालय: वियना, ऑस्ट्रिया
ज्ञापन पर हस्ताक्षर किए।
• लक्ष्य: आतंकवादी संगठनों के लिए संवेदनशील तकनीकों तक
• शिखर सम्मेलन ने चीन और जीसीसी देशों के बीच रणनीतिक पहुंच को प्रतिबंधित करना।
साझेदारी को स्थापित करने और मजबूत करने का फैसला
• 1996 में स्थापित और इसके 42 सदस्य हैं।
किया।
• वासेनार अरेंजमेंट का प्ले नरी मुख्य निर्णय ले ने वाला निकाय
जीसीसी के बारे में: है जो आम सहमति पर काम करता है।
• 1981 में स्थापित, जीसीसी के छह सदस्य देश हैं: बहरीन, कुवैत, • चीन, जिसने 48 देशों के एनएसजी में भारत के प्रवेश को
ओमान, कतर, सऊदी अरब और संयुक्त अरब अमीरात (यूएई)। अवरुद्ध किया, वासेनार व्यवस्था का सदस्य नहीं है

भारत के लिए महत्व:


• भारत 2017 में WA में इसके 42वें सदस्य के रूप में शामिल हुआ।

• वासेनार व्यवस्था की सदस्यता ने पश्चिमी देशों से संवेदनशील


वस्तुओ ं और प्रौद्योगिकियों तक पहुंच प्राप्त करने के भारत के
प्रयासों को बढ़ाया।

संभावित प्रश्न:

Q. भारत के लिए वासेनार अरेंजमेंट (डब्ल्यूए) में शामिल होने


का क्या महत्व है?

समाचार स्रोत: हिं दुस्तान टाइम्स

राष्ट्रीय सुरक्षा सलाहकार (एनएसए) की बैठक

Source: Britannica खबरों में क्यों?


• भारत ने पांच मध्य एशियाई देशों - कजाकिस्तान, किर्गि स्तान,
ताजिकिस्तान, उज्बेकिस्तान और तुर्क मेनिस्तान के एनएसए
की बैठक की मेजबानी की।
वासेनार व्यवस्था

एनएसए मीट की मुख्य विशेषताएं :


खबरों में क्यों?
• कनेक्टिविटी और आतंक के वित्तपोषण का मुकाबला करना।
भारत 1 जनवरी 2023 को एक वर्ष की अवधि के लिए वासेनार
अरेंजमेंट (WA) की अध्यक्षता ग्रहण करेगा। • अफगानिस्तान की स्थिति पर ध्यान देना।

प्रमुख बिं दु मध्य एशिया में भारत के लिए चुनौती:


के बारे में • भारत किसी भी मध्य एशियाई राज्य के साथ एक सामान्य भूमि
वासेनार अरेंजमेंट (WA) सीमा साझा नहीं करता है, संबंधों को बढ़ावा देने और विस्तार
करने में एक बड़ी बाधा रही है।
• वासेनार अरेंजमेंट (डब्ल्यूए) एक बहुपक्षीय प्रौद्योगिकी नियंत्रण
समझौता है। • बेल्ट एं ड रोड इनिशिएटिव के रूप में मध्य एशिया में चीन का
शामिल होना इस क्षेत्र में भारत के प्रभाव को कम करता है।
• इसके तहत सदस्यों के बीच पारंपरिक हथियारों और दोहरे
उपयोग वाली वस्तुओ ं और प्रौद्योगिकियों के हस्तांतरण पर • धार्मि क अतिवाद और आतंकवाद।
नियमित सूचना का आदान-प्रदान होता है।
भारत के लिए मध्य एशिया का महत्व
» दोहरे उपयोग का तात्पर्य किसी वस्तु या तकनीक की क्षमता से
है जिसका उपयोग कई उद्देश्यों के लिए किया जाता है - आमतौर • भारतीय उद्योग के लिए बाजार।
पर शांतिपूर्ण और सैन्य। • रूस और मध्य पूर्व के समृद्ध संसाधनों के लिए थलचर मार्ग।
» उदाहरण के लिए: उपग्रहों का उपयोग विभिन्न प्रकार के • अपेक्षाकृत कम दूरी पर महत्वपूर्ण ऊर्जा आपूर्ति ।
नागरिक उद्देश्यों के लिए किया जा सकता है, जैसे कि दूरसंचार, » उदाहरण के लिए: कजाकिस्तान और तुर्क मेनिस्तान इस क्षेत्र
जीपीएस नेविगेशन और मौसम की भविष्यवाणी, लेकिन के दो सबसे बड़े तेल उत्पादक देश हैं।
उनका उपयोग सैन्य उद्देश्यों के लिए भी किया जा सकता है, जैसे
• खनिजों और जलविद्युत संसाधनों में समृद्ध।
कि खुफिया जानकारी एकत्र करना और मिसाइल मार्गदर्शन।

46 I प्रबोधन Download Monthly Articulate Magazine


• भारत मध्य एशियाई देशों को “एशिया का दिल” मानता है और वे
शंघाई सहयोग संगठन (एससीओ) के सदस्य भी हैं। क्या है ‘जीरो-कोविड’ रणनीति?
• यह एक ऐसी रणनीति है जिसका उद्देश्य सख्त लॉकडाउन
• मध्य एशिया रणनीतिक रूप से यूरोप और एशिया के बीच स्थित
लगाकर, सीमाओ ं को बंद करके और यात्रा प्रतिबंध लगाकर
है और व्यापार, निवेश और विकास के लिए व्यापक संभावनाएं
कोविड-19 मामलों की संख्या को कम करना है।
प्रदान करता है।

• भारत मध्य एशियाई क्षेत्र को भारत के “विस्तारित पड़ोस” का अतिरिक्त जानकारी:


हिस्सा मानता है।
‘अम्ब्रेला मूवमेंट’ एक राजनीतिक आंदोलन था जो 2014 के
• भारत ने 2012 की ‘कनेक्ट सेंट्रल एशिया (सीएए) नीति’ शुरू हांगकांग लोकतंत्र विरोध के दौरान उभरा था।
की, जो राजनीतिक, आर्थि क, सुरक्षा और सांस्कृतिक संबंधों को
शामिल करने वाला एक व्यापक आधार वाला ढांचा है। समाचार स्रोत: द इं डियन एक्सप्रेस

संभावित प्रश्न:

Q. मध्य एशिया में भारत के भू-राजनीतिक हितों की चर्चा ईगल अधिनियम


कीजिए।
खबरों में क्यों?
समाचार स्रोत: द हिं दू
ग्रीन कार्ड जारी करने पर प्रति-देश कोटा को समाप्त करने के लिए
हाल ही में संयुक्त राज्य अमेरिका (यूएस) कांग्रेस में कानूनी रोजगार
के लिए ग्रीन कार्ड के लिए समान पहुंच (ईएजीएलई) अधिनियम
श्वेत पत्र क्रांति 2022 पेश किया गया था।

ईगल अधिनियम के बारे में:


खबरों में क्यों?
• ईगल अधिनियम को “कानूनी रोजगार अधिनियम के लिए
चीन में हाल के हफ्तों में देश की कठिन शून्य-कोविड नीति के ग्रीन कार्ड तक समान पहुंच” कहा जाता है।
खिलाफ विरोध हो रहा है।
• यह ‘जन्मस्थान’ पर ‘योग्यता’ के आधार पर लोगों की भर्ती पर
प्रमुख बिं दु: केंद्रित है।
• आंदोलन को व्यापक रूप से ‘श्वेत पत्र क्रांति’ करार दिया जा रहा
• महत्व: इससे भारतीयों को लाभ होने की संभावना है।
है।

• ऐसा इसलिए है क्योंकि इन विरोध प्रदर्शनों के दौरान कई समाचार स्रोत: द इं डियन एक्सप्रेस
प्रदर्शनकारियों को सफेद A4 आकार के कागज की खाली शीट
पकड़े हुए देखा गया है।

रूसी तेल पर मूल्य सीमा

खबरों में क्यों?

सात राष्ट्रों का समूह रूसी कच्चे तेल के प्रति बैरल 60 डॉलर की


कीमत सीमा पर सहमत हुआ।

Download Monthly Articulate Magazine प्रबोधन | 47


प्रमुख बिं दु: इसे कैसे लागू किया जाएगा?
• गठबंधन में शामिल होने वाले देश तब तक रूसी तेल नहीं
खरीदेंगे, जब तक कि कीमत उस सीमा तक कम नहीं हो जाती,
जहां सीमा निर्धारित है।

• उन देशों के लिए जो गठबंधन में शामिल नहीं होते हैं, उदाहरण


के लिए, बीमा और मुद्रा भुगतान सहित गठबंधन देशों द्वारा
प्रदान की जाने वाली सभी सेवाओ ं तक पहुंच खो देंगे।

मूल्य सीमा योजना का प्रभाव:


• मूल्य कैप का उद्देश्य रूस के वित्त को नुकसान पहुँचाना है, साथ
ही साथ तेल की कीमतों में अचानक, बड़ी वृद्धि को रोकना है
यदि रूस का तेल अचानक वैश्विक बाजार में उपलब्ध नहीं है।

• बीमा कंपनियाँ और अन्य फर्में जिन्हें तेल भेजने की आवश्यकता


होती है, वे केवल रूसी कच्चे तेल के साथ ऐसा करने में सक्षम
होंगी यदि इसकी कीमत कैप पर या उससे कम हो।

रूस प्रतिक्रिया:
प्राइस कैप योजना के बारे में: • रूस “कुछ भी” आपूर्ति नहीं करेगा यदि यह रूसी हितों के विपरीत
• यह आर्थि क विनियमन का एक रूप है जो कीमतों पर एक है।
ऊपरी सीमा स्थापित करता है जो एक उपयोगिता प्रदाता चार्ज • यह मूल्य कैप पर जोर देने वाले देशों को निर्यात पर प्रतिबंध
कर सकता है। लगाने पर विचार कर रहा है।
• मूल्य सीमा योजना रूस के खिलाफ पश्चिमी देशों द्वारा लगाए
क्या भारत प्राइस कैप का पालन करेगा?
गए प्रतिबंधों में नवीनतम है।
• अब तक, भारत बाध्य नहीं है, और इस बात के बहुत कम संकेत
• रूसी कच्चे तेल की कीमत अंतरराष्ट्रीय ब्रेंट बेंचमार्क के लिए
हैं कि योजना में शामिल होने की संभावना है।
छूट पर है ,और जी 7 रूसी तेल राजस्व को कम रखने के लिए
उस प्रसार को व्यापक रखना चाहता है। • युद्ध शुरू होने के बाद से रूस से भारत का तेल सेवन 50 गुना
बढ़ गया है।

48 I प्रबोधन Download Monthly Articulate Magazine


रक्षा
जी7 के बारे में:
भारत और चीन क्षेत्रीय विवाद
• यह 1975 में छह के समूह के रूप में शुरू हुआ - तेल संकट
और वैश्विक मुद्रा विनिमय मुद्दों के परिणामस्वरूप अमेरिका, खबरों में क्यों?
ब्रिटे न, पश्चिम जर्मनी, इटली, फ्रांस और जापान के नेताओ ं
• हाल ही में, अरुणाचल प्रदेश के तवांग सेक्टर में यांग्त्ज़ी नदी के
की बैठक के साथ।
किनारे भारत और चीन के सैनिक आपस में भिड़ गए थे।
• कनाडा 1976 में शामिल हुआ- इसे G-7 बना दिया। यूरोपीय
• दोनों सदनों में सीमा की स्थिति पर चर्चा से इनकार किए जाने
संघ के नेतृत्व को जी-7 की बैठकों में हमेशा आमंत्रित किया
के बाद सांसदों ने वाकआउट किया।
जाता है।
• विपक्ष के नेता ने नियम 267 के तहत सीमा संघर्ष पर चर्चा के
• 1998 में इसे G-8 बनाने के लिए रूस को जोड़ा गया।
लिए कार्य को स्थगित करने की मांग करते हुए एक प्रस्ताव
• हालाँकि, 2008 में जॉर्जि या के साथ रूसी युद्धों और 2014 में पेश किया।
क्रीमिया के कब्जे के बाद, क्लब ने रूस को स्थायी रूप से
बाहर कर दिया।

समाचार स्रोत: द हिं दू

गश्त-ए-इरशाद

खबरों में क्यों?


• ईरान ने कथित तौर पर अपनी गश्त-ए-इरशाद पुलिस को भंग
कर दिया।

छवि स्रोत: इं डियन एक्सप्रेस

पृष्ठभूमि
• तवांग सेक्टर में वास्तविक नियंत्रण रेखा (एलएसी) में ऐसे कई
स्थल हैं जहां भारत और चीन सीमा रेखा पर सहमत नहीं हैं।
छवि: इं डियन एक्सप्रेस • तवांग में तीन “सहमत स्थान” हैं जहां एलएसी पर भारतीय और
भंग करने के पीछे कारण: चीनी दृष्टिकोण एक दूसरे से अलग हैं। इनमें से एक स्थान
यांग्त्से है, जो लुं गरू चारागाह के उत्तर में स्थित है और तवांग
महसा अमिनी, एक युवा ईरानी-कुर्द महिला, नैतिकता पुलिस की
शहर से लगभग 25 किमी दूर है।
हिरासत में मृत्यु हो गई, जिसके कारण पूरे ईरान में व्यापक प्रदर्शन
हुए। • चीन तवांग और लगभग पूरे अरुणाचल राज्य को अपनी जमीन
बताता है।
प्रमुख बिं दु
• छठे दलाई लामा का जन्म तवांग में हुआ था, जो एक महत्वपूर्ण
के बारे में
तिब्बती बौद्ध तीर्थ स्थल भी है।
गश्त-ए-इरशाद
• 1959 में तिब्बत से भारत में प्रवेश करने के बाद, वर्तमान 14वें
• गश्त-ए-इरशाद (मार्गदर्शन गश्ती) विनम्रता और हिजाब की दलाई लामा ने तवांग में अभयारण्य की मांग की और जारी
संस्कृति को फैलाने के लिए एक नैतिकता पुलिस है। रखने से पहले मठ में कुछ दिन बिताए।
• यह राष्ट्रपति महमूद अहमदीनेजाद के तहत स्थापित किया गया
था। तवांग में हाल के भारत-चीन संघर्ष के पीछे क्या संदर्भ है?
• यांग्त्से घटना के कुछ दिन पहले , चीन ने दावा किया था कि
संयुक्त भारतीय-अमेरिकी सैन्य ड्रिल ऑपरेशन युद्ध अभ्यास ने
1993 और 1996 से सीमा समझौते के प्रावधानों का उल्लं घन
किया था।

Download Monthly Articulate Magazine प्रबोधन | 49


• पूर्वी क्षेत्र में चीनी सक्रियता का विश्ले षण करते समय सैन्य आईएनएस मोरमुगाओ
तनाव की पृष्ठभूमि पर विचार किया जाना चाहिए।
खबरों में क्यों?
आगे की राह:
INS मोरमुगाओ, एक P15B स्टील्थ-गाइडेड मिसाइल विध्वंसक, को
• अपने हितों को प्रभावी ढं ग से संरक्षित करने के लिए, भारत को मुंबई में भारतीय नौसेना में कमीशन किया गया था।
चीन में अपनी सीमा के करीब किसी भी नए घटनाक्रम के प्रति
सतर्क रहना चाहिए।

• इसके अतिरिक्त, इसे कर्मि यों और अन्य रसद आपूर्ति के प्रभावी


पारगमन को सुनिश्चित करने के लिए अपने चुनौतीपूर्ण
सीमावर्ती क्षेत्रों में एक मजबूत बुनियादी ढांचा विकसित करने
की आवश्यकता है।

• सीमा प्रहरियों को अपनी बातचीत जारी रखनी चाहिए, तुरत



पीछे हटना चाहिए, सुरक्षित दूरी बनाए रखनी चाहिए और तनाव
कम करना चाहिए।

• दोनों पक्षों को ऐसी कोई भी कार्रवाई करने से बचना चाहिए जो


स्थिति को खराब कर सकती है और चीन-भारत सीमा चिंताओ ं प्रमुख बिं दु
पर सभी मौजूदा समझौतों और प्रक्रियाओ ं का पालन करना
के बारे में
चाहिए।

प्रोजेक्ट 15A प्रोजेक्ट 15B

3 स्टील्थ गाइडेड-मिसाइल विध्वंसक: चार स्टील्थ गाइडेड-मिसाइल विध्वंसक:

आईएनएस कोलकाता आईएनएस विशाखापत्तनम,

आईएनएस कोच्चि आईएनएस मोरमुगाओ (2022-दिसंबर में लॉन्च),

आईएनएस चेन्नई आईएनएस इं फाल

आईएनएस सूरत

अरनाला
आईएनएस मोरमुगाओ
ख़बरों में क्यों?
• युद्धपोत का नाम गोवा के ऐतिहासिक बंदरगाह शहर मोरमुगाओ
भारतीय नौसेना ने चेन्नई में स्वदेश निर्मित जहाज “अरनाला” लॉन्च
के नाम पर रखा गया है।
किया।
• यह परमाणु, जैविक और रासायनिक युद्ध स्थितियों में लड़
सकता है।

• यह आधुनिक निगरानी रडार के अलावा सतह से सतह और


सतह से हवा में मार करने वाली मिसाइलों से लै स है जो हथियार
प्रणालियों को लक्ष्य डेटा प्रदान करता है।

• निर्माता: मझगांव डॉक शिपबिल्डर्स लिमिटेड।MDL स्थित हैं


मुंबई, महाराष्ट्र में. यह रक्षा मंत्रालय (84% शेयरहोल्डिंग) के
स्वामित्व वाली एक सरकारी कंपनी है।

• विशिष्ट विशेषता: 75% स्वदेशी रूप से निर्मित है जो सरकार के


‘आत्म निर्भर भारत’ (आत्मनिर्भर भारत) के लक्ष्य पर जोर देती
है। छवि स्त्रोत:भारतीय नौसेना

समाचार स्त्रोत: द इं डियन एक्सप्रेस बारे में:


• ARNALA जहाज ASW SWC प्रोजेक्ट के तहत बनाया गया है
जो भारतीय नौसेना के अभय वर्ग ASW जहाजों की जगह लेगा।

• महान मराठा योद्धा छत्रपति शिवाजी महाराज ने अरनाला द्वीप


को जो रणनीतिक समुद्री महत्व दिया था, उसके सम्मान में

50 I प्रबोधन Download Monthly Articulate Magazine


जहाज को अर्नाला नाम दिया गया था। सी ईगल / ओरलान 10 यूएवी

ASW SWC प्रोजेक्ट क्या है? ख़बरों में क्यों?


• एं टी-सबमरीन वारफेयर शैलो वाटर क्राफ्ट (ASW-SWC) • Orlan-10 रूसी फर्म स्पेशल टे क्नोलॉजी सेंटर एलएलसी द्वारा
कॉर्वेट, एं टी-सबमरीन वारफेयर जहाजों का एक वर्ग है, जिसे विकसित एक मध्यम-श्रेणी, बहुउद्देश्यीय मानव रहित हवाई
तटीय जल और कम तीव्रता वाले समुद्री संचालन (LIMO) में वाहन (यूएवी) है।
संचालन करने के लिए डिज़ाइन किया गया है, जिसमें उपसतह
• यह रूस द्वारा यूक्रे न पर 20,000 तोपों के गोले गिराने के लिए
निगरानी भी शामिल है।
इस्तेमाल किया जाने वाला एक भ्रामक और अपेक्षाकृत कम
• कोचीन शिपयार्ड (CSL) और गार्डन रीच शिपबिल्डर्स एं ड तकनीक वाला ड्रोन है।
इं जीनियर्स (GRSE) द्वारा निर्मित ये जहाज 80% से अधिक घरेलू
सामग्री से बने होंगे।

अग्नि 5 मिसाइल

खबरों में क्यों?

भारत ने परमाणु सक्षम बैलिस्टिक मिसाइल अग्नि वी का रात्रि


परीक्षण सफलतापूर्वक किया।

प्रमुख बिं दु

के बारे में
वैश्विक लड़ाकू हवाई कार्यक्रम (जीसीएपी)
अग्नि 5 मिसाइल
• यूके, इटली और जापान ने छठी पीढ़ी के ग्लोबल कॉम्बैट एयर
• लं बी दूरी की सतह से सतह पर मार करने वाली परमाणु सक्षम प्रोग्राम के लिए टीमिंग की घोषणा की।
बैलिस्टिक मिसाइल।
• लड़ाकू जेट विकसित करने के लिए यह एक नई साझेदारी है।
• द्वारा विकसित: डीआरडीओ
• GCAP क्षमताओ ं का एक नेटवर्क शामिल करेगा जैसे कि बिना
• अग्नि 5 फुर्तीली है कि यह एक “कनिस्टराइज्ड” मिसाइल है। चालक दल के विमान, उन्नत सेंसर, अत्याधुनिक हथियार और
» इसका मतलब है कि मिसाइल को सड़क और रेल प्ले टफार्मों नवीन डेटा सिस्टम।
से लॉन्च किया जा सकता है, जिससे इसे तेज गति से तैनात और
लॉन्च करना आसान हो जाता है।
• यह 5,000 किलोमीटर दूर स्थित लक्ष्य को सटीकता से भेद विजय दिवस
सकती है।
» महत्व: यह सीमा लगभग पूरे चीन को मिसाइल की लक्ष्य सीमा
के भीतर रखती है।
• अग्नि मिसाइलों का विकास 1980 की शुरुआत में डॉ. एपीजे
अब्दुल कलाम के तहत एकीकृत निर्देशित मिसाइल विकास
कार्यक्रम के तहत शुरू हुआ।

ब्रह्मोस मिसाइल अग्नि-5

भारत-रूस संयुक्त उद्यम डीआरडीओ द्वारा स्वदेशी रूप से


विकसित

क्रूज़ मिसाइल बैलिस्टिक मिसाइल

मिसाइल श्रेणी • यह 16 दिसंबर को 1971 के युद्ध में पाकिस्तान पर भारत की जीत


के उपलक्ष्य में मनाया जाता है जिसके कारण एक संप्रभु देश के
अग्नि I 700-800 कि.मी
रूप में बांग्लादेश का निर्माण हुआ।
अग्नि II 2000 किमी से अधिक
• युद्ध के आठ महीने बाद, अगस्त 1972 में, भारत और पाकिस्तान
अग्नि III 2,500 किमी से अधिक ने शिमला समझौता किया।
अग्नि IV 3,500 किमी से अधिक • समझौते के तहत, भारत 93,000 पाकिस्तानी युद्ध बंदियों को
रिहा करने पर सहमत हुआ।
समाचार स्रोत: हिं दुस्तान टाइम्स

Download Monthly Articulate Magazine प्रबोधन | 51


समाचार स्त्रोत: द हिं दू

रक्षा अभ्यास:

अभ्यास भागीदार स्थान

सूर्य किरण भारत और नेपाल सलझंडी (नेपाल)

युद्ध-अभ्यास भारत, अमेरिका उत्तराखंड का औली जिला

काजिं द-22 भारत और कजाकिस्तान शिलांग, मेघालय

भारत - इं डो समन्वय गश्त भारत -इं डोनेशिया बेलावन, इं डोनेशिया

Enroll Now

52 I प्रबोधन Download Monthly Articulate Magazine


Download Monthly Articulate Magazine प्रबोधन | 53
तटीय लाल रेत के टीले एक हिस्सा हैं और उत्तर चतुर्धातुक भूगर्भीय युग का प्रतिनिधित्व
करते हैं।
खबरों में क्यों?
• वे अलग-अलग भू-आकृतिक भू-आकृतियों और सुविधाओ ं के
तटीय लाल रेत के टीलों, जिन्हें लोकप्रिय रूप से ‘एर्रा मैटी डिब्बालु ’ साथ बैडलैं ड स्थलाकृति प्रदर्शित करते हैं, जिसमें नाले , रेत के
के नाम से जाना जाता है, को जलवायु परिवर्तन के प्रभाव का टीले , दबे हुए चैनल, समुद्र तट की लकीरें आदि शामिल हैं।
अध्ययन करने के लिए संरक्षित करने की आवश्यकता है, क्योंकि
• साइट का पुरातात्विक महत्व भी है क्योंकि वे अक्सर अतीत की
एरा मैटी डिब्बालू ने हिमनद और गर्म दोनों अवधियों में बना रहा है।
मानवीय गतिविधियों के साक्ष्य को संरक्षित करते हैं।
» उदाहरण के लिए, रेत के टीले उपकरण, हथियार और मिट्टी के
बर्तनों जैसी कलाकृतियों के साथ-साथ चूल्हों, दफन स्थलों
और संरचनाओ ं जैसी अन्य विशेषताओ ं को संरक्षित कर सकते
हैं।

विश्व मृदा दिवस (डब्ल्यूएसडी) 2022

खबरों में क्यों?

प्रमुख बिं दु:


प्रमुख बिं दु:
थीम: “मृदा: जहां भोजन शुरू होता है”
• 2014 में भारतीय भूवैज्ञानिक सर्वेक्षण (जीएसआई) द्वारा तटीय
लाल रेत के टीलों को भू-विरासत स्थल घोषित किया गया था। लक्ष्य: महत्वपूर्ण पर्यावरणीय मुद्दों के बारे में सार्वजनिक जागरूकता
बढ़ाना, जिससे मिट्टी का क्षरण हो सकता है, जैसे कि क्षरण, कार्बनिक
• आंध्र प्रदेश सरकार ने 2016 में इसे ‘संरक्षित स्थलों’ की श्रेणी में
पदार्थों की हानि और मिट्टी की उर्वरता में गिरावट।
सूचीबद्ध किया था।
मिट्टी के लाभ:
• स्थान: विशाखापत्तनम तट।
• पौधों को आवश्यक पोषक तत्व प्रदान करके हमारे ग्रह पर
• वितरण: तमिलनाडु में तेरी रेत, विशाखापत्तनम में एरा मैटी जीवन का समर्थन करता है।
डिब्बालू , और श्रीलं का में एक और साइट।
• जीवों के एक बड़े, विविध समुदाय का घर जो मिट्टी की संरचना
» वे भूमध्यरेखीय क्षेत्रों या समशीतोष्ण क्षेत्रों में नहीं होते हैं।
को बढ़ाता है, महत्वपूर्ण पोषक तत्वों का पुनर्चक्रण करता है,
और खरपतवारों, पौधों के कीटों और रोगों के प्रबंधन में सहायता
महत्व:
करता है।
• यह एक जीवंत वैज्ञानिक विकास स्थल है, जो जलवायु परिवर्तन
के वास्तविक समय के प्रभावों को दर्शाता है। • मृदा कार्बनिक कार्बन को संरक्षित या बढ़ाकर जलवायु
परिवर्तन को कम करता है।
• इस साइट के लाल तलछट पृथ्वी के विकास की निरंतरता का

ह्रास और उसके परि णाम

54 I प्रबोधन Download Monthly Articulate Magazine


परिणाम: बाजरा 2023 के अंतर्राष्ट्रीय वर्ष के उद्घाटन समारोह
• किसी न किसी रूप में मिट्टी का क्षरण भारत के कुल भूमि क्षेत्र का रोम में आयोजन
का लगभग 29% प्रभावित करता है।
खबरों में क्यों?
• कृषि उत्पादकता, इन-सीटू जैव विविधता संरक्षण, जल गुणवत्ता
और भूमि-निर्भर समुदायों की सामाजिक-आर्थि क भलाई के संयुक्त राष्ट्र के खाद्य और कृषि संगठन (FAO) ने रोम, इटली में
लिए खतरा है। अंतर्राष्ट्रीय बाजरा वर्ष - 2023 (IYM2023) के उद्घाटन समारोह का
आयोजन किया।
मृदा संरक्षण: प्रमुख बिं दु:
• भारत सरकार मृदा संरक्षण के लिए पांच सूत्री रणनीति लागू
के बारे में
कर रही है।
बाजरा क्या हैं ?
» मिट्टी को रसायन मुक्त बनाना,
• बाजरा को उनके उच्च पोषण मूल्य के कारण ‘सुपर ग्रेन’ कहा
» मृदा जैव विविधता को बचाना,
जाता है।
» मृदा कार्बनिक पदार्थ को बढ़ाना » उदाहरण के लिए, ज्वार, बाजरा और रागी।
» मिट्टी की नमी को बनाए रखना और मिट्टी के क्षरण को कम • बाजरा में आहार फाइबर अधिक होता है।
करना
• विटामिन, खनिज, फाइटोकेमिकल्स और आहार फाइबर
» मिट्टी के कटाव को रोकना। सहित पोषक तत्वों के उच्च घनत्व के कारण, बाजरा कुपोषण
• केंद्र सरकार ने 2015 में मृदा स्वास्थ्य कार्ड (SHC) योजना शुरू और सूक्ष्म पोषक तत्वों की कमी को दूर करने के लिए उत्कृष्ट
की। अनाज है।
» SHC का उपयोग मृदा स्वास्थ्य की वर्तमान स्थिति का आकलन • इसके अतिरिक्त, बाजरा को उगाने के लिए उच्च गुणवत्ता वाली
करने और मृदा स्वास्थ्य में परिवर्तन का निर्धारण करने के मिट्टी की आवश्यकता नहीं होती है और इसलिए यह बढ़ती
लिए किया जाता है। आबादी की जरूरतों को आसानी से पूरा कर सकता है।
• प्रधान मंत्री कृषि सिं चाई योजना, मिट्टी के कटाव को रोकने,
प्राकृतिक वनस्पतियों के पुनर्जनन, वर्षा जल संचयन और बाजरा के अंतर्राष्ट्रीय वर्ष का महत्व: 2023
भूजल तालिका के पुनर्भरण के लिए। • खाद्य सुरक्षा और पोषण में बाजरा के योगदान के बारे में
जागरूकता बढ़ाना।
• नेशनल मिशन फॉर सस्टेनेबल एग्रीकल्चर (NMSA)
योजनाएँ जैविक खेती और प्राकृतिक खेती जैसी पारंपरिक • बाजरा के सतत उत्पादन और गुणवत्ता में सुधार के लिए
स्वदेशी प्रथाओ ं को बढ़ावा देती हैं। हितधारकों को प्रेरित करें।

• खाद्य और कृषि संगठन (एफएओ) डेटा एनालिटिक्स का


समाचार स्रोत: पीआईबी
उपयोग करके पूर्वानुमान उपकरण विकसित करने के लिए
राष्ट्रीय वर्षापोषित क्षेत्र प्राधिकरण और कृषि और किसान
कल्याण मंत्रालय (एमओए एं ड एफडब्ल्यू) के साथ सहयोग
कर रहा है। ग्रीनलैं ड के उत्तरी किनारे पर पर्माफ्रॉस्ट
» यह विशेष रूप से वर्षा सिंचित क्षेत्रों में फसल विकल्पों पर सूचित
निर्णय ले ने में कमजोर किसानों की सहायता करेगा। खबरों में क्यों?
• आगे का रास्ता: उत्तरी ग्रीनलैं ड में हिम युग तलछट में डीएनए के सूक्ष्म टु कड़े पाए
» संरक्षण जुताई: इसमें कृषि तकनीकों का उपयोग करना गए।
शामिल है जो मिट्टी की गड़बड़ी को कम करता है, जैसे कि
न्यूनतम जुताई या नो-टिल विधियों का उपयोग करके फसलें
लगाना। यह कटाव को कम करने और मिट्टी की संरचना को
बनाए रखने में मदद कर सकता है।

» कवर फसलें : फलियां या घास जैसी कवर फसलें लगाने से


मिट्टी को कटाव से बचाने में मदद मिल सकती है और मिट्टी में
कार्बनिक पदार्थ जोड़कर मिट्टी की उर्वरता में सुधार हो सकता
है।

» कंपोस्टिंग: कार्बनिक पदार्थों से बनी खाद का उपयोग, जैसे


खाद्य अपशिष्ट और यार्ड अपशिष्ट, मिट्टी में पोषक तत्वों और
कार्बनिक पदार्थों को जोड़ सकते हैं और मिट्टी की संरचना में छवि स्त्रोत: द हिं दू
सुधार करने में मदद कर सकते हैं।
समाचार स्रोत: द इं डियन एक्सप्रेस

Download Monthly Articulate Magazine प्रबोधन | 55


प्रमुख बिं दु:

मुख्य निष्कर्ष:
• अनुवांशिक सामग्री कम से कम दो मिलियन वर्ष पुरानी है।

• इससे पता चला कि आज, यह एक बंजर आर्कटिक रेगिस्तान है,


लेकिन उस समय यह जानवरों की एक श्रेणी के साथ पेड़ों और
वनस्पतियों का एक घना हरियाली वाला परिदृश्य था।

• वैज्ञानिकों ने जानवरों, पौधों और सूक्ष्मजीवों के साक्ष्य की खोज


की, जिसमें बारहसिंगा, खरगोश, लेम्मिं ग, सन्टी और चिनार के
पेड़ शामिल हैं।

पर्यावरण डीएनए:

• शोधकर्ताओ ं ने मिट्टी के नमूनों से पर्यावरणीय डीएनए, जिसे


छवि स्त्रोत: www.whereig.com
ईडीएनए भी कहा जाता है, निकाला।
आइल ऑफ मैन और मैनक्स के बारे में
• यह आनुवंशिक सामग्री है जो जीव अपने परिवेश में बहाते
• यह एक स्वशासी ब्रिटिश क्राउन डिपेंडेंसी है जो यूनाइटेड किंगडम
हैं - उदाहरण के लिए, बाल, अपशिष्ट, थूक, या सड़ने वाले
का हिस्सा नहीं है लेकिन जिसके निवासी ब्रिटिश नागरिक हैं।
शवों के माध्यम से।
• यह उत्तरी इं ग्लैंड और आयरलैं ड के बीच आयरिश सागर में स्थित
समाचार स्रोत: द हिं दू है।

• नवीनतम जनगणना के आंकड़ों से पता चला है कि द्वीप पर


आइल ऑफ मैन द्वीप रहने वाले 84,069 लोगों में से लगभग 2,200 लोग भाषा
बोलने, पढ़ने या लिखने में सक्षम थे।
खबरों में क्यों?
• नई रणनीति का लक्ष्य अगले दशक में आइल ऑफ मैन पर
आइल ऑफ मैन की प्राचीन भाषा मैनक्स को पुनर्जीवित करने का मैनक्स गेलिक बोलने वालों की संख्या को 5,000 तक बढ़ाना
प्रयास किया जा रहा है। है।

56 I प्रबोधन Download Monthly Articulate Magazine


Enroll Now
58 I प्रबोधन Download Monthly Articulate Magazine
पर्यावरण संरक्षण के लिए अंतर्राष्ट्रीय संगठन और समझौते
जैविक विविधता पर सीओपी 15 सम्मेलन

खबरों में क्यों?

जैविक विविधता पर संयुक्त राष्ट्र सम्मेलन (CBD) के पक्षकारों का 15वां सम्मेलन (COP15) हाल ही में मॉन्ट्रियल, कनाडा में आयोजित
किया गया था।

सीबीडी के बारे में: • 196 देश सीबीडी के एक पक्षकार हैं।

• 1992 में रियो डी जनेरियो में पृथ्वी शिखर सम्मेलन में, जैविक • भारत भी कन्वेंशन का एक पक्षकार है। भारत ने 1994 में इसकी
विविधता पर संयुक्त राष्ट्र सम्मेलन, जिसे अनौपचारिक रूप से पुष्टि की।
जैव विविधता सम्मेलन के रूप में जाना जाता है, हस्ताक्षर के
• यह अपने हस्ताक्षरकर्ताओ ं पर कानूनी रूप से बाध्यकारी है।
लिए खोली गई एक बहुपक्षीय संधि है।
• सचिवालय: मॉन्ट्रियल, कनाडा।
• यह संयुक्त राष्ट्र पर्यावरण कार्यक्रम (UNEP) के अंतर्गत आता
है।

• कुनमिं ग-मॉन्ट्रियल ग्लोबल बायोडायवर्सिटी फ्रेमवर्क » इस ढांचे में 23 लक्ष्य हैं जिन्हें दुनिया को 2030 तक हासिल
(GBF) को अपनाया। करने की जरूरत है।

• प्रकृति के लिए धन » इसका उद्देश्य यह सुनिश्चित करना है कि प्रति वर्ष $200


बिलियन सार्वजनिक और निजी स्रोतों से संरक्षण पहलों के
लिए दिया जाए।

» अमीर देशों को 2025 तक हर साल कम से कम 20 अरब डॉलर


और 2030 तक कम से कम 30 अरब डॉलर का योगदान देना
चाहिए।

Download Monthly Articulate Magazine प्रबोधन | 59


• हानिकारक सब्सिडी » देश 2025 तक जैव विविधता को कम करने वाली सब्सिडी की
पहचान करने और फिर उन्हें खत्म करने, चरणबद्ध या सुधार
करने के लिए प्रतिबद्ध हैं।

» वे 2030 तक उन प्रोत्साहनों को प्रति वर्ष कम से कम $500


बिलियन तक कम करने और संरक्षण के लिए सकारात्मक
प्रोत्साहन बढ़ाने पर सहमत हुए।

• निगरानी और रिपोर्टिं ग प्रगति » भविष्य में प्रगति की निगरानी के लिए सभी सहमत लक्ष्यों को
प्रक्रियाओ ं द्वारा समर्थित किया जाएगा।

» जलवायु परिवर्तन पर अंकुश लगाने के संयुक्त राष्ट्र के नेतृत्व


वाले प्रयासों के तहत ग्रीनहाउस गैस उत्सर्जन के लिए उपयोग
किए जाने वाले समान प्रारूप के बाद राष्ट्रीय कार्य योजनाएँ
निर्धारित और उनकी समीक्षा की जाएगी।

समाचार स्रोत: डीटीई

30X30 गोल नुकसानों को “2030 तक शून्य के करीब” लाया जा सके।

खबरों में क्यों? प्रकृति और लोगों के लिए उच्च महत्वाकांक्षा गठबंधन (HAC)।

UNCBD के 15वें कांफ्रेंस ऑफ पार्टीज (COP15) ने लक्ष्यों को • 2021: पेरिस में वन प्लै नेट समिट में गठबंधन को आधिकारिक
दोहराया। रूप से लॉन्च किया गया।

• यह भारत सहित 100 से अधिक देशों का एक अंतर-सरकारी


समूह है।

• इसने 30X30 लक्ष्य की परिकल्पना की, जिसे हाल ही में COP-


15 में औपचारिक रूप से शामिल किया गया था।

आइची जैव विविधता लक्ष्य

खबरों में क्यों?

आइची जैव विविधता लक्ष्य मॉन्ट्रियल, कनाडा में जैविक विविधता


पर संयुक्त राष्ट्र सम्मेलन (सीबीडी) की बैठक में चर्चा का हिस्सा हैं।

प्रमुख बिं दु

के बारे में

लक्ष्य के घटक आइची लक्ष्य

• 2030 तक 30% भूमि और 30% तटीय और समुद्री क्षेत्रों की रक्षा • नागोया सम्मेलन में जैविक विविधता पर सम्मेलन (सीबीडी)
करने के लिए प्रतिबद्ध, जिसे 30-बाई-30 के रूप में जाना जाता द्वारा ‘आइची लक्ष्य’ को अपनाया गया था।
है। • यह एक अल्पकालिक योजना है जो 20 महत्वाकांक्षी अभी तक
• यह सौदा 20% के पहले के लक्ष्य से पूरे दशक में 30% खराब प्राप्त करने योग्य लक्ष्यों का एक सेट प्रदान करती है, जिन्हें
भूमि और पानी को बहाल करने की भी इच्छा रखता है। सामूहिक रूप से आइची लक्ष्य के रूप में जाना जाता है।

• दुनिया बहुत सारी प्रजातियों के साथ अक्षुण्ण परिदृश्य और • आइची लक्ष्य 20 महत्वाकांक्षी लक्ष्यों का एक सेट प्रदान करता
क्षेत्रों को नष्ट करने से रोकने का प्रयास करेगी, जिससे उन है जिसमें विभाजित किया गया है:

60 I प्रबोधन Download Monthly Articulate Magazine


सामरिक लक्ष्य विवरण

A • सरकार और समाज में जैव विविधता को मुख्यधारा में लाकर


जैव विविधता के नुकसान के अंतर्निहित कारणों का पता
लगाना।

B • जैव विविधता पर प्रत्यक्ष दबाव कम करना और टिकाऊ


उपयोग को बढ़ावा देना।

C • पारिस्थितिक तंत्र, प्रजातियों और आनुवंशिक विविधता की


रक्षा करके जैव विविधता की स्थिति में सुधार करना।

D • जैव विविधता और पारिस्थितिक तंत्र सेवाओ ं से सभी को लाभ


बढ़ाना।

E • भागीदारी योजना, ज्ञान प्रबंधन और क्षमता निर्माण के माध्यम


से कार्यान्वयन में वृद्धि करना।

समाचार स्रोत: द इं डियन एक्सप्रेस

प्रकृति के लिए गठबंधन

खबरों में क्यों? • SIDS तीन भौगोलिक क्षेत्रों में स्थित है:
» कैरेबियन, पैसिफिक, अटलांटिक, हिं द महासागर,
15वें सीओपी (Conference of parties)में कई विकासशील छोटे
भूमध्यसागरीय और दक्षिण चीन सागर (AIMS)
द्वीप राज्यों (Small Island Developing states) ने ‘प्रकृति के
लिए गठबंधन’ बनाने पर सहमति व्यक्त की है।
प्रकृति के लिए गठबंधन
प्रमुख बिं दु • इसे ग्लोबल बायोडायवर्सि टी फ्रेमवर्क (जीबीएफ) के
के बारे में कार्यान्वयन के लिए विकसित किया गया है।

छोटे द्वीप विकासशील राज्य (एसआईडीएस) • गठबंधन का नेतृत्व काबो वर्डे, समोआ और सेशेल्स कर रहे हैं।
• एसआईडीएस 38 संयुक्त राष्ट्र सदस्य राज्यों और संयुक्त राष्ट्र • लक्ष्य: एक एकीकृत मोर्चा बनाकर इन स्थानों में जैव विविधता
के 20 गैर-संयुक्त राष्ट्र सदस्य/एसोसिएट सदस्यों का एक के उद्देश्यों को लागू करने के अधिक से अधिक साधनों के रूप
अलग समूह है। में सहमत आम SIDS प्राथमिकताओ ं और जरूरतों की वकालत
• वे अद्वितीय सामाजिक, आर्थि क और पर्यावरणीय कमजोरियों करना।
का सामना करते हैं।
समाचार स्रोत: डाउन टू अर्थ

Download Monthly Articulate Magazine प्रबोधन | 61


पर्यावरण शिक्षा, जागरूकता और प्रशिक्षण (EEAT)
सहायता स्वीकृत की गई थी:
• 1983-84: पर्यावरण, वन और जलवायु परिवर्तन मंत्रालय
» नेशनल ग्रीन कॉर्प्स (एनजीसी) कार्यक्रम और
द्वारा केंद्रीय क्षेत्र की योजना शुरू की गई।
» राष्ट्रीय प्रकृति कैम्पिं ग कार्यक्रम (एनएनसीपी)।
• उद्दे श्य: पर्यावरण जागरूकता को बढ़ावा देना और पर्यावरण
संरक्षण में छात्रों की भागीदारी को जुटाना। • ईईएटी योजना को चालू वित्त वर्ष के दौरान ‘पर्यावरण शिक्षा
कार्यक्रम’ में नया रूप दिया गया है।
• इस योजना के तहत दो प्रमुख कार्यक्रमों के लिए अनुदान

जीव-जंतु
ग्रेट इं डियन बस्टर्ड

खबरों में क्यों? • खाने की आदतें: ग्रेट इं डियन बस्टर्ड सर्वाहारी होते हैं। वे कीड़े,
छोटे स्तनधारियों और छोटे सरीसृपों का शिकार करते हैं।
हाल ही में, सुप्रीम कोर्ट ने पूछा है कि क्या लु प्तप्राय पक्षी ग्रेट इं डियन
बस्टर्ड की रक्षा के लिए ‘प्रोजेक्ट जीआईबी’ शुरू किया जा सकता है। • प्राकृतिक वास:

प्रमुख बिं दु: » आवास की रूप घास के मैदान

ग्रेट इं डियन बस्टर्ड क्या है? » मुख्य रूप से राजस्थान और गुजरात में पाया जाता है

• GIB घास के मैदान की प्रमुख पक्षी प्रजाति हैं,इसलिए इन्हे » यह राजस्थान का राजकीय पक्षी है।
पारिस्थितिक तंत्र के स्वास्थ्य का बैरोमीटर माना जाता है। • सुरक्षा की स्थिति:

» प्रकृति के संरक्षण के लिए अंतर्राष्ट्रीय संघ लाल सूची: » गंभीर रूप से संकटग्रस्त

» वन्य जीवों और वनस्पतियों की लुप्तप्राय प्रजातियों में » परिशिष्ट 1


अंतर्राष्ट्रीय व्यापार पर सम्मेलन (सीआईटीईएस):
» अनुसूची 1
» वन्यजीव (संरक्षण) अधिनियम, 1972:

खतरे: ग्रेट इं डियन बस्टर्ड: संरक्षण के प्रयास


• ओवरहेड पावर ट्रांसमिशन लाइनें: वे केबलों से टकराती हैं और • 2015: केंद्र ने GIB प्रजाति पुनर्प्राप्ति कार्यक्रम शुरू किया।
मर जाती हैं » इसके तहत, डब्ल्यूआईआई और राजस्थान वन विभाग ने संयुक्त
» भारतीय वन्यजीव संस्थान (डब्ल्यूआईआई) के अनुसार, रूप से प्रजनन केंद्र स्थापित किए जहां जीआईबी के अंडे कृत्रिम
राजस्थान में हर साल 18 जीआईबी ओवरहेड बिजली लाइनों से रूप से उत्पादित किए गए थे।
टकराने के बाद मर जाते हैं।
समाचार स्रोत: द इं डियन एक्सप्रेस
» 2021: सुप्रीम कोर्ट ने आदेश दिया कि राजस्थान और गुजरात
में कोर और संभावित GIB आवासों में सभी ओवरहेड बिजली
पारेषण लाइनों को भूमिगत किया जाना चाहिए।
• घास के मैदानों की कमी नाटोवेनेटर पॉलीडोन्टस
• शिकार करना
खबरों में क्यों?

हाल ही में, वैज्ञानिकों ने मंगोलिया में एक गोज़-नेक्ड डायनासोर

62 I प्रबोधन Download Monthly Articulate Magazine


जीवाश्म की खोज की है जो संभवतः एक अर्ध-जलीय गोताखोर दो भारतीय कछुओ ं को लु प्तप्राय प्रजातियों (CITES) में अंतर्राष्ट्रीय
शिकारी था। व्यापार पर कन्वेंशन के परिशिष्ट I में जोड़ा गया था।

प्रमुख बिं दु:


• दो कछुए एक रेड क्राउं ड रुफ्ड (लाल-मुकुट वाला छत वाला)
कछुआ और लीथ्स सॉफ्टशेल (लीथ का नरम खोल वाला)
कछुआ है।

• परिशिष्ट I जिसका अर्थ है कि प्रजातियों को विलु प्त होने का


खतरा है।

प्रमुख बिं दु:


• नाटोवेनेटर पॉलीडोन्टस एक अर्ध-जलीय डायनासोर था जो
क्रिटे शस काल के दौरान लगभग 72 मिलियन वर्ष पहले मीठे
पानी के पारिस्थितिकी तंत्र में रहता था। गोबी राजस्थान में
इसके अच्छी तरह से संरक्षित अवशेषों का पता लगाया गया था।

लिसु व्रेन बैबलर रेड-क्राउं ड रूफ्ड कछुए के बारे में

खबरों में क्यों पर्यावास:

बर्डवॉचर्स ने सुदर
ू पूर्वोत्तर अरुणाचल प्रदेश में व्रेन बैबलर्स की एक • भारत, नेपाल और बांग्लादेश के मूल निवासी और व्यापक रूप
नई प्रजाति की खोज की है, जिसे उनके द्वारा लिसु व्रेन बैबलर नाम से गंगा और ब्रह्मपुत्र नदी घाटियों में पाए जाते हैं।
दिया गया है। • वर्तमान में भारत में, राष्ट्रीय चंबल नदी घड़ियाल अभयारण्य
एकमात्र भौगोलिक क्षेत्र है जहां प्रजातियां पर्याप्त संख्या में पाई
जाती हैं।

व्रेन बैबलर के बारे में:


लीथ के सॉफ्ट-शेल कछुए के बारे में
छोटे एशियाई पक्षियों की लगभग 20 प्रजातियाँ हैं जो बब्बलर
• लीथ का सॉफ्ट-शेल कछुआ भारत के लिए स्थानिक है।
परिवार टिमालीडे से संबंधित हैं। 1988 में भारत में व्रेन बैबलर के देखे
जाने की केवल एक रिपोर्ट आई है।व्रेन- बैबलर मुख्य रूप से दक्षिणी • पर्यावास: मुख्य रूप से दक्षिणी प्रायद्वीपीय भारत में नदियों और
एशिया में होते हैं। जलाशयों में निवास करता है।

चिं ताएं :
• प्रदूषण और अनियंत्रित शहरीकरण के कारण पर्यावास हानि।
रेड-क्राउन्ड रूफ्ड और लीथ्स सॉफ्टशेल कछुए
• अवैध व्यापार और
खबरों में क्यों? • अवैध शिकार
CITES COP19 में, विलु प्त होने के उच्च जोखिम का सामना कर रहे समाचार स्रोत: डाउन टू अर्थ

Download Monthly Articulate Magazine प्रबोधन | 63


प्रदूषण • यह गैसीय प्रदूषकों को हटाने का प्रयास करता है। SO2 थर्मल
प्रसंस्करण, उपचार और दहन के कारण होता है।
फ्लू गैस डिसल्फराइजेशन
भारत में FGD संयंत्र स्थापित करने की आवश्यकता
• SO2 उत्सर्जकों में भारत शीर्ष स्थान पर बना हुआ है।

• 2019 में, भारत ने वैश्विक मानवजनित (मानव निर्मित) SO2


उत्सर्जन का 21% उत्सर्जन किया।

• समयसीमा:
» भारत ने प्रारंभ में 2017 की समय सीमा निर्धारित की थी।
• इसे आगे 2025 तक के लिए बढ़ा दिया गया था।

समाचार स्रोत: द इं डियन एक्सप्रेस

खबरों में क्यों?

एक अध्ययन में पाया गया है कि पश्चिम बंगाल ने किसी भी कार्बन संग्रहण, उपयोग और भंडारण (CCUS) नीतिगत ढांचा
कोयला आधारित थर्मल पावर प्लांट में फ्लू गैस डीसल्फराइजेशन
खबरों में क्यों?
(एफडीजी) स्थापित नहीं किया है।
हाल ही में, नीति आयोग ने भारत में कार्बन संग्रहण, उपयोग और
फ्लू गैस डिसल्फराइजेशन के बारे में:
भंडारण( कार्बन कैप्चर, यूटिलाइजेशन एं ड स्टोरेज - CCUS )
• यह जीवाश्म-ईंधन बिजली संयंत्रों की निकास गैसों से सल्फर
पॉलिसी फ्रेमवर्क और इसकी तैनाती तंत्र पर एक अध्ययन रिपोर्ट
डाइऑक्साइड (SO2) को हटाने के लिए उपयोग की जाने वाली
जारी की।
तकनीकों का एक समूह है।

छवि स्रोत: सीसीपीए.सीए

64 I प्रबोधन Download Monthly Articulate Magazine


प्रमुख बिं दु: कोयला गैसीकरण, या बिजली की भट्टियों का उपयोग करता
है।
कार्बन कैप्चर यूटिलाइजेशन एं ड स्टोरेज (CCUS) के बारे में:
• इसका उद्देश्य कार्बन उत्सर्जन को या तो भंडारण या पुन: उपयोग • यह भारत को लागत में कटौती, जीएचजी उत्सर्जन को कम
करके कम करना है ताकि कब्जा कर लिया गया कार्बन करने, इस्पात निर्यात में सुधार और इस्पात की गुणवत्ता में
डाइऑक्साइड वातावरण में प्रवेश न करे। सुधार करने में मदद करेगा।

• यह हार्ड-टू -एबेट सेक्टर (लौह और इस्पात, सीमेंट, अलौह • 2019 में पूर्वी भारत में प्रधान मंत्री ऊर्जा गंगा परियोजना का
धातुओ ं और रसायनों का उत्पादन) से डीकार्बोनाइजेशन में शुभारंभ हुआ, जो “ग्रीन स्टील” की ओर एक बदलाव के हिस्से
मददगार है। के रूप में सभी आस-पास के इस्पात कारखानों को गैस की
आपूर्ति करेगा।
शामिल प्रक्रिया:
ग्रीन स्टील की आवश्यकता क्यों है ?
• बिजली उत्पादन या औद्योगिक गतिविधि, जैसे स्टील या सीमेंट
बनाने से उत्पन्न कार्बन डाइऑक्साइड को कैप्चर करना; • इस्पात उत्पादन वैश्विक उत्सर्जन का लगभग 8% है।

• परिवहन • अंतर्राष्ट्रीय ऊर्जा एजेंसी (IEA) के अनुसार, दुनिया के जलवायु


लक्ष्यों को पूरा करने के लिए स्टील से उत्सर्जन को 2050 तक
• इसे गहरे भूमिगत भंडारण।
50% तक कम किया जाना चाहिए और फिर गिरावट जारी
» कार्बन उत्सर्जन के लिए संभावित भंडारण स्थलों में खारे रखनी चाहिए।
जलभृत या कम तेल और गैस जलाशय शामिल हैं।
• भारत में जीएचजी उत्सर्जन के मामले में इस क्षेत्र की हिस्सेदारी
• सीसीयूएस का महत्व:
5 प्रतिशत है।
» कोयले की समृद्ध निधि का उपयोग करते हुए स्वच्छ उत्पादों के
उत्पादन को सक्षम बनाना

» आयात कम करना और आत्मनिर्भर भारत की ओर अग्रसर


नमामि गंगे कार्यक्रम
करना

» रोजगार सृजन खबरों में क्यों?

» कोयला गैसीकरण और नवजात हाइड्रोजन अर्थव्यवस्था जैसे संयुक्त राष्ट्र ने नमामि गंगे पहल को प्राकृतिक दुनिया को पुनर्जीवित
सूर्योदय क्षेत्रों को सक्षम करना करने के लिए शीर्ष 10 विश्व बहाली फ्लैगशिप पहलों में से एक के
रूप में मान्यता दी है।
समाचार स्रोत: द हिं दू
नमामि गंगे कार्यक्रम (NGP) के बारे में
• 2014:इसे राष्ट्रीय नदी गंगा के प्रदूषण, संरक्षण और कायाकल्प
के प्रभावी उन्मूलन के दोहरे उद्देश्यों के साथ 2014 में लॉन्च
ग्रीन स्टील किया गया था।

• यह जल शक्ति मंत्रालय के अधीन था और विश्व बैंक द्वारा


खबरों में क्यों?
समर्थित था।
सरकार सरकारी परियोजनाओ ं में ग्रीन स्टील के उपयोग को
• कार्यान्वयन: राष्ट्रीय गंगा परिषद (प्रधानमंत्री की अध्यक्षता
अनिवार्य करने पर विचार कर रही है।
में) एक निर्णय ले ने वाली संस्था है। राष्ट्रीय स्वच्छ गंगा मिशन
ग्रीन स्टील के बारे में: (एनएमसीजी), और इसके राज्य समकक्ष संगठनों यानी राज्य
• यह स्टील है जो जीवाश्म ईंधन के उपयोग के बिना तैयार किया कार्यक्रम प्रबंधन समूह (एसपीएमजी) द्वारा कार्यान्वयन किया
जाता है जाता है।

• यह निम्न-कार्बन ऊर्जा स्रोतों जैसे हाइड्रोजन (हरा या नीला), • फंडिं ग: केंद्रीय रूप से वित्त पोषित, गैर-व्यपगत कॉर्पस

Download Monthly Articulate Magazine प्रबोधन | 65


राष्ट्रीय गंगा परिषद

संगठन की प्रकृति गैर-सांविधिक; शासनादेश के तहत गठित


गठन वर्ष 2016
संघटन अध्यक्ष– भारत के प्रधान मंत्री

उपाध्यक्ष- केंद्रीय जल शक्ति मंत्री सदस्य: केंद्रीय मंत्री, राज्य के


मुख्यमंत्री, नीति आयोग के उपाध्यक्ष, केंद्रीय और राज्य विभागों के
सचिव
क्षेत्राधिकार गंगा नदी बेसिन राज्य- हिमाचल प्रदेश, उत्तराखंड, उत्तर प्रदेश,
दिल्ली-एनसीआर, हरियाणा, झारखंड, बिहार, पश्चिम बंगाल और
गंगा की प्रमुख सहायक नदियों वाले राज्य

• एनजीपी के मुख्य स्तंभ: » रिवर-फ्रंट डेवलपमेंट,

» सीवरेज उपचार अवसंरचना, » जैव विविधता,

» नदी-सतह की सफाई » जन जागरण

» वनीकरण, » गंगा ग्राम

» औद्योगिक प्रवाह निगरानी, समाचार स्रोत: पीआईबी

66 I प्रबोधन Download Monthly Articulate Magazine


MAG10

Learn on Unacademy

Download Monthly Articulate Magazine प्रबोधन | 67


68 I प्रबोधन Download Monthly Articulate Magazine
तकनीक • जैसा कि नाम से पता चलता है, फेशियल रिकॉग्निशन
टे क्नोलॉजी (FRT) किसी व्यक्ति के फोटो या वीडियो से उसका
डिजीयात्रा पहल- फेस रिकॉग्निशन टे क्नोलॉजी
चेहरा कैप्चर करके उसकी पहचान कर सकती है।

खबरों में क्यों? • यह वास्तविक समय में भी काम करता है और चेहरे की

सरकार ने फेशियल रिकग्निशन टे क्नोलॉजी का उपयोग करके पहचान करने और उन्हें मौजूदा डेटाबेस में मैप करने के लिए

हवाई यात्रा को परेशानी मुक्त बनाने के लिए चुनिं दा हवाई अड्डों पर गहरे तंत्रिका नेटवर्क का उपयोग करके उन्नत मशीन लर्निं ग

डिजीयात्रा पहल के तहत पेपरले स एं ट्री की शुरुआत की है। एल्गोरिदम पर निर्भर करता है।

प्रमुख बिं दु • इसके अलावा, यह बायोमेट्रिक मानचित्र बनाने के लिए किसी


व्यक्ति के चेहरे की विशिष्ट विशेषताओ ं की पहचान करता है, जो
के बारे में एक एल्गोरिथम संभावित व्यक्तियों से मेल खाता है।
फेस रिकॉग्निशन टे क्नोलॉजी

छवि स्त्रोत: nec

फ़ायदे:
कम कर सकता है, जिससे रीयल-टाइम निगरानी हो सकती है।
• बेहतर अपराध रोकथाम और नियंत्रण
• मानवाधिकारों के लिए जोखिम: निजता के अधिकार सहित
• आसान और सुरक्षित सीमा नियंत्रण अन्य मानवाधिकारों के लिए उत्पन्न जोखिम ,कानून और
• आतंकवाद का मुकाबला व्यवस्था में इसके किसी भी कथित लाभ से कहीं अधिक है।

• हवाई अड्डों, रेलवे स्टेशनों पर कानून प्रवर्तन • बिना सहमति के डेटा का संग्रह: यह लाखों छवियों के डेटाबेस में
खोज करता है, बिना ज्ञान या सहमति के स्क्रै प किया जाता है,
• उपभोक्ता सुविधा- जैसे तेज़ चेक-इन
और अक्सर विफल रहता है।
चिं ता:
डिजी यात्रा पहल:
• इसका उपयोग बड़े पैमाने पर निगरानी के लिए किया जा
• डिजी यात्रा पहल का उद्देश्य देश में हवाई यात्रा को कागज रहित
सकता है।
और परेशानी मुक्त बनाना है।
» उदाहरण के लिए: चीन में, सरकार ने देश में मुस्लिम
अल्पसंख्यक उइगरों को ट्रैक करने के लिए प्रौद्योगिकी का • कार्यान्वयन: इसकी परिकल्पना भारतीय विमानपत्तन
उपयोग किया है। प्राधिकरण के तहत डिजी यात्रा फाउं डेशन (26% हिस्सेदारी) द्वारा
की गई है और शेष स्वामित्व निम्नलिखित हवाई अड्डों के पास है:
• रीयल-टाइम निगरानी: यह प्रयोजनों और अभिव्यक्तियों को

Download Monthly Articulate Magazine प्रबोधन | 69


कोचीन, बैंगलोर, दिल्ली, हैदराबाद, मुंबई उपयोग को प्रोत्साहित करे और दुरुपयोग के खिलाफ आवश्यक
सुरक्षा उपायों को लागू करे।
डिजीयात्रा कैसे काम करती है?
• व्यक्तिगत डेटा की सुरक्षा के लिए, ऐसी नीतियां बनाई जानी
• ऐप डाउनलोड करें, आधार/बोर्डिं ग पास से लिं क करें, और एक
चाहिए जो FRT के उपयोग से संबंधित जोखिमों को संबोधित
सेल्फी लें जो एयरपोर्ट फेशियल तकनीक से जुड़ जाएगी।
करें।
• यह सत्यापन को तेज़, पूरी तरह से निर्बाध और परेशानी मुक्त
बनाता है। संभावित प्रश्न

Q. फेशियल रिकॉग्निशन टे क्नोलॉजी क्या है? इससे जुड़ी


आगे का रास्ता:
चिंताओ ं और लाभों पर चर्चा करें।
• सरकार को एक अधिक गहन घरेलू ढांचा तैयार करना चाहिए
जो समग्र रूप से समाज के लाभ के लिए नई तकनीक के समाचार स्रोत: द इं डियन एक्सप्रेस

पहला ड्रोन स्टेशन

खबरों में क्यों?

दवायें पहुंचाने के लिए मेघालय में पहला ड्रोन स्टेशन स्थापित किया देता है।
गया था।
उपयोग:
प्रमुख बिं दु:
• GPT का उपयोग विभिन्न प्रकार के प्राकृतिक भाषा प्रसंस्करण
• ड्रोन का नाम: सर्टि प्ले न X34, क्षमता: 4 किग्रा
कार्यों के लिए किया जा सकता है, जिसमें भाषा अनुवाद, सारांश,
• लाभ: दूरदराज के इलाकों में किसानों के लिए हवाई परिवहन। प्रश्न उत्तर आदि शामिल हैं।
किसानों को बीज उपलब्ध कराना और उनकी कृषि उपज को
बाजारों तक पहुंचाना। समाचार स्रोत: द हिं दू

IT
चैटजीपीटी चैटबॉट एं ड-टू-एं ड एन्क्रिप्शन
• OpenAI, कंपनी- ने ChatGPT (जनरेटिव प्री-ट्रेन्ड ट्रांसफ़ॉर्मर)
खबरों में क्यों?
नामक एक नया चैटबॉट पेश किया
एप्पल (Apple) ने घोषणा की कि वह एं ड-टू -एं ड एन्क्रिप्शन द्वारा
• ChatGPT एक संवादात्मक संवाद मॉडल है, एक चैटिंग रोबोट
संरक्षित डेटा बिं दुओ ं की संख्या में वृद्धि करेगा।
है, जिसे कृत्रिम बुद्धिमत्ता (AI) और मशीन लर्निं ग द्वारा प्रशिक्षित
किया गया है। प्रमुख बिं दु

• यह इं सानों की तरह प्रतिक्रिया करता है। के बारे में

• यह ट्रांसफॉर्मर नामक एक मशीन लर्निं ग तकनीक का एं ड-टू-एं ड एन्क्रिप्शन


उपयोग करता है, जो इसे बड़ी मात्रा में डेटा को प्रोसेस करने • एं ड-टू -एं ड एन्क्रिप्शन एक संचार प्रक्रिया है जो दो उपकरणों के
और उच्च-गुणवत्ता वाले टे क्स्ट को उत्पन्न करने की अनुमति बीच साझा किए जा रहे डेटा को एन्क्रिप्ट करती है।

70 I प्रबोधन Download Monthly Articulate Magazine


• यह क्लाउड सर्वि स प्रोवाइडर्स, इं टरनेट सर्वि स प्रोवाइडर्स (ISPs) • एं ड-टू -एं ड एन्क्रिप्शन की प्रक्रिया एक एल्गोरिथ्म का उपयोग
और साइबर क्रिमिनल्स जैसे थर्ड पार्टीज को डेटा ट्रांसफर होने करती है जो स्टैंडर्ड टे क्स्ट को एक अपठनीय प्रारूप में बदल
के दौरान एक्सेस करने से रोकता है। देती है।

छवि क्रेडिट: नॉर्डपास

अनुप्रयोग
• व्यावसायिक दस्तावेज़ों, वित्तीय विवरणों, कानूनी कार्यवाहियों
और व्यक्तिगत वार्तालापों को स्थानांतरित करते समय एं ड-टू -
एं ड एन्क्रिप्शन का उपयोग किया जाता है।

• इसे एक ऐसी तकनीक के रूप में भी देखा जाता है जो


उपयोगकर्ताओ ं के डेटा को सरकारी एजेंसियों द्वारा स्नूपिंग से
सुरक्षित करती है।

संभावित प्रश्न:

Q. ‘एं ड टू एं ड एन्क्रिप्शन’ क्या है? उपयुक्त उदाहरण की सहायता प्रमुख बिं दु:
से समझाइए कि यह किस प्रकार आँकड़ों की सुरक्षा करता सेमी-ऑटोमेटेड ऑफ़साइड टे क्नोलॉजी (SAOT) के बारे में:
है।
• यह वीडियो मैच अधिकारियों और ऑन-फील्ड अधिकारियों के
लिए एक सहायक उपकरण है जो उन्हें तेज, अधिक प्रतिलिपि
प्रस्तुत करने योग्य और अधिक सटीक ऑफसाइड निर्णय ले ने
में मदद करता है।
सेमी-ऑटोमेटेड ऑफसाइड टे क्नोलॉजी (एसपीएटी)-फीफा
• तकनीकी:
2022
» वीडियो असिस्टेंट रेफरी (VAR) सिस्टम का हिस्सा हैं जो मौजूदा
खबरों में क्यों? ट्रैकिंग टू ल के साथ मैच फ़ु टबॉल के अंदर एक सेंसर है।
• इन-बॉल डिवाइस सटीक स्थितीय डेटा प्रदान करता है जबकि
हाल ही में, फीफा ने नई सेमी-ऑटोमेटेड ऑफ़साइड टे क्नोलॉजी
कैमरे गेंद और खिलाड़ियों दोनों को ट्रैक करते हैं।
(एसएओटी) पेश की जो त्वरित निर्णय के लिए जिम्मेदार थी।

Download Monthly Articulate Magazine प्रबोधन | 71


कैप्टिव गैर-सार्वजनिक नेटवर्क (सीएनपीएन) • कनेक्शन प्रक्रिया के दौरान एन्क्रिप्शन की कमी भी 5G
तकनीक का उपयोग करने वाले उपकरणों को साइबर हमले
खबरों में क्यों? और डेटा चोरी के लिए एक आसान लक्ष्य बनाती है।

भारती एयरटे ल ने महाराष्ट्र में महिंद्रा एं ड महिंद्रा की चाकन सुविधा


संभावित प्रश्न:
में ‘कैप्टिव प्राइवेट नेटवर्क ’ के लिए टे क महिंद्रा के साथ साझेदारी
की है। Q. भारत में 5G रोलआउट के लिए चुनौतियों पर चर्चा करें।

प्रमुख बिं दु:


समाचार स्रोत: द इं डियन एक्सप्रेस
के बारे में

कैप्टिव गैर-सार्वजनिक नेटवर्क (CNPN)


• कैप्टिव गैर-सार्वजनिक नेटवर्क (CNPN) एक निजी 5G सामान्य नेटवर्क पहुंच विनियमन
नेटवर्क है, जो पूरी तरह से एक फर्म के अपने उपयोग के लिए
• GNA का अर्थ अंतरराज्यीय पारेषण प्रणाली के लिए “ओपन
स्थापित किया जाता है।
एक्सेस” है। यह “एक राष्ट्र, एक ग्रिड” के विचार के अनुरूप है।
• सार्वजनिक नेटवर्क के विपरीत, यह बाहरी संचार के लिए बंद है।
• ट्रांसमिशन सेवा के रूप में जीएनए ग्रिड सीमाओ ं के अधीन
• यह उच्च-आवृत्ति, निम्न-तरंगदैर्घ्य वायुतरंगों पर निर्भर करता शेड्यूलिंग के मामले में बिजली के खरीदारों और विक्रेताओ ं
है और इसलिए कारखानों, विनिर्माण संयंत्रों, अस्पतालों, के लिए अधिक लचीलापन और खुली पहुंच की क्षमता प्रदान
विश्वविद्यालयों आदि जैसे संस्थानों के लिए आदर्श है। करता है, और वर्तमान बिं दु-से-बिं दु खुली पहुंच तंत्र की कठोरता
से ग्रस्त नहीं है।
5जी तकनीक के फायदे
• तेज़ गति: 4जी और 4जी एलटीई की तुलना में 5जी मोबाइल
फोन और अन्य उपकरणों पर तेजी से काम करता है।
» उदाहरण के लिए: हाल के एक सर्वेक्षण के अनुसार, 5G का साइबर ख़तरे
उपयोग करने वाले उपभोक्ताओ ं ने डाउनलोडिंग प्रक्रिया में प्रति रैंसमवेयर
दिन लगभग 23 घंटे की बचत की।
• लो ले टेंसी: 4G की तुलना में 5G में लो ले टें सी है जो AI, IoT और खबरों में क्यों?
वर्चुअल रियलिटी जैसे नए एप्लिकेशन को कुशलता से सपोर्ट हाल ही में, अखिल भारतीय आयुर्विज्ञान संस्थान (AIIMS) में ई-सेवाएं
करेगा। एक संदिग्ध रैनसमवेयर हमले से रुक गईं।
» विलं बता देरी का एक उपाय है। एक नेटवर्क में, विलं बता उस प्रमुख बिं दु:
समय को मापती है जो कुछ डेटा को पूरे नेटवर्क में अपने गंतव्य
के बारे में
तक पहुंचने में लगता है।
रैंसमवेयर
• बढ़ी हुई क्षमता: 5G में 4G की तुलना में 100 गुना अधिक क्षमता
देने की क्षमता है। • रैंसमवेयर एक खतरनाक सॉफ़्टवेयर है जो कंप्यूटर पर
फ़ाइलों को लॉक कर देता है।
• अधिक बैंडविड्थ: 5G बैंडविड्थ को बढ़ाएगा जो डेटा को जल्द से
जल्द ट्रांसफर करने में मदद करेगा। • डिक्रिप्शन कुंजी के बदले मालिक से फिरौती की मांग की जाती
है।
• आर्थि क महत्व: भारत पर 5G का संचयी आर्थि क प्रभाव 2035
तक 450 बिलियन डॉलर तक पहुंचने की उम्मीद है। • उदाहरण के लिए
» वान्ना क्राइ (wannacry)
भारत में 5G रोलआउट के लिए चुनौतियाँ:
• इसके लिए यूनिट क्षेत्र में 4g से अधिक सिग्नल टावरों की भारत में कौन सी एजेंसियां साइबर हमलों से निपटती हैं?
आवश्यकता होती है, जिससे बिजली की खपत अधिक होती है। • सीईआरटी-इन एक राष्ट्रीय नोडल एजेंसी है जो साइबर हमलों
• 5जी तकनीक से उड़ान संचालन में बाधा आती है। पर इनपुट एकत्र, विश्ले षण और प्रसारित करती है;

• यह बैटरी को खत्म करके और उसके जीवनकाल को कम • राष्ट्रीय महत्वपूर्ण सूचना अवसंरचना की सुरक्षा के लिए
करके, सेलुलर डिवाइस को कमजोर कर देता है। राष्ट्रीय महत्वपूर्ण सूचना अवसंरचना संरक्षण केंद्र की स्थापना
की गई है।
• 5G प्रौद्योगिकियां मोबाइल फोन उपयोगकर्ताओ ं को उच्च
डाउनलोड गति सुनिश्चित करने की अनुमति देती हैं। • दुर्भावनापूर्ण सॉफ़्टवेयर प्रोग्रामों का पता लगाने के लिए साइबर
स्वच्छता केंद्र (बॉटनेट क्लीनिंग एं ड मालवेयर एनालिसिस
• उच्च स्पेक्ट्रम मूल्य निर्धारण
सेंटर) शुरू किया गया है।
• सीमित वैश्विक कवरेज
समाचार स्रोत: द हिं दू

72 I प्रबोधन Download Monthly Articulate Magazine


ब्लू बगिं ग

• ब्लू बगिंग हैकिंग का एक रूप है जो हमलावरों को किसी में चुनौतियों को ध्यान में रखते हुए, जांच करें कि भारत ने
डिवाइस को उसके ब्लू टू थ कनेक्शन के माध्यम से एक्सेस व्यापक राष्ट्रीय साइबर सुरक्षा रणनीति को किस हद तक
करने देता है। सफलतापूर्वक विकसित किया है। (2022)
• एक बार डिवाइस या फोन ब्लू -बग हो जाने के बाद, एक हैकर
कॉल सुन सकता है, पढ़ सकता है और संदेश भेज सकता है और
संपर्क चुरा सकता है और संशोधित कर सकता है।

• ब्लू बगिंग तब हो सकती है जब कोई ब्लू टू थ-सक्षम डिवाइस डीपफेक तकनीक


हैकर के 10 मीटर के दायरे में हो।
खबरों में क्यों?
PYQ चीन ने डीप सिं थेसिस (या “डीपफेक”) तकनीक को नियंत्रित करने
Q. साइबर सुरक्षा के विभिन्न तत्व कौन से हैं? साइबर सुरक्षा वाले नए उपायों का एक सेट जारी किया है।

प्रमुख बिं दु

के बारे में

डीपफेक टे क्नोलॉजी

Download Monthly Articulate Magazine प्रबोधन | 73


पहचान की चोरी, वित्तीय धोखाधड़ी, आदि।
• डीपफेक कृत्रिम छवियों और ऑडियो का एक संकलन है, जिसे
मशीन-लर्निं ग एल्गोरिदम के साथ गलत सूचना फैलाने और • डीपफेक तकनीक का उपयोग पूर्व अमेरिकी राष्ट्रपतियों बराक
वास्तविक व्यक्ति की उपस्थिति, आवाज को बदलने के लिए ओबामा और डोनाल्ड ट्रम्प जैसी उल्ले खनीय हस्तियों को
एक साथ रखा गया है। प्रतिरूपित करने के लिए किया गया है।

• यह ऐसे लोगों को पैदा कर सकता है जो अस्तित्व में नहीं हैं और


संभावित प्रश्न:
यह नकली वास्तविक लोगों को कह और कर सकता है जो
उन्होंने नहीं कहा या नहीं किया। Q. डीपफेक शब्द की व्याख्या करें। इस तकनीक से उत्पन्न
खतरों का विश्ले षण करें।
• डीपफेक तकनीक का उपयोग अब दुर्भावनापूर्ण गतिविधियों
के लिए किया जा रहा है जैसे कि घोटाले और झांसा, चुनावी समाचार स्रोत: द हिं दू
हस्तक्षेप, सोशल इं जीनियरिंग, स्वचालित गलत सूचना हमले ,

डॉक्सिंग

खबरों में क्यों? अंतरिक्ष


नई एं टी-डॉक्सिं ग नीति के उल्लं घन के लिए ट्विटर ने कई पत्रकारों जेमिनिड्स उल्का बौछार
के खाते को निलं बित कर दिया है।
खबरों में क्यों?
प्रमुख बिं दु
उल्का वर्षा, जेमिनिड्स दिसंबर के मध्य के आसपास चरम पर होगी।
के बारे में
• डॉक्सिंग (Doxxing) में संवेदनशील, निजी जानकारी को प्रमुख बिं दु
ऑनलाइन उजागर करना शामिल है। के बारे में
• यह ऑनलाइन उत्पीड़न का एक रूप है जिसका अर्थ है किसी का जेमिनिड्स उल्का बौछार
वास्तविक नाम, पता, नौकरी या अन्य पहचान करने वाले डेटा • उल्काएं धूमकेतुओ ं के टु कड़े हैं।
को सार्वजनिक रूप से उजागर करना।
• वे तेज गति से पृथ्वी के वायुमंडल में प्रवेश करते हैं और जलकर
• पीड़ित को अपमानित करने या डराने-धमकाने के उद्देश्य से ‘बौछार’ बनाते हैं।
डॉक्सिं ग पीड़ित की सहमति के बिना होता है।
• जेमिनीड्स की उत्पत्ति एक क्षुद्रग्रह, 3200 फेथॉन से हुई है।
समाचार स्रोत: द इं डियन एक्सप्रेस • जेमिनीड उल्कावृष्टि लगभग जेमिनी तारामंडल की दिशा से
उत्पन्न होती प्रतीत होती है।

74 I प्रबोधन Download Monthly Articulate Magazine


छवि क्रेडिट: Space.com और व्यक्तिगत ट्रैकर्स में क्षेत्रीय नेविगेशन प्रणाली के उपयोग
को बढ़ाएगा।
समाचार स्रोत: द इं डियन एक्सप्रेस

इं डियन रीजनल नेविगेशन सैटेलाइट सिस्टम (IRNSS): NavIC


• IRNSS को आधिकारिक तौर पर NAVIC कहा जाता है जो
भारतीय तारामंडल के साथ नेविगेशन (NavIC) भारतीय नक्षत्र के साथ नेविगेशन के लिए एक संक्षिप्त नाम है।

• द्वारा विकसित: इसरो


खबरों में क्यों?
• इसमें आठ उपग्रह शामिल हैं और यह पूरे भारत के भूभाग और
NavIC के उपयोग को बढ़ावा देने के लिए, भारतीय अंतरिक्ष
इसकी सीमाओ ं से 1,500 किमी (930 मील) तक कवर करता
अनुसंधान संगठन (ISRO) अपने सभी भविष्य के उपग्रहों में L1
है।
आवृत्ति पेश करेगा।

प्रमुख बिं दु देश सैटेलाइट नैविगेशन सिस्टम

के बारे में • यूनाइटेड स्टेट्स • जीपीएस

एल 1 आवृत्ति • रूस • ग्लोनास

• L1 सबसे पुराना और सबसे स्थापित GPS संकेत है। • चीन • बाइडू

• यह कम शक्ति का उपयोग करने वाले पहनने योग्य उपकरणों • यूरोपियन यूनियन • गैलीलियो

Download Monthly Articulate Magazine प्रबोधन | 75


• प्रणाली: NaVIC प्रणाली में एक अंतरिक्ष खंड (शुरुआत में, सात भारत का पहला निजी वाहन लॉन्चपैड
IRNSS उपग्रहों का एक समूह) और भू खंड (पूरे भारत में फैला
हुआ, 24x7 संचालित) है।इसरो पुराने उपग्रहों को बदलने और खबरों में क्यों?
उपग्रहों की संख्या बढ़ाने के लिए काम कर रहा है। अग्निकुल कॉसमॉस ने श्रीहरिकोटा में सतीश धवन अंतरिक्ष केंद्र
• उद्देश्य: इसे भारत में उपयोगकर्ताओ ं के साथ-साथ इसकी (एसडीएससी) में भारत का पहला निजी अंतरिक्ष यान लॉन्चपैड
सीमा से 1500 किमी तक फैले क्षेत्र में सटीक स्थिति सूचना स्थापित किया।
सेवा प्रदान करने के लिए डिज़ाइन किया गया है, जो इसका प्रमुख बिं दु:
प्राथमिक सेवा क्षेत्र है।
• इसे ISRO और IN-SPACe (भारतीय राष्ट्रीय अंतरिक्ष संवर्धन
भारत क्यों NAVIC को बढ़ावा दे रहा है? और प्राधिकरण केंद्र) के समर्थन में निष्पादित किया गया था।

• विशेष रूप से "रणनीतिक क्षेत्रों" के लिए नेविगेशन सेवा • यह निजी प्रतिभागियों के लिए भारतीय अंतरिक्ष उद्योग को
आवश्यकताओ ं के लिए विदेशी उपग्रह प्रणालियों पर निर्भरता खोलने की दिशा में एक बड़ा कदम होगा और यह ISRO/DOS
को दूर करने के लिए के समर्पण को दर्शाता है।

• स्वदेशी नाविक आधारित समाधान विकसित करने में लगे सम्बंधित खबर:
स्थानीय उद्योग को बढ़ावा देने के लिए अपने मंत्रालयों को
• हाल ही में स्काईरूट्स लै ब्स ने विक्रम एस- भारत का पहला
एनएवीआईसी अनुप्रयोगों का उपयोग करने के लिए प्रोत्साहित
निजी क्षेत्र का रॉकेट लॉन्च किया।
करना।
• इस मिशन को “मिशन प्रारंभ” नाम दिया गया था।
PYQ
समाचार स्रोत: द इं डियन एक्सप्रेस
Q. जीपीएस युग में ‘स्टैंडर्ड पोजिशनिंग सिस्टम’ और ‘प्रेसिजन
पोजिशनिंग सिस्टम’ से आप क्या समझते हैं? केवल सात
उपग्रहों को नियोजित करने वाले अपने महत्वाकांक्षी IRNSS
कार्यक्रम से भारत को मिलने वाले लाभों पर चर्चा करें। सरस 3: रेडियो टे लीस्कोप
(2015)
खबरों में क्यों?
समाचार स्रोत: द हिं दू
सरस 3, एक रेडियो टे लीस्कोप, ने ब्रह्मांड के पहले सितारों और
आकाशगंगाओ ं की प्रकृति का सुराग प्रदान किया है।

76 I प्रबोधन Download Monthly Articulate Magazine


प्रमुख बिं दु:
• रेडियो स्पेक्ट्रम-3 (सरस-3) पृष्ठभूमि के आकार का एं टीना
माप आवश्यक संवेदनशीलता तक पहुंचने वाला दुनिया
भर में पहला टे लीस्कोप है और इसकी वजह से खगोलविद
और शोधकर्ता रेडियो चमकदार आकाशगंगाओ ं के गुणों को
निर्धारित करने में सक्षम हुए हैं, जो केवल 200 मिलियन वर्षों
के बाद बनी हैं। बिग बैंग, एक अवधि जिसे कॉस्मिक डॉन के रूप
में जाना जाता है।

• रमन रिसर्च इं स्टीट्यूट (आरआरआई) (बेंगलु रु) द्वारा स्वदेशी


रूप से डिजाइन और निर्मित।

• हमारे “कॉस्मिक डॉन” से समय की गहराई से बेहद हल्की रेडियो


छवि स्रोत: द हिं दू तरंग संकेतों का पता लगाने के लिए जब शुरुआती ब्रह्मांड में
पहले तारे और आकाशगंगाएँ बनीं।

वॉलाबाई

• यह एक रेडियो टे लीस्कोप है जो खगोलविदों को रात के आकाश बारे में


का त्रि-आयामी नक्शा बनाने में मदद कर रहा है।
स्वॉट
• WALLABY का मतलब वाइडफ़ील्ड ASKAP L-बैंड लिगेसी • SWOT ‘शॉर्ट फॉर सरफेस वाटर एं ड ओशन टोपोग्राफी’, यह
ऑल-स्काई ब्लाइं ड सर्वे है। एक उन्नत रडार उपग्रह है ।

• उद्देश्य: दुनिया के महासागरों, झीलों और नदियों का व्यापक


सर्वेक्षण करना।
सतह जल और महासागर स्थलाकृति (SWOT) • एस उन्नत माइक्रोवेव रडार प्रौद्योगिकी का उपयोग करना

खबरों में क्यों? • इससे यह समझने में मदद मिलेगी कि वैश्विक तापमान और
जलवायु परिवर्तन को नियंत्रित करने वाली प्राकृतिक प्रक्रिया
SWOT नामक नासा के नेतृत्व वाले अंतर्राष्ट्रीय उपग्रह को दक्षिणी
में महासागर वायुमंडलीय गर्मी और कार्बन डाइऑक्साइड को
कैलिफोर्नि या से प्रक्षेपित किया गया।
कैसे अवशोषित करते हैं।
प्रमुख बिं दु:
समाचार स्रोत: द हिं दू

स्वास्थ्य
ज़ोंबी वायरस

ख़बरों में क्यों?

फ्रांसीसी शोधकर्ताओ ं ने हाल ही में रूस में एक जमी हुई झील के


नीचे छिपे 48,500 साल पुराने ज़ोंबी वायरस के पुनरुद्धार के बाद
एक और महामारी की संभावना के बारे में चेतावनी जारी की।
• जॉम्बी वायरस शब्द का प्रयोग उन वायरसों के लिए किया
जाता है जो हजारों वर्षों से सुप्त अवस्था में हैं।

Download Monthly Articulate Magazine प्रबोधन | 77


संभावित परिणाम:
प्रमुख बिं दु
• जीवित संस्कृतियों का अध्ययन करने के बाद, यह निर्धारित
किया गया है कि सभी “ज़ोंबी वायरस” में संक्रामक होने की के बारे में
क्षमता है और इसलिए “स्वास्थ्य संबंधी चिंता” का प्रतिनिधित्व रोग एक्स
करते हैं। • रोग एक्स एक अज्ञात रोगज़नक़ को संदर्भित करता है जो
• यह भविष्यवाणी की गई है कि जब पर्माफ्रॉस्ट पिघलता है, तो अगली महामारी का कारण बन सकता है।
लं बे समय तक सुप्त रहने वाले वायरस एक माइक्रोबियल • यह सार्वजनिक स्वास्थ्य आपातकालीन संदर्भ में अनुसंधान
कैप्टन अमेरिका की तरह निकलें गे, जिससे भविष्य में कोविड- एवं विकास के लिए तैयार की गई डब्ल्यूएचओ की प्राथमिकता
19 जैसी महामारी अधिक बार होगी। वाली बीमारियों की सूची का हिस्सा है

समाचार स्रोत: द इं डियन एक्सप्रेस WHO अगली महामारी पर कैसे नज़र रख रहा है ?
• WHO R&D ब्लूप्रिं ट पर निर्भर है - एक वैश्विक रणनीति और
तैयारी योजना - बड़े पैमाने पर स्वास्थ्य संकटों की जाँच करने
रोग एक्स के लिए।

• आर एं ड डी खाका
खबरों में क्यों?
» समन्वय में सुधार और एक सक्षम वातावरण को बढ़ावा देना
विश्व स्वास्थ्य संगठन (WHO) ने कहा कि वह प्राथमिकता वाले
» आर एं ड डी प्रक्रियाओ ं में तेजी लाना
रोगजनकों (रोग X) की सूची को अपडेट कर रहा है जो अगली
महामारी का कारण बन सकता है। » महामारी के लिए नए मानदंड और मानक विकसित करना

WHO की प्राथमिकता वाली बीमारियों की वर्तमान सूची में शामिल हैं :

बीमारियां स्थान जहां यह सबसे पहले पाई गईं

• कोविड-19 वुहान ,चीन

• क्रीमियन-कांगो रक्तस्रावी बुखार क्रीमिया

• इबोला वायरस रोग सूडान और रिपब्लिक ऑफ कांगो

• लस्सा बुखार लस्सा,नाइजीरिया

• मध्य पूर्व श्वसन सिं ड्रोम कोरोनावायरस (मिडिल ईस्ट सऊदी अरब
रेस्पिरेटरी सिं ड्रोम - MERS-CoV)

• गंभीर तीव्र श्वसन सिं ड्रोम (सीवियर एक्यूट रेस्पिरेटरी सिं ड्रोम गुओ ंगडॉन्ग, चीन
- SARS)

• निपाह मले शिया

• हेनीपाविरल रोग ऑस्ट्रेलिया

• रिफ्ट वैली बुखार केन्या

• जीका वायरस रोग युगांडा और तंजानिया

• मारबर्ग वायरस रोग मारबर्ग,जर्मनी और बेलग्रेड, यूगोस्लाविया (अब सर्बि या)

समाचार स्रोत: द हिं दू

खसरा

खबरों में क्यों?

मुंबई में खसरे का प्रकोप देखा गया है।

78 I प्रबोधन Download Monthly Articulate Magazine


छवि स्त्रोत: मेडिकवर • संचरण: खांसने और छींकने, निकट व्यक्तिगत संपर्क से फैलता
है।
प्रमुख बिं दु:

खसरा के बारे में समाचार स्त्रोत: द हिं दू


• खसरा या ‘खसरा’ एक अत्यधिक संक्रामक वायरल रोग है जो
ज्यादातर बच्चों को प्रभावित करता है।
मंकीपॉक्स का नाम बदलकर MPOX कर दिया गया
• पैरामाइक्सोवायरस परिवार में एक वायरस के कारण होता है।
खबरों में क्यों?
• टीकाकरण के अलावा खसरे का कोई विशिष्ट उपचार नहीं है।
» खसरे के टीके को अक्सर रूबेला और/या कण्ठमाला के टीके- WHO ने नस्लवादी कलं क का हवाला देते हुए मंकीपॉक्स का नाम
खसरा-कण्ठमाला-रूबेला (MMR) या खसरा-कण्ठमाला- बदलकर mpox कर दिया। दशकों पुराने पशु रोग का मूल नाम
रूबेला-वैरीसेला (MMRV) के साथ शामिल किया जाता है। भेदभावपूर्ण और नस्लवादी माना जा सकता है।

Download Monthly Articulate Magazine प्रबोधन | 79


प्रमुख बिं दु:
• दोनों नामों का एक साथ एक साल तक इस्तेमाल किया जाएगा,
जबकि ‘मंकीपॉक्स’ को चरणबद्ध तरीके से हटा दिया गया है।

मंकीपॉक्स के बारे में:


• मंकीपॉक्स वायरस एक ऑर्थोपॉक्सवायरस है, जो वायरस का
एक जीनस है जिसमें वेरियोला वायरस शामिल है, जो चेचक का
कारण बनता है।

• मनुष्यों में वायरस का पहला मामला 1970 में कांगो लोकतांत्रिक


गणराज्य (DRC) में दर्ज किया गया था।

• लक्षण: बुखार, सिरदर्द, मांसपेशियों में दर्द, पीठ दर्द और प्रमुख बिं दु:
थकावट।यह लिम्फ नोड्स में सूजन (लिम्फैडेनोपैथी) का
ह्यूमन पैपिलोमा वायरस के बारे में:
कारण भी बनता है।
• एचपीवी एक यौन संचारित संक्रमण है जो पुरुषों और महिलाओ ं
• संचरण: मंकीपॉक्स एक ज़ूनोसिस है, यानी एक ऐसी बीमारी दोनों में विभिन्न प्रकार के कैंसर का कारण बन सकता है।
जो संक्रमित जानवरों से मनुष्यों में फैलती है।
• यह सबसे आम यौन संचारित संक्रमण है।
• मंकीपॉक्स के लिए ऊष्मायन अवधि (संक्रमण से लक्षणों तक
• यह योनि, मौखिक और गुदा मैथुन सहित किसी भी प्रकार के
का समय) आमतौर पर 7-14 दिनों का होता है, लेकिन यह 5-21
यौन संपर्क के माध्यम से प्रेषित किया जा सकता है।
दिनों तक हो सकता है और अभी तक मंकीपॉक्स का कोई
सुरक्षित, सिद्ध इलाज नहीं है। • एचआईवी, धूम्रपान करने वालों और हार्मोनल गर्भ निरोधकों
का उपयोग करने वाले लोगों को एचपीवी से संबंधित कैंसर के
समाचार स्रोत: इं डियन एक्सप्रेस विकास का उच्च जोखिम हो सकता है।

सर्वाइकल कैंसर के बारे में:


• यह एक आम यौन संचारित संक्रमण है। कुछ प्रकार के एचपीवी
सरवाइकल कैंसर टीका
के साथ लं बे समय तक चलने वाला संक्रमण सर्वाइकल कैंसर
खबरों में क्यों? का मुख्य कारण है।

9-14 साल की लड़कियों को अगले साल सर्वाइकल कैंसर का टीका • काफी हद तक रोकथाम योग्य होने के बावजूद यह भारत में
दिया जाएगा। महिलाओ ं का दूसरा सबसे आम कैंसर है।

• 1.23 लाख मामलों और प्रति वर्ष लगभग 67,000 मौतों के साथ


भारत वैश्विक बोझ का लगभग पांचवां हिस्सा है।

• भारत में, सर्वाइकल कैंसर का 2020 में सभी कैंसर का 9.4%

80 I प्रबोधन Download Monthly Articulate Magazine


और नए मामलों का 18.3% (1,23,907) हिस्सा था। द्वारा बाजार प्राधिकरण के लिए अनुमोदित किया गया था।

• यदि सभी प्रीपेबर्टल लड़कियों को विश्व स्तर पर एचपीवी • यह टीका वीएलपी (वायरस जैसे कण) पर आधारित है, जो
टीकाकरण दिया जाए तो इसे समाप्त किया जा सकता है। हेपेटाइटिस बी के टीके के समान है, और एचपीवी वायरस के
एल1 प्रोटीन के खिलाफ एं टीबॉडी बनाकर सुरक्षा प्रदान करता
सर्ववैक वैक्सीन के बारे में: है।
• यह एक चतुर्भुज टीका है जो टीकाकरण प्रक्रिया के माध्यम से
सर्वाइकल कैंसर को रोकने में सक्षम है। महत्व:
• भारत में उपलब्ध एचपीवी टीके विदेशी निर्माताओ ं द्वारा 2,000
• यह एचपीवी के चार प्रकारों - 16, 18, 6 और 11 से सुरक्षा प्रदान
रुपये से 3,500 रुपये प्रति खुराक की अनुमानित लागत पर
करता है।
तैयार किए गए थे। सर्ववैक के काफी सस्ते होने की संभावना है,
• यह भारत का पहला स्वदेशी रूप से विकसित क्वाड्रिवेलेंट जिसकी कीमत लगभग रु. 200 से 400।
ह्यूमन पैपिलोमावायरस वैक्सीन (qHPV) है।
• इसके अतिरिक्त, इसने खुराक और आयु समूहों दोनों में सभी
• : बायोटे क्नोलॉजी विभाग (DBT) के सहयोग से सीरम इं स्टीट्यूट लक्षित एचपीवी उपभेदों के खिलाफ एक मजबूत एं टीबॉडी
ऑफ इं डिया (SII) द्वारा विकसित है। प्रतिक्रिया दिखाई है जो बेसलाइन से लगभग 1,000 गुना अधिक
• जुलाई 2022 में, वैक्सीन को ड्रग कंट्रोलर जनरल ऑफ इं डिया है।

विदेशी निर्माताओ ं द्वारा HPV टीके सर्वावैक

लगभग रुपए 2002-3500 200-400

समाचार स्रोत: द इं डियन एक्सप्रेस

BF.7: ओमिक्रॉन का सब-वैरिएं ट

माना जाता है कि चीन में कोविड-19 संक्रमण में मौजूदा उछाल वहां
फैले ओमिक्रोन के बीएफ.7 सब-वैरिएं ट द्वारा संचालित है।
• जब वायरस उत्परिवर्तित होते हैं, तो वे वंशावली और उप-वंश
बनाते हैं - जैसे पेड़ के मुख्य तने में शाखाएं और उप-शाखाएं
निकलती हैं ; SARS-CoV-2

• BF.7 BA.5.2.1.7 के समान है, जो कि Omicron उप-वंश BA.5


की उप-वंशावली है।

राष्ट्रीय दुर्ल भ रोग नीति 2021

खबरों में क्यों?


• हाल ही में, एक राज्यसभा सदस्य ने राष्ट्रीय दुर्ल भ रोग नीति पर छवि स्त्रोत: मनीकंट्रोल
चिंता जताई।
दुर्ल भ रोगों के बारे में:
• एक ‘दुर्ल भ बीमारी’ को कम व्यापकता वाली स्वास्थ्य स्थिति
के रूप में परिभाषित किया जाता है जो सामान्य आबादी में
अन्य प्रचलित बीमारियों की तुलना में कम संख्या में लोगों को
प्रभावित करती है।
» 6,000-8,000 वर्गीकृत दुर्ल भ बीमारियां हैं, लेकिन 5% से कम
के पास इलाज के लिए उपचार उपलब्ध हैं।

» उदाहरण: लाइसोसोमल स्टोरेज डिसऑर्डर (एलएसडी), पोम्पे


रोग, सिस्टिक फाइब्रोसिस, मस्कुलर डिस्ट्रॉफी, स्पाइना
बिफिडा, हीमोफिलिया, आदि।

Download Monthly Articulate Magazine प्रबोधन | 81


— हीमोफिलिया एक दुर्ल भ आनुवंशिक विकार है जो खून के एन्यूरिज्म
जमने की प्रक्रिया को प्रभावित करता है।
खबरों में क्यों?
— यह किसी एक प्रोटीन की कमी के कारण होता है जो रक्त
को जमने में मदद करते हैं। अमेरिकी पत्रकार ग्रांट वाहल का कतर विश्व कप के दौरान थोरेसिक
एओर्टि क एन्यूरिज्म से निधन हो गया।
— हीमोफिलिया के परिणामस्वरूप सहज रक्तस्राव चोट या
सर्जरी के बाद रक्तस्राव हो सकता है। अभिनेत्री एमिलिया क्लार्क ने भी मस्तिष्क एन्यूरिज्म
» लगभग 95% दुर्ल भ बीमारियों का कोई स्वीकृत उपचार नहीं (धमनीविस्फार ) के साथ अपने संघर्ष का खुलासा किया।
है, और 10 में से 1 से कम रोगियों को रोग-विशिष्ट उपचार प्राप्त अर्थ:
होता है। • एन्यूरिज्म रक्त वाहिका की दीवार का एक स्थानीय कमजोर
» इन बीमारियों की विभिन्न देशों में अलग-अलग परिभाषाएँ हैं होना है, जिससे उस क्षेत्र में वाहिका फूल जाती है।
और उन बीमारियों से हैं जो जनसंख्या के 10,000 में 1 से 6 प्रति • यह समय के साथ आकार में बढ़ता है, और प्रभावित रक्त वाहिका
10,000 में प्रचलित हैं। की दीवार उत्तरोत्तर कमजोर होती जाती है।

दुर्ल भ रोगों की राष्ट्रीय नीति के बारे में: • गंभीर रूप से प्रभावित होने के लिए मस्तिष्क और हृदय की रक्त
वाहिकाएं सबसे आम स्थान हैं।
• सरकार ने दुर्ल भ बीमारी के रोगियों के इलाज के लिए 2021 में
दुर्ल भ बीमारियों के लिए राष्ट्रीय नीति (एनपीआरडी) शुरू की है।
संभावित कारण:

राष्ट्रीय दुर्ल भ रोग नीति की मुख्य विशेषताएं • धूम्रपान, आयु, उच्च कोले स्ट्रॉल, मोटापा, उच्च रक्तचाप, या
ऊतक विकार।
• वर्गीकरण: नीति ने दुर्ल भ बीमारियों को तीन समूहों में वर्गीकृत
किया है: • गर्भावस्था भी तिल्ली के एन्यूरिज्म के जोखिम को बढ़ा सकती
» समूह 1: एक बार के उपचारात्मक उपचार के लिए उपयुक्त है।
विकार
उपचार और रोकथाम:
» समूह 2: जिन्हें दीर्घकालिक या आजीवन उपचार की
• फ्लो डायवर्जन स्टेंट, मेश स्टेंट आदि से जुड़ी कुछ नवीन
आवश्यकता होती है।
तकनीकों को लागू किया जा रहा है।
» समूह 3: ऐसे रोग जिनके लिए निश्चित उपचार उपलब्ध है,
• एक स्वस्थ जीवन शैली, धूम्रपान छोड़ना और संतुलित आहार
लेकिन लाभ, बहुत अधिक लागत और आजीवन चिकित्सा के
ले ने से रोकथाम में मदद मिलती है।
लिए इष्टतम रोगी चयन करना चुनौतियां हैं।
• नीति का उद्देश्य स्वदेशी अनुसंधान और दवाओ ं के स्थानीय
उत्पादन पर ध्यान केंद्रित करना है।
बिहार में जहरीली शराब त्रासदी
• जो लोग दुर्ल भ बीमारियों से पीड़ित हैं, उन्हें राष्ट्रीय आरोग्य निधि
की एकछत्र योजना के तहत 20 लाख रुपये तक की वित्तीय खबरों में क्यों?
सहायता दी जाएगी।
बिहार सरकार ने सारण जिले में अब तक जहरीली शराब पीने से हुई
• क्राउडफंडिं ग मैकेनिज्म: दुर्ल भ बीमारियों के इलाज की लागत 82 मौतों की जांच के लिए एसआईटी का गठन किया है।
को कवर करने के लिए पॉलिसी क्राउडफंडिंग मैकेनिज्म का
शराब का विनियमन:
उपयोग करेगी।
• राज्य विषय: शराब से संबंधित कानून, उत्पाद कर, और शराब
• दुर्ल भ बीमारियों की एक राष्ट्रीय अस्पताल आधारित रजिस्ट्री:
का निर्माण, वितरण और बिक्री पूरी तरह से राज्य के अधिकार
- यह रजिस्ट्री पर्याप्त डेटा सुनिश्चित करने के लिए बनाई जाएगी
में हैं।
और अनुसंधान और विकास में रुचि रखने वालों के लिए ऐसी
बीमारियों की व्यापक परिभाषा उपलब्ध है। • अनुच्छेद 47 के निदेशक सिद्धांत शराब के उपयोग पर प्रतिबंध
को प्रोत्साहित करते हैं।
निष्कर्ष:
• 2016: बिहार में शराब की बिक्री और उपयोग पर प्रतिबंध लगा
राष्ट्रीय दुर्ल भ रोग नीति सतत विकास लक्ष्यों, विशेष रूप से लक्ष्य दिया गया।
3 (अच्छे स्वास्थ्य और कल्याण) को प्राप्त करने की दिशा में एक
• बिहार और अन्य राज्यों में शराब की कालाबाजारी और स्थानीय
महत्वपूर्ण कदम है।
स्तर पर उत्पादित नकली शराब के सेवन से होने वाली मौतों का
समाचार स्रोत: द हिं दू सिलसिला जारी है।

82 I प्रबोधन Download Monthly Articulate Magazine


एमएमआर क्या है ?

मातृ मृत्यु अनुपात (MMR) एक विशिष्ट अवधि के दौरान प्रति


100,000 जीवित जन्मों पर मातृ मृत्यु का अनुपात है।

के बारे में
• इस उपलब्धि के साथ, भारत ने प्रति लाख जीवित जन्मों पर 100
से कम MMR का राष्ट्रीय स्वास्थ्य नीति (NHP) लक्ष्य हासिल
कर लिया है।

• सतत विकास लक्ष्य (एसडीजी) लक्ष्य हासिल करने वाले राज्यों


की संख्या के मामले में हुई प्रगति केरल (19), महाराष्ट्र (33),
तेलं गाना (43), आंध्र प्रदेश (45), तमिलनाडु (54), झारखंड हैं।
(56), गुजरात (57) और कर्नाटक (69)।
अन्य राज्य और केंद्र शासित प्रदेश जहां शराब प्रतिबंधित है :
एमएमआर को कम करने के लिए शुरू की गई योजनाएं :
• 1960: गुजरात राज्य ने अपने गठन के बाद से मद्यनिषेध
• राष्ट्रीय स्वास्थ्य मिशन (एनएचएम)
कानून लागू किया।
• सुरक्षित मातृत्व आश्वासन (सुमन)
• 1989: नागालैं ड शराब पूर्ण निषेध अधिनियम, बिक्री और
खपत पर प्रतिबंध। • प्रधानमंत्री सुरक्षित मातृत्व अभियान (PMSMA)

• 2019: 1997 से पहले के प्रतिबंध को संशोधित करने के लिए • प्रधानमंत्री मातृ वंदना योजना (PMMVY) आदि
मिजोरम शराब निषेध अधिनियम पारित किया गया।

• केंद्र शासित प्रदेश लक्षद्वीप भी शराब पर प्रतिबंध लगाता है।


बेस/क्षार संपादन तकनीक

खबरों में क्यों?

मातृ मृत्यु दर (एमएमआर) बेस/क्षार संपादन तकनीक (बेस एडिटिंग टे क्निक)का उपयोग
करके एक किशोर लड़की के लाइलाज कैंसर को ठीक कर दिया
ख़बरों में क्यों? गया है।

• भारत के महारजिस्ट्रार (आरजीआई) के अनुसार 2018-20 प्रमुख बिं दु


में मातृ मृत्यु दर (एमएमआर) प्रति लाख जीवित जन्मों में
के बारे में
उल्ले खनीय रूप से घटकर 97 हो गई।
बेस एडिटिं ग
• भारत 2030 तक प्रति लाख जीवित जन्मों पर 70 से कम
• बेस आनुवंशिक कोड के निर्माण खंड हैं।
एमएमआर के एसडीजी लक्ष्य (एसडीजी 3.1) को पूरा करने की
राह पर है। • चार प्रकार के आधार - एडेनिन (ए), साइटोसिन (सी), गुआनिन
(जी), और थाइमिन (टी) मौजूद हैं।

• बेस एडिटिंग में वैज्ञानिक बेस की आणविक संरचना को बदलते


हैं जो बदले में आनुवंशिक निर्देशों को बदलते हैं।

Download Monthly Articulate Magazine प्रबोधन | 83


संभावित प्रश्न:

Q. मानव जाति के लिए बेस एडिटिंग तकनीक के संभावित लाभ क्या हैं?

समाचार स्रोत: द इं डियन एक्सप्रेस

84 I प्रबोधन Download Monthly Articulate Magazine


Download Monthly Articulate Magazine प्रबोधन | 85
अमूर्त सांस्कृतिक विरासत

खबरों में क्यों?

बगेट ( Baguette) मुख्य फ्रांसीसी रोटी - को संयुक्त राष्ट्र की अमूर्त


सांस्कृतिक विरासत की सूची में अंकित किया गया था।

छवि क्रेडिट: द इं डियन एक्सप्रेस

के बारे में

अमूर्त सांस्कृतिक विरासत क्या है?


• अमूर्त सांस्कृतिक विरासत का अर्थ है प्रथाओ ं, अभ्यावेदन,
अभिव्यक्ति, ज्ञान और कौशल - हमारे पूर्वजों से विरासत में
मिले और हमारे वंशजों को दिए गए।

• इसमें मौखिक परंपराएं , प्रदर्शन कलाएं , सामाजिक प्रथाएं ,


अनुष्ठान, उत्सव की घटनाएं , ज्ञान और प्रथाएं शामिल हो
सकती हैं।

• संस्कृति मंत्रालय, भारत सरकार देश के भीतर अमूर्त


सांस्कृतिक विरासत को बढ़ावा और संरक्षित करती है।

समाचार स्रोत: इं डियन एक्सप्रेस

यूनेस्को की विश्व धरोहर स्थलों की संभावित सूची में


तीन और स्थल जोड़े गए

खबरों में क्यों?

गुजरात के वडनगर शहर, मोढे रा में प्रतिष्ठित सूर्य मंदिर, और त्रिपुरा


में उनाकोटी की रॉक कट मूर्ति यां UNCESO विश्व धरोहर स्थलों की
अस्थायी सूची में शामिल की गई हैं।

प्रमुख बिं दु:


• यूनेस्को की अस्थायी सूची में अब भारत के 52 स्थल हैं।

• यूनेस्को की अस्थायी सूची “उन संपत्तियों की सूची है जो प्रत्येक


राज्य पार्टी नामांकन के लिए विचार करना चाहती है।

86 I प्रबोधन Download Monthly Articulate Magazine


अष्टाध्यायी पाणिनी

वह शायद चौथी शताब्दी ईसा पूर्व - छठी शताब्दी ईसा पूर्व, बुद्ध और
खबरों में क्यों?
महावीर की उम्र में रहते थे।
उनकी पीएच.डी. थीसिस प्रकाशित, कैम्ब्रिज के विद्वान डॉ. ऋषि
वह संभवतः सलतुरा (गांधार) में रहता था।
राजपोपत ने संस्कृत की सबसे बड़ी पहेली, ‘अष्टाध्यायी’ में पाई जाने
वाली व्याकरण की समस्या को हल करने का दावा किया है। साहित्यिक कृति : अष्टाध्यायी

‘अष्टाध्यायी’
• ‘अष्टाध्यायी’ एक भाषाई पाठ है, जो लगभग 6वीं शताब्दी ईसा
पूर्व का है, जो इस बात के लिए मानक निर्धारित करता है कि
संस्कृत को कैसे लिखा और बोला जाना चाहिए।

• इसने एक प्रकार के आशुलिपि में 4,000 से अधिक व्याकरणिक


नियम निर्धारित किए, जो मामलों, मनोदशाओ ं, व्यक्तियों, काल
आदि के नाम के लिए एकल अक्षरों या शब्दांशों को नियोजित
करता है।

• अष्टाध्यायी पर प्रसिद्ध टिप्पणियों में पतंजलि का महाभाष्य


(दूसरी शताब्दी ईसा पूर्व) और जयादित्य और वामन की
काशिका वृत्ति (7वीं शताब्दी ईस्वी) शामिल हैं।

छवि क्रेडिट: द इं डियन एक्सप्रेस

प्रमुख बिं दु
संग्रहालय अनुदान योजना (एमजीएस) और
के बारे में विज्ञान की संस्कृति को बढ़ावा देने के लिए योजना
खोज (एसपीओसी)
• कैंब्रिज के विद्वान डॉ. ऋषि राजपोपत का दावा है कि ईसा पूर्व
खबरों में क्यों?
चौथी शताब्दी में पाणिनि द्वारा लिखे गए एक प्राचीन संस्कृत
ग्रंथ अष्टाध्यायी में एक समस्या का समाधान हो गया है। संस्कृति मंत्रालय संग्रहालयों के संशोधन और आधुनिकीकरण
के लिए संग्रहालय अनुदान योजना (एमजीएस) और विज्ञान की
• पाठ में एक भाषा मशीन शामिल है’ जो रूट और प्रत्यय दिए
संस्कृति को बढ़ावा देने के लिए योजना (एसपीओसीएस) संचालित
जाने पर व्याकरणिक रूप से सही शब्दों और वाक्यों का निर्माण
करता है।
करती है, लेकिन एक ही समय में दो या दो से अधिक नियमों
को लागू करने के कारण भ्रम के कारण मशीन हमेशा सटीक प्रमुख बिं दु:
नहीं थी। • मंत्रालय: संस्कृति मंत्रालय

• पाणिनि ने इस मुद्दे को हल करने के लिए एक “मेटा-नियम” • उद्दे श्य:


प्रदान किया, जिसे ऐतिहासिक रूप से “अष्टाध्यायी जीत के क्रम » मौजूदा संग्रहालयों और विज्ञान केंद्रों में विविध प्रदर्शनियों,
में बाद में आने वाले नियम” के रूप में व्याख्या की गई थी। दीर्घाओ ं और आगंतुकों की सुविधाओ ं के उन्नयन और एक
• डॉ राजपोपत ने तर्क दिया कि यह व्याख्या गलत थी और मेटा- अनुकूल स्थानिक सेटिंग बनाने के लिए वित्तीय सहायता प्रदान
नियम को शब्द के दाहिने हाथ पर लागू किया जाना चाहिए, करना।
जिससे “अष्टाध्यायी” अंततः एक सटीक भाषा मशीन बन जाती » संग्रहालय संग्रह का डिजिटलीकरण
है। • पिछले दो वर्षों (2020-21 और 2021-22) के दौरान संग्रहालय
अनुदान योजना के तहत 8 संग्रहालयों को वित्त पोषित किया
पाणिनि का मेटा नियम क्या है ?
गया है।
समान शक्ति के दो नियमों के बीच विरोध की स्थिति में, अष्टाध्यायी
के क्रम में बाद में आने वाले नियम की जीत होती है।

महत्व:
कोच्चि-मुजिरिस बिएनेल
यह खोज पाणिनि की प्रणाली का उपयोग करके लाखों संस्कृत शब्दों
के निर्माण की अनुमति देती है और संभावित रूप से कंप्यूटरों को खबरों में क्यों?
संस्कृत भाषा एल्गोरिदम सिखाने के लिए उपयोग की जा सकती है। कोच्चि-मुजिरिस बिएनेल उद्घाटन के पांचवें संस्करण को 23
दिसंबर (2011 में स्थापित) के लिए स्थगित कर दिया गया था।

Download Monthly Articulate Magazine प्रबोधन | 87


प्रमुख बिं दु:

कोच्चि-मुज़िरिस बिएनेल:
• उद्दे श्य: “एक ऐसा मंच बनाना जो समकालीन, वैश्विक दृश्य
कला सिद्धांत और अभ्यास को भारत में पेश करेगा।”

कला द्विवार्षि क क्या हैं?


• कला का एक अंतरराष्ट्रीय बड़े पैमाने का प्रदर्शन जो हर दो
साल में एक विशेष स्थल पर होता है। इब्न बतूता और उनकी भारत यात्रा के बारे में:
• द्विवार्षि क आमतौर पर गैर-वाणिज्यिक उद्यम होते हैं - कला वह 14वीं सदी का एक मोरक्कन खोजकर्ता था, जिसने एशिया,
मेलों के विपरीत - जो एक क्यूरट
े ोरियल थीम के आसपास यूरोप और अफ्रीका के चारों ओर बड़े पैमाने पर यात्रा की और दिल्ली
होता है। में मुहम्मद बिन तुगलक के दरबार का दौरा किया।
• उनके द्वारा भारत में देखे गए स्थान: सिं ध, दिल्ली, सारा का
समाचार स्रोत: द इं डियन एक्सप्रेस
राजपूत साम्राज्य,कालीकट, मदुर,ै चटगाँव, उत्तरी असम।

• उनके द्वारा महत्वपूर्ण उल्ले ख:


» सिं धु के तट पर भारतीय गैंडों का अस्तित्व।
अल रिहला
» कुव्वत अल-इस्लाम मस्जिद और कुतुब मीनार की भव्यता।
खबरों में क्यों?
» भारत में अश्व आधारित डाक कूरियर प्रणाली चालू थी।
अल रिहला” कतर में फीफा विश्व कप 2022 के लिए आधिकारिक
» सती प्रथा - विधवाओ ं को जलाना और जाति प्रथा प्रचलित थी।
मैच बॉल थी।
» उपजाऊ मिट्टी ने किसानों को प्रति वर्ष 2 फसलें उगाने में सक्षम
यह महत्वपूर्ण क्यों है?
बनाया।
• अल रिहला का अर्थ अरबी में “यात्रा” है।
» देवगिरि के शानदार किले का दौरा किया, जिसका नाम
• “रिहला” इब्न बतूता के यात्रा वृत्तांत का नाम भी है। तुगलकों द्वारा दौलताबाद रखा गया।

» कालीकट और कोल्लम के बंदरगाहों के माध्यम से काली मिर्च,


सूती कपड़े, मलमल, रेशम, ब्रोकेड और साटन जैसी वस्तुओ ं का
समुद्री व्यापार।

महत्त्वपूर्ण दिन:

तारीख़ आयोजन विवरण

3 दिसंबर राजेंद्र प्रसाद जयंती • 3 दिसंबर, 1884 को जीरादेई, सीवान (बिहार) में जन्मे।

• 1911 में कलकत्ता अधिवेशन के दौरान कांग्रेस में शामिल हुए।

• चंपारण और असहयोग आंदोलन में सक्रिय रूप से भाग लिया।

• बंबई में 1934 के कांग्रेस अधिवेशन के अध्यक्ष।

• उन्हें भारत रत्न से सम्मानित किया गया था।

88 I प्रबोधन Download Monthly Articulate Magazine


3 दिसंबर खुदीराम बोस जयंती • उनका जन्म 3 दिसंबर 1889 को हुआ था।

• 1908 में वे ‘अनुशीलन समिति’ से जुड़े।

• उन्होंने और प्रफुल्ल चंद्र चाकी ने कलकत्ता के मुख्य प्रेसीडेंसी


मजिस्ट्रेट किं ग्सफोर्ड को मारने की कोशिश की, हालांकि, वे
असफल रहे और खुदीराम बोस को मौत की सजा सुनाई गई।

• 11 अगस्त 1908 को 18 साल की उम्र में उन्हें फाँसी दे दी गई।

19 दिसंबर 1927 में रामप्रसाद बिस्मिल, • ये तीनों काकोरी ट्रेन डकैती (काकोरी षडयंत्र) का हिस्सा थे, जो
रोशन सिं ह और अशफाक 9 अगस्त, 1925 को लखनऊ, (यूपी) के पास एक गाँव काकोरी
उल्ला खां को फांसी दे दी गई में हुआ था।

• राम प्रसाद बिस्मिल, रोशन सिं ह और अशफाक-उला खान


हिं दुस्तान रिपब्लिकन एसोसिएशन (HRA) के सदस्य थे, जिसे
बाद में हिं दुस्तान सोशलिस्ट रिपब्लिकन एसोसिएशन (HSRA)
के नाम से जाना गया।

• उन्हें काकोरी षड्यंत्र का दोषी ठहराया गया और 19 दिसंबर 1927


को फैजाबाद जेल में फांसी दे दी गई।

20 दिसंबर बीसी चट्टोपाध्याय ने वन्दे • उन्हें भारत का सबसे महान उपन्यासकार और कवि माना जाता
मातरम् की रचना की है।

• उन्होंने संस्कृत में वंदे मातरम गीत की रचना की, जो स्वतंत्रता


संग्राम में लोगों के लिए प्रेरणा का स्रोत था।

• उनके महाकाव्य उपन्यास आनंदमठ - सन्यासी विद्रोह (1770-


1820) की पृष्ठभूमि में सेट इस गीत ने ने भारत को अपना
राष्ट्रीय गीत वंदे मातरम दिया।

22 दिसंबर राष्ट्रीय गणित दिवस • हर साल 22 दिसंबर को श्रीनिवास रामानुज के जन्मदिन को


(रामानुजन जन्मदिन) ‘राष्ट्रीय गणित दिवस’ के रूप में मनाया जाता है।

गणित में योगदान:


• सूत्र और समीकरण: पाई के लिए उसकी अनंत श्रृंखला की
खोज।

• उन्होंने गणित की कई जटिल समस्याओ ं को हल करने के लिए


गेम थ्योरी दी।

• 1729 को रामानुज की संख्या के रूप में जाना जाता है।

• अन्य योगदानों में हाइपरज्यामितीय श्रृंखला, रीमैन श्रृंखला,


दीर्घवृत्तीय समाकल, मॉक थीटा फलन, अपसारी श्रृंखला का
सिद्धांत और जीटा फलन के कार्यात्मक समीकरण शामिल हैं।

27 दिसंबर कांग्रेस 1911 का कलकत्ता • कलकत्ता में आयोजित किया गया


अधिवेशन
• अध्यक्षता बिशन नारायण धर ने की।

• जन गण मन पहली बार कांग्रेस अधिवेशन में गाया गया था।

Download Monthly Articulate Magazine प्रबोधन | 89


30 दिसंबर एससी बोस ने पोर्ट ब्ले यर में • 30 दिसंबर 1943 को सुभाष चंद्र बोस ने पहली बार भारतीय
भारतीय ध्वज लहराया ध्वज फहराया।
Unacademy Margdarshak - Your guide to
success!• उन्हों
Unacademy
ने अंडमान और निकोबार has द्वीपों काalways
नाम बदलकर “शहीद-
envisioned द्वीप”
helping learners
और “स्वराज-द्वीप” रखा।to crack their
dream exams.
• उनके And
प्रसिद्धin
नारेthis mission,
हैं “तुम मुझे खून दो,we bring
मैं तुम्हें आजादी दूंगा” और
to you this “दिल्ली
new चलो”।channel that will help you
to plan, choose and prepare for the right
• वे its
career and हरिपुpath
रा सत्र (1938)
to successमें कांग्रेस अध्यक्ष
based बने on और त्रिपुरी सत्र,
your 1939 (म.प्र.)concentration
dream, का राष्ट्रपति चुनाव जीता। power,
competitive strength, attitude, skills,
interests, and passion.
30 दिसंबर मुस्लिम लीग की स्थापना की • अखिल भारतीय मुस्लिम लीग नामक एक राजनीतिक दल की
स्थापना 1906 में ब्रिटिश भारत में हुई थी और तब इसे मुस्लिम
Whether you are in class 10th or UG, whether you
लीग के रूप में जानाare
जाने preparing
लगा। for a
Govt. job exam or already working in an MNC, we will bring career
• यह भारतीय मुसलमानों की चिंताओ ं की वकालत करने और
guiding videos for all of you with real-life examples. Make the most
भारतीय राष्ट्रीय कांग्रेस के प्रतिद्वंद्वी संगठन के रूप में स्थापित
with career guidance from experts like bureaucrats, top rankers, and
किया गया था।
Top Unacademy Educators that will inspire you to pursue your
dreams with precision and a renewed passion.

UPSC Unstoppables - A channel to help


every UPSC aspirant in preparation for the
Current Affairs. This is a one-stop solution
for current affairs related to every domain
and source, i.e. leading newspapers like
The Hindu, and Indian Express, leading
magazines like Yojana, Kurukshetra, Down
to earth, core subjects like International
Relations, Economy, Polity, Science &
Technology, Environment, and others.

90 I प्रबोधन Download Monthly Articulate Magazine


MAG10

92 Subscribe to Unacademy
September 2022

Download Monthly Articulate Magazine प्रबोधन | 91


92 I प्रबोधन Download Monthly Articulate Magazine
2022 फीफा विश्व कप की मुख्य विशेषताएं :
जे रॉबर्ट ओपेनहाइमर- परमाणु बम के जनक
• परमाणु बम का श्रेय जे रॉबर्ट ओपेनहाइमर को दिया जाता है। फीफा ने टू र्नामेंट के सर्वश्रेष्ठ प्रदर्शन करने वालों के लिए कुछ
प्रतिष्ठित पुरस्कारों की घोषणा की और ये निम्नलिखित हैं,
• क्रिस्टोफर नोलन की बहुप्रतीक्षित बायोपी की प्रत्याशा में
• गोल्डन बूट (सर्वोच्च गोल): किलियन एम्बाप्पे (फ्रांस)
हाल ही में मीडिया का ध्यान आकर्षित करने के आलोक में
अमेरिकी सरकार ने प्रसिद्ध वैज्ञानिक जे. रॉबर्ट ओपेनहाइमर • गोल्डन ग्लव्स: एमिलियानो मार्टि नेज (अर्जेंटीना)।
पर मैक्कार्थी युग के अनुचित आरोपों को कानूनी रूप से बरी
• युवा खिलाड़ी: एं जो फर्नांडीज (अर्जेंटीना)।
कर दिया है।
• गोल्डन बॉल (टू र्नामेंट का सर्वश्रेष्ठ प्रदर्शन): लियोनेल मेसी
(अर्जेंटीना)।

• फीफा फेयर प्ले अवार्ड: इं ग्लैं ड


पी टी उषा
विश्व कप गेंद के बारे में:
खबरों में क्यों? • अल-रिहाला पहली विश्व कप गेंद है जो विशेष रूप से पानी
भारतीय ओलं पिक संघ (IOA) के हालिया चुनाव में, प्रतिष्ठित महिला आधारित रंगों और गोंद से बनाई गई है।
एथलीट पीटी उषा IOA की पहली महिला अध्यक्ष चुनी गईं। • क्वार्टर फाइनल तक अल-रिहाला का इस्तेमाल किया गया।
पीटी उषा के बारे में: • सेमीफाइनल और फाइनल मैच में अल-हिल्म का इस्तेमाल
• पी.टी. उषा कई एशियाई खेलों की स्वर्ण पदक विजेता थीं, किया गया था।
जिन्होंने 1984 में लॉस एं जिल्स में ओलं पिक 400 मीटर बाधा
• फीफा विश्व कप के पहले संस्करण में कोई आधिकारिक गेंद
दौड़ के फाइनल में चौथा स्थान हासिल किया था।
नहीं थी लेकिन दोनों फाइनलिस्ट अर्जेंटीना (टिएन्टो) और
• 1981 में उन्होंने 100 और 200 मीटर का राष्ट्रीय रिकॉर्ड बनाया। उरुग्वे (टी-मॉडल बॉल) अपनी-अपनी गेंद ले कर आए।
• हाल ही में उन्हें 2022 में राज्यसभा के लिए मनोनीत किया गया
आधिकारिक शुभंकर:
था।
• लाईब आधिकारिक शुभंकर था, और अगर इसका अनुवाद
भारतीय ओलं पिक संघ के बारे में: किया जाए, तो इसे “सुपर-स्किल्ड प्ले यर” के रूप में जाना जाता
• स्थापना: 1927 में सोसायटी पंजीकरण अधिनियम 1860 के है।
तहत एक गैर-लाभकारी संगठन के रूप में।

• वर्तमान में यह युवा और खेल मंत्रालय के तहत एक स्वायत्त


निकाय है।

कार्य:
• यह ओलं पिक, एशियाई खेलों आदि के लिए खिलाड़ियों का
चयन करता है।

• यह राष्ट्रमंडल खेलों के लिए खिलाड़ियों का चयन करने के


लिए भारतीय राष्ट्रमंडल खेल संघ के रूप में कार्य करता है।

जूल्स रिमेट ट्रॉफी


फीफा विश्व कप 2023
खबरों में क्यों?
खबरों में क्यों? हाल ही में, फीफा विश्व कप की मेजबानी कतर द्वारा की गई थी, और
फुटबॉल विश्व कप का 2022 संस्करण अर्जेंटीना ने फ्रांस को हराकर जूल्स रिमेट ट्रॉफी 1970 तक ट्रॉफी का आधिकारिक नाम था।
जीता था। इस विश्व कप की मेजबानी कतर ने की थी। जूल्स रिमेट ट्रॉफी के बारे में:
फीफा (फेडरेशन इं टरनेशनेल डी फुटबॉल) विश्व कप के बारे में: • ट्रॉफी एबल लाफले र, एक फ्रांसीसी मूर्ति कार द्वारा बनाई गई
• यह हर चौथे साल आयोजित किया जाता है। थी, और इसका नाम फीफा टू र्नामेंट के संस्थापक और अध्यक्ष
के नाम पर रखा गया था।
• पहला विश्व कप 1930 में उरुग्वे में आयोजित किया गया था और
इसे उरुग्वे ने जीता था। • ट्रॉफी पर सोना चढ़ाया गया था और इसे लापीस लाजुली के
आधार पर बनाया गया था।
• ट्रॉफी, 1970 तक जूल्स रिमेट ट्रॉफी के रूप में जानी जाती थी
जिसने टू र्नामेंट का प्रस्ताव रखा था। • 1970 में ब्राजील ने इटली पर 4-1 से जीत के साथ अपना तीसरा
खिताब जीतकर पूरी तरह से ट्रॉफी का दावा किया।

Download Monthly Articulate Magazine प्रबोधन | 93


• 1983 में ब्राजील से ट्रॉफी फिर से चोरी हो गई थी और माना पुरस्कार जीता है।
जाता है कि यह पिघल गई थी।
• गुजराती भाषा में एक और भारतीय फिल्म, छे लो शो (अनुवाद
- लास्ट फिल्म शो) को भी पुरस्कारों के लिए शॉर्टलिस्ट किया
गया था।
किताब: ऐज़ गुड एज़ माई वर्ड
• ले खक: केएम चंद्रशेखर

• वह 2007 से 2011 तक भारत के कैबिनेट सचिव और 1970 बैच ऑक्सफोर्ड डिक्शनरी का वर्ड ऑफ द ईयर 2022
के आईएएस अधिकारी थे। पुस्तक हमारे संवैधानिक, वैधानिक • गोब्लिन मोड को ऑक्सफोर्ड डिक्शनरी के वर्ड ऑफ द ईयर के
और नियामक प्राधिकरणों के कार्यों के संबंध में पर्याप्त सामग्री रूप में चुना गया है।
प्रदान करती है।
• यह शब्द उस व्यवहार को संदर्भित करता है जो “अपरिमित रूप से
आत्म-अनुग्रहकारी, अकर्मण्य, मैलापन या लालची है, आमतौर
पर एक तरह से जो सामाजिक मानदंडों या अपेक्षाओ ं का खंडन
पुस्तक: सिनेमा स्पेक्युलेशन करता है” - ऐसी विशेषताएं जो कई लोगों ने लॉकडाउन के
दौरान प्रदर्शित की हों। उदाहरण के लिए, इं स्टाग्राम पर बिना
• ले खक: क्वेंटिन टारनटिनो
फिल्टर/बेस्ट ड्रेस आदि के सेल्फी अपलोड करना।
• वह एक प्रसिद्ध हॉलीवुड फिल्म निर्माता हैं जिन्हें किल बिल
और Django Unchained के लिए जाना जाता है।

मरियम-वेबस्टर- वर्ड ऑफ द ईयर 2022


• “गैसलाइटिंग” शब्द, जिसे मरियम-वेबस्टर “मुख्य रूप से अपने
95वें अकादमी पुरस्कारों की सूची
स्वयं के लाभ के लिए किसी को अत्यधिक धोखा देने का कार्य
• 2023 ऑस्कर शॉर्टलिस्ट हाल ही में एकेडमी ऑफ मोशन या अभ्यास” के रूप में परिभाषित करता है, को 2022 के लिए
पिक्चर आर्ट्स एं ड साइं सेज द्वारा जारी किए गए थे। वर्ष का शब्द नामित किया गया था।
• भारतीय-तेलु गु फिल्म आरआरआर ने एमएम केरावनी द्वारा • फर्जी खबरों, राजनीतिक अभियानों के सत्य के बाद के युग में
रचित अपने उत्साहित गीत “नातु नातु” के लिए गोल्डन ग्लोब यह शब्द लोकप्रिय हो गया है।

94 I प्रबोधन Download Monthly Articulate Magazine


बहुविकल्पीय प्रश्न
5. 'नियम 267' के संदर्भ में निम्नलिखित कथनों पर विचार
1. 'पुनर्विचार याचिका' की अवधारणा के दायरे में है:
कीजिये:
(a) अनुच्छेद 137
1. एक सांसद को इसके तहत मामले पर चर्चा करने के लिए
(b) अनुच्छेद 324
2. राज्यसभा के सभापति की मंजूरी ले नी होती है।
(c) अनुच्छेद 142
3. यदि किसी मुद्दे को नियम 267 के तहत स्वीकार किया
(d) अनुच्छेद 280 जाता है, तो यह दर्शाता है कि यह दिन का सबसे महत्वपूर्ण
राष्ट्रीय मुद्दा है।

2. निम्नलिखित जोड़ियों पर विचार करें: ऊपर दिए गए कथनों में से कौन सा/से सही है/हैं?

सामुदायिक : राज्य (a) केवल 1

1. भोगटा : उत्तराखंड (b) केवल 2

2. दारलोंग : अंडमान और निकोबार (c) 1 और 2 दोनों

3. बेट्टा-कुरुबा : झारखंड (d) न तो 1 और न ही 2

4. नारिकोरवन : तमिलनाडु

ऊपर दिए गए कितने युग्म सही सुमेलित हैं? 6. हाल ही में संपन्न फीफा विश्व कप के संदर्भ में निम्नलिखित
कथनों पर विचार कीजिएः
(a) केवल एक जोड़ी
1. उपयोग किए गए फुटबॉल का नाम "अल रिहाला" था जिसे
(b) केवल दो जोड़ी विशेष रूप से फीफा 2022 के लिए डिज़ाइन किया गया था।
(c) केवल तीन जोड़े 2. लाईब आधिकारिक शुभंकर था।
(d) सभी चार जोड़े 3. सबसे ज्यादा गोल करने के लिए लियोनेल मेसी को
गोल्डन बूट से नवाजा गया।

3. "जी20 कॉमन फ्रेमवर्क " के संदर्भ में निम्नलिखित कथनों पर ऊपर दिए गए कथनों में से कौन सा/से सही है/हैं?
विचार करें: (a) 1 और 2 केवल
1. यह पेरिस क्लब के साथ मिलकर G20 द्वारा समर्थित एक (b) 2 और 3 केवल
पहल है।
(c) केवल1 और 3।
2. यह कम आय वाले देशों को अस्थिर ऋण के साथ समर्थन
करने की एक पहल है। (d) उक्त में से कोई भी सही नहीं है।

ऊपर दिए गए कथनों में से कौन-सा/से सही है/हैं

(a) केवल 1 7. 'ओपन नेटवर्क फॉर डिजिटल कॉमर्स (ONDC)' निम्नलिखित


में से किस मंत्रालय के अधीन है ?
(b) केवल 2
(a) वाणिज्य मंत्रालय़
(c) 1 और 2 दोनों
(b) उपभोक्ता मामले , खाद्य और सार्वजनिक वितरण मंत्रालय
(d) न तो 1 और न ही 2
(c) कपड़ा मंत्रालय

(d) शिक्षा मंत्रालय


4. निम्नलिखित में से कौन-सा/से 'मार्केट इन्फ्रास्ट्रक्चर संस्थान'
है /हैं?

1. स्टॉक एक्सचेंजों 8. निम्नलिखित जोड़ियों पर विचार करें:

2. डिपॉजिटरी जीआई टै ग उत्पाद राज्य

3. समाशोधन गृह 1. ओनाटु कारा एलु तमिलनाडु

नीचे दिए गए कूट का प्रयोग कर सही उत्तर चुनिए: 2. गमोचा असम

(a) 1 और 2 केवल 3. कोडुंगलू र पोट्टु वेलारी केरल

(b) केवल 2 ऊपर दिए गए कितने युग्म सही सुमेलित हैं?

(c) 2 और 3 केवल (a) केवल एक जोड़ी

(d) 1, 2 और 3 (b) केवल दो जोड़ी

Download Monthly Articulate Magazine प्रबोधन | 95


(c) केवल तीन जोड़ी 13. 'कोएलिशन फॉर नेचर' के संदर्भ में, निम्नलिखित कथनों पर
विचार करें:
(d) कोई भी जोड़ा नहीं
1. इसे ग्लोबल बायोडायवर्सि टी फ्रेमवर्क (जीबीएफ) के
कार्यान्वयन के लिए विकसित किया गया है।
9. 'वासनार अरेंजमेंट (डब्ल्यूए)' के संदर्भ में, निम्नलिखित
2. गठबंधन का नेतृत्व काबो वर्डे, समोआ और सेशेल्स कर
कथनों पर विचार करें:
रहे हैं।
1. भारत इसका संस्थापक सदस्य है।
ऊपर दिए गए कथनों में से कौन सा/से सही है/हैं?
2. यह एक बहुपक्षीय प्रौद्योगिकी नियंत्रण समझौता है।
(a) केवल 1
ऊपर दिए गए कथनों में से कौन सा/से सही है/हैं?
(b) केवल 2
(a) केवल 1
(c) 1 और 2 दोनों
(b) केवल 2
(d) न तो 1 और न ही
(c) 1 और 2 दोनों

(d) न तो 1 और न ही 2
14. 30X30 लक्ष्य किसके द्वारा शुरू किया गया है :

(a) प्रकृति और लोगों के लिए उच्च महत्वाकांक्षा गठबंधन


10. 'ग्रुप ऑफ फ्रेंड्स' पहल शुरू की गई है: (HAC)।
(a) शांति सैनिकों के खिलाफ अपराधों के लिए जवाबदेही को (b) WHO
बढ़ावा देना
(c) ग्रीनपीस इं डिया
(b) परमाणु निरस्त्रीकरण को बढ़ावा देना
(d) प्रकृति के लिए विश्वव्यापी निधि
(c) जलवायु परिवर्तन के लिए छोटे द्वीप विकासशील राज्यों
को सहायता प्रदान करना
15. समाचारों में प्राय: देखा जाने वाला शब्द 'डिजीज एक्स' किससे
(d) अफगानिस्तान को खाद्य सहायता प्रदान करने के लिए
संबंधित है ?

(a) अज्ञात रोगज़नक़ जो अगले महामारी का कारण बन


11. 'आईएनएस मोरमुगाओ' के संदर्भ में, निम्नलिखित कथनों पर सकता है।
विचार करें:
(b) SARSCov का संस्करण।
1. यह केवल सतह से हवा में मार करने वाली मिसाइलों से
(c) क्रीमियन-कांगो रक्तस्रावी बुखार
लै स है।
(d) इबोला वायरस रोग
2. इसे मझगांव डॉक शिपबिल्डर्स लिमिटेड ने बनाया है।

ऊपर दिए गए कथनों में से कौन सा/से सही नहीं है/हैं?


16. 'जेमिनिड्स उल्कापात' के संदर्भ में, निम्नलिखित कथनों पर
(a) केवल 1
विचार करें;
(b) केवल 2
1. उल्काएं धूमकेतुओ ं के टु कड़े हैं।
(c) 1 और 2 दोनों
2. वे तेज गति से पृथ्वी के वायुमंडल में प्रवेश करते हैं और
(d) न तो 1 और न ही 2 जलकर 'बौछार' बनाते हैं।

ऊपर दिए गए कथनों में से कौन सा/से सही है/हैं?


12. 'सतह जल और महासागर स्थलाकृति (SWOT)' के संदर्भ में, (a) केवल 1
निम्नलिखित कथनों पर विचार करें:
(b) केवल 2
1. यह दुनिया के महासागरों, झीलों और नदियों का व्यापक
(c) 1 और 2 दोनों
सर्वेक्षण करता है।
(d) न तो 1 और न ही 2
2. यह उन्नत माइक्रोवेव रडार तकनीक का उपयोग करता है।

ऊपर दिए गए कथनों में से कौन सा/से सही है/हैं?


17. समाचारों में प्राय: देखा जाने वाला शब्द 'WALLABY' संबंधित
(a) केवल 1
है:
(b) केवल 2
(a) रेडियो दूरबीन
(c) 1 और 2 दोनों
(b) गोल्डीलॉक्स जोन
(d) न तो 1 और न ही 2

96 I प्रबोधन Download Monthly Articulate Magazine


(c) उल्कापिंड के लोगों के हितों की रक्षा के लिए कानून बनाने के लिए
अलग-अलग स्वायत्तता प्राप्त है।
(d) छोटा तारा
ऊपर दिए गए कथनों में से कौन सा/से सही है/हैं?

(a) केवल 1
18. 'अष्टाध्यायी' किसके द्वारा रचित ग्रन्थ है ?
(b) केवल 2
(a) पाणिनी
(c) 1 और 2 दोनों
(b) कालिदास
(d) न तो 1 और न ही 2
(c) चरक

(d) बाणभट्ट
23. नई दिल्ली अंतर्राष्ट्रीय मध्यस्थता केंद्र (NDIAC) के संदर्भ में,
निम्नलिखित कथनों पर विचार करें:
19. खाड़ी सहयोग परिषद के संदर्भ में निम्नलिखित कथनों पर
1. इसकी स्थापना 2019 में नई दिल्ली अंतर्राष्ट्रीय मध्यस्थता
विचार कीजिये:
केंद्र (NDIAC) अधिनियम के तहत की गई थी।
1. इसकी स्थापना 1981 में हुई थी।
2. इसने 1995 में स्थापित इं डिया इं टरनेशनल आर्बिट्रेशन
2. इसके आठ सदस्य देश हैं। सेंटर का स्थान लिया।
3. इराक खाड़ी सहयोग परिषद का सदस्य है। 3. यह वैकल्पिक विवाद समाधान में अनुसंधान और प्रशिक्षण
ऊपर दिए गए कथनों में से कौन सा/से सही है/हैं? को बढ़ावा देने में मदद करता है।

(a) केवल 1 ऊपर दिए गए कथनों में से कौन सा/से सही है/हैं?

(b) केवल 2 और 3 (a) केवल 1 और 2

(c) केवल 1 और 3 (b) केवल 2 और 3

(d) केवल 3 (c) केवल 1 और 3

(d) 1, 2 और 3

20. 'ग्रेटर टिप्रालैं ड' का संबंध है:

(a) त्रिपुरा 24. ग्रेट इं डियन बस्टर्ड के संदर्भ में, निम्नलिखित कथनों पर विचार
करें:
(b) मेघालय
1. GIB को घास के मैदान की प्रमुख पक्षी प्रजाति माना जाता
(c) सिक्किम
है।
(d) इनमे से कोई भी नहीं
2. यह गुजरात का राजकीय पक्षी है।

3. यह प्रजातियों की IUCN लाल सूची में गंभीर रूप से


21. विश्व मृदा दिवस (डब्ल्यूएसडी) के बारे में निम्नलिखित में से संकटग्रस्त के रूप में सूचीबद्ध है।
कौन सा कथन सत्य नहीं है?
ऊपर दिए गए कथनों में से कौन सा/से सही है/हैं?
(a) WSD हर साल 5 दिसंबर को मनाया जाता है।
(a) 1 और 2 केवल
(b) WSD 2022 की थीम "मानव जाति को जीवित रखने के
(b) 2 और 3 केवल
लिए मिट्टी को जीवित रखें" है।
(c) केवल 1 और 3
(c) WSD की सिफारिश इं टरनेशनल यूनियन ऑफ सॉइल
साइं सेज (IUSS) द्वारा वियना में मुख्यालय से की गई थी। (d) 1, 2 और 3

(d) किसी न किसी रूप में मिट्टी का क्षरण भारत के कुल भूमि
क्षेत्र का लगभग 29% प्रभावित करता है। 25. जैविक विविधता पर संयुक्त राष्ट्र सम्मेलन (CBD) के बारे में
निम्नलिखित में से कौन सा कथन सत्य है ?

22. भारत में छठी अनुसूची के बारे में निम्नलिखित कथनों पर 1. यह एक अंतरराष्ट्रीय संधि है जिसे 1992 में अपनाया गया
विचार करें: था।

1. भारतीय संविधान के अनुच्छेद 244 में छठी अनुसूची का 2. यह अपने हस्ताक्षरकर्ताओ ं पर कानूनी रूप से बाध्यकारी
प्रावधान है। है।

2. छठी अनुसूची के तहत, निर्वाचित प्रतिनिधियों द्वारा 3. भारत ने 1994 में कन्वेंशन पर हस्ताक्षर किए।
प्रशासित स्वायत्त जिलों और परिषदों को अनुसूचित जाति 4. CBD को संयुक्त राष्ट्र पर्यावरण कार्यक्रम (UNEP) द्वारा

Download Monthly Articulate Magazine प्रबोधन | 97


प्रशासित किया जाता है। (c) 1 और 2 दोनों

नीचे दिए गए कूट का प्रयोग कर सही उत्तर चुनिए: (d) न तो 1 और न ही 2

(a) केवल 1, 2 और 3

(b) केवल 1, 3 और 4 30. बी सी चट्टोपाध्याय का प्रसिद्ध उपन्यास "आनंदमठ"


निम्नलिखित में से किस विद्रोह से प्रेरित था?
(c) केवल 2 और 3
(a) पाइका विद्रोह
(d) 1, 2, 3 और 4
(b) मुंडा विद्रोह

(c) सन्यासी विद्रोह


26. हाल ही में हवाई अड्डों पर फेसले स एं ट्री की सुविधा के लिए
सरकार द्वारा कौन सा ऐप लॉन्च किया गया है? (d) कोल विद्रोह

1. दिशा ऐप

2. अतिथि ऐप 31. निम्न में से कौन दुर्ल भ रोगों की विशेषता नहीं है ?

3. सेल्फस्कैन ऐप (a) यह अक्सर आनुवंशिक विकार होते हैं।

4. डिजीयात्रा ऐप (b) यह अक्सर जीर्ण और प्रगतिशील होते हैं

(c) उनके पास अक्सर प्रभावी उपचार या इलाज की कमी होती


है
27. नई चेतना अभियान के संदर्भ में निम्नलिखित कथनों पर
विचार कीजिएः (d) यह आम हैं और आसानी से निदान किए जाते हैं

1. इसका उद्देश्य महिलाओ ं को हिं सा को पहचानने और रोकने


के लिए तैयार करना और उन्हें उनके अधिकारों के बारे में
32. 'किफायती और मध्य-आय आवास के लिए विशेष विं डो
जागरूक करना है।
(SWAMIH)' के बारे में निम्नलिखित कथनों पर विचार करें:
2. इसे महिला एवं बाल विकास मंत्रालय द्वारा लॉन्च किया
1. इसका उद्देश्य स्ट्रेस्ड, ब्राउनफील्ड और रेरा-पंजीकृत
गया है।
आवासीय परियोजनाओ ं को पूरा करने के लिए प्राथमिकता
ऊपर दिए गए कथनों में से कौन सा/से सही नहीं है/हैं? ऋण वित्तपोषण प्रदान करना है।

(a) केवल 1 2. यह एक श्रेणी II वैकल्पिक निवेश कोष (एआईएफ) है।

(b) केवल 2 ऊपर दिए गए कथनों में से कौन सा/से सही नहीं है/हैं?

(c) 1 और 2 दोनों (a) केवल 1

(d) न तो 1 और न ही 2 (b) केवल 2

(c) 1 और 2 दोनों

28. हाल ही में समाचारों में रहा शब्द “मिशन प्रारंभ” निम्नलिखित (d) न तो 1 और न ही 2
में से किससे संबंधित है ?

1. इसरो का डीप स्पेस मिशन


33. निर्यातित उत्पादों पर शुल्कों और करों की छूट (RoDTEP)
2. विक्रम एस रॉकेट का प्रक्षेपण (RoDTEP) योजना का मुख्य उद्दे श्य क्या है ?

3. असम में तेल क्षेत्रों की खोज के लिए मिशन मोड योजना (a) भारतीय वस्तुओ ं और सेवाओ ं के निर्यात में वृद्धि करना

4. उत्तरी राज्यों में महिला एसएचजी को बढ़ावा देने की पहल (b) आयातित वस्तुओ ं और सेवाओ ं की लागत को कम करने
के लिए

(c) सरकार के राजस्व में वृद्धि करने के लिए


29. 'प्रधानमंत्री आवास योजना ग्रामीण' के संबंध में निम्नलिखित
कथनों पर विचार करें: (d) चीन से तैयार माल की डाम्पिं ग का मुक़ाबला करने के
लिए।
1. यह योजना अप्रैल 2016 में शुरू की गई थी।

2. ग्रामीण विकास मंत्रालय कार्यान्वयन के लिए जिम्मेदार


है। 34. निम्नलिखित में से कौन से देश संयुक्त राष्ट्र सुरक्षा परिषद
(यूएनएससी) के अस्थायी सदस्य हैं ।
ऊपर दिए गए कथनों में से कौन सा/से सही है/हैं?
(a) अल्बानिया, ब्राजील, गैबॉन
(a) केवल 1
(b) गैबॉन इं डिया, ऑस्ट्रेलिया
(b) केवल 2

98 I प्रबोधन Download Monthly Articulate Magazine


(c) केन्या, मैक्सिको, सऊदी अरब 39. गश्त-ए इरशाद का हाल ही में समाचार में उल्ले ख किया गया
था, यह एक है :
(d) संयुक्त अरब अमीरात, नॉर्वे, स्वीडन
(a) आईएसआईएस की एक आतंकवादी इकाई

(b) तालिबान के पुलिस बल ने अफगानिस्तान पर शासन


35. बेस एडिटिं ग तकनीक के संदर्भ में निम्नलिखित में से कौन से
किया
कथन सही हैं?
(c) पाकिस्तान में आईएसआई का नया विंग
1. आधार आनुवंशिक कोड के निर्माण खंड हैं।
(d) नैतिकता पुलिस ईरान में
2. आधार संपादन आधार की आणविक संरचना को बदलता
है और इस प्रकार आनुवंशिक निर्देशों को बदलता है।

3. इसका उपयोग मनुष्यों में कैंसर को ठीक करने के लिए 40. फर्स्ट लॉस डिफॉल्ट गारंटी (FLDG) का समाचार में उल्ले ख
नहीं किया जा सकता है। किया गया था, यह इसके साथ जुड़ा हुआ है :

नीचे दिए गए कूट का प्रयोग कर सही उत्तर चुनिए: (a) फिनटे क और बैंकों के बीच ऋण देने का मॉडल

(a) 1 और 2 केवल (b) UNFCCC द्वारा जलवायु बीमा योजना

(b) 2 और 3 केवल (c) श्रीलं का के लिए आईएमएफ बेलआउट पैकेज

(c) केवल 1 और 3 (d) अल्प विकसित देशों को पोस्ट-कोविड ऋण राहत।

(d) 1, 2 और 3

41. बेलगावी का हाल ही में समाचारों में उल्ले ख किया गया था, यह
निम्नलिखित में से किस राज्य के बीच का सीमावर्ती जिला है?
36. निम्नलिखित युग्मों पर विचार कीजिएः
(a) महाराष्ट्र और कर्नाटक
अभ्यास में भाग ले ने वाले देश
(b) आंध्र प्रदेश और तेलं गाना
1. इं ड-इं डो भारत समन्वित गश्ती : भारत-इं डोनेशिया
(c) महाराष्ट्र और मध्य प्रदेश
2. काजिं द-22 : भारत और कजाकिस्तान
(d) तेलं गाना और महाराष्ट्र
3. युद्ध-आभास : भारत और रूस

ऊपर दिए गए युग्मों में से कौन-सा/से सही सुमेलित है/हैं?


42. ChatGPT का हाल ही में समाचारों में उल्ले ख किया गया था,
(a) केवल एक जोड़ी
यह एक है :
(b) केवल दो जोड़ी
(a) आर्टिफिशियल इं टेलिजेंस चैटबॉट
(c) केवल तीन जोड़ी
(b) रैंसमवेयर का प्रकार
(d) सभी जोड़ी
(c) सोशल मीडिया के लिए ओपन-सोर्स सॉफ्टवेयर

(d) आतंकवादी द्वारा मैलवेयर हमलों की जांच के लिए


37. जनरल नेटवर्क एक्सेस रेगुलेशन का हाल ही में समाचारों में एफबीआई उपकरण
उल्ले ख किया गया था, यह संबंधित है:

(a) 5जी टे क्नोलॉजीज


43. किसी राजनीतिक दल को राष्ट्रीय दल के रूप में मान्यता
(b) फाइबर ब्रॉडबैंड इं टरनेट देने के लिए निम्नलिखित में से कौन-सा मानदंड पूरा करना
(c) टे लीविजन समाचार प्रसारण आवश्यक है ?

(d) अंतरराज्यीय विद्युत संचरण 1. यदि यह छह या अधिक राज्यों में 'मान्यता प्राप्त' है।

2. यदि इसके उम्मीदवारों ने कम से कम 4 राज्यों (नवीनतम


लोकसभा या विधानसभा चुनावों में) में कुल वैध मतों
38. श्वेत पत्र क्रांति का संबंध है :
का कम से कम 6% प्राप्त किया है और पार्टी के पिछले
(a) तेहरान में महिलाओ ं के अधिकार लोकसभा चुनावों में कम से कम 4 सांसद हैं।
(b) चीन में लॉकडाउन के विरोध में प्रदर्शन 3. अगर उसने कम से कम 3 राज्यों से लोकसभा में कुल
(c) बुल्गारिया में लोकतंत्र समर्थक विरोध प्रदर्शन सीटों में से कम से कम 4% सीटें जीती हैं।

(d) सीरिया में लोकतंत्र संकट नीचे दिए गए कूट का प्रयोग कर सही उत्तर चुनिए:

(a) केवल 1 और 2

(b) केवल 2

Download Monthly Articulate Magazine प्रबोधन | 99


(c) केवल 1 और 3 47. असम की सीमा किसके साथ लगती है

(d) 1, 2 और 3 1 अरुणाचल प्रदेश

2. मेघालय

44. हाल ही में, यूएसए ने ईगल अधिनियम पारित किया है , यह 3. मिजोरम


इससे संबंधित है:
4. नागालैं ड
(a) अफगानिस्तान को वित्तीय मदद
5. त्रिपुरा
(b) पाकिस्तान को वित्तीय मदद
6. पश्चिम बंगाल
(c) गैर-अमेरिकी नागरिकों को ग्रीन कार्ड
7. सिक्किम
(d) LGBTQ विवाहों को वैध बनाना
सही उत्तर चुने:

(a) 1,2,3,4,5,6
45. ग्लोबल कॉम्बैट एयर प्रोग्राम (GCAP) एक लड़ाकू जेट
(b) 2,4,5,6,7
विकास कार्यक्रम है जिसमें निम्नलिखित में से कौन सा देश
शामिल है? (c) 6,7,5,2,1

1. जापान (d) 3,4,5,6,7

2. भारत

3. इटली 48. समाचारों में प्राय: देखा जाने वाला शब्द 'डॉक्सिंग' संबंधित है:

4. यूके (a) किसी की व्यक्तिगत जानकारी को ऑनलाइन प्रकट


करने का कार्य।
नीचे दिए गए कूट का प्रयोग कर सही उत्तर चुनिए:
(b) साइबर हमला
(a) केवल 1 और 2
(c) चीनी घुसपैठ का मुकाबला करने के लिए सैन्य रणनीति
(b) केवल 1, 3 और 4
(d) नई पक्षी प्रजातियाँ
(c) केवल 1 और 3

(d) 1, 2, 3 और 4
49. हाल ही में, बर्डवॉचर्स ने किस राज्य में व्रेन बैबलर्स की एक नई
प्रजाति की खोज की है ?
46. ​​निम्नलिखित में से कौन से देश G7 और G20 दोनों संगठनों के
(a) नागालैं ड
सदस्य हैं?
(b) मेघालय
1. रूस
(c) सिक्किम
2. कनाडा
(d) अरुणाचल प्रदेश
3. फ्रांस

4. इटली
50. यूनेस्को की अस्थायी सूची के निम्नलिखित युग्मों पर विचार
5. चीन
करें:
नीचे दिए गए कूट का प्रयोग कर सही उत्तर चुनिए:
1. मोढे रा सूर्य मंदिर गुजरात
(a) केवल 1, 3 और 5
2. वडनगर राजस्थान
(b) केवल 2, 4 और 5
3. उनाकोटि त्रिपुरा
(c) केवल 2, 3 और 4
दिए गए युग्मों में से कौन-सा सही सुमेलित है:
(d) 1, 2, 3, 4 और 5
(a) केवल 1 जोड़े

(b) केवल 2 जोड़े

(c) केवल 3 जोड़े

(d) उपर्युक्त में से कोई नहीं

100 I प्रबोधन Download Monthly Articulate Magazine


उत्तर बॉक्स:

1 (a) 6 (a) 11 (a) 16 (c) 21 (b) 26 (d) 31 (d) 36 (b) 41 (a) 46 (c)
2 (a) 7 (a) 12 (c) 17 (a) 22 (a) 27 (a) 32 (d) 37 (d) 42 (a) 47 (a)
3 (c) 8 (b) 13 (c) 18 (a) 23 (c) 28 (b) 33 (a) 38 (b) 43 (b) 48 (a)
4 (d) 9 (b) 14 (a) 19 (a) 24 (c) 29 (c) 34 (a) 39 (d) 44 (c) 49 (d)
5 (c) 10 (a) 15 (a) 20 (a) 25 (d) 30 (b) 35 (a) 40 (a) 45 (b) 50 (b)

Enroll Now

Download Monthly Articulate Magazine प्रबोधन | 101

You might also like